Está en la página 1de 243

Solucionario: Vibraciones y Ondas de A. P.

French
Mauricio Vargas Villegas
2018

1
Índice
1. Movimientos periódicos 3

2. Superposición de movimientos 16

3. Vibraciones libres de los sistemas fı́sicos 25

4. Vibraciones forzadas y resonancia 67

5. Osciladores acoplados y modos normales 105

6. Modos normales de sistemas continuos. Análisis de Fourier 150

7. Ondas progresivas 185

8. Efectos debido a los lı́mites e interferencias 223

2
1. Movimientos periódicos
1. Consideremos un vector z definido por la ecuación z = z1 z2 , siendo z1 =
a + ib y z2 = c + id.
a) Demostrar que la longitud de z es igual al producto de las longi-
tudes de z1 y z2 .
b) Demostrar que el ángulo comprendido entre z y el eje x, es la suma
que forma por separado z1 y z2 con el eje x.

Solución

a)
√ Se hallan las longitudes
√ de los vectores individuales, resultando: ||z1 || =
a2 + b2 y ||z2 || = c2 + d2 , y su producto es,
p p p
||z1 ||||z2 || = a2 + b2 c2 + d2 = (a2 + b2 )(c2 + d2 )
p
= (a2 + b2 )(c2 + d2 )
Ahora se halla el valor la longitud de z = z1 z2 = (a + ib)(c + id) =
ac + iad + ibc − bd (teniendo en cuenta que i2 = −1). Reorganizando el
vector en las partes reales y complejas se tiene que z = (ac−bd)+i(ad+bc).
Entonces la longitud es:
p
||z|| = (ac − bd)2 + (ad + bc)2
p
= (a2 c2 + b2 d2 − 2acbd) + (a2 d2 + b2 c2 + 2adbc)
donde se cancela −2acbd con +2adbc, resultando:
p p
||z|| = a2 c2 + b2 d2 + a2 d2 + b2 c2 = a2 (c2 + d2 ) + b2 (d2 + c2 )
p
= (a2 + b2 )(c2 + d2 ) = ||z1 ||||z2 ||
Demostrando que ||z|| = ||z1 ||||z2 ||.

b) Usando el hecho de que a = z1 cos(θ1 ), b = z1 sin(θ1 ), c = z2 cos(θ2 ) y


que d = z2 sin(θ2 ), entonces:
z1 = z1 cos(θ1 ) + iz1 sin(θ1 ) = z1 [cos(θ1 ) + i sin(θ1 )]
z2 = z2 cos(θ2 ) + iz2 sin(θ2 ) = z2 [cos(θ2 ) + i sin(θ2 )]
y por tanto,
z1 z2 = z1 [cos(θ1 ) + i sin(θ1 )]z2 [cos(θ2 ) + i sin(θ2 )]
= z1 z2 [cos(θ1 ) cos(θ2 ) + i sin(θ1 ) cos(θ2 ) + i cos(θ1 ) sin(θ2 ) − sin(θ1 ) sin(θ2 )]
= z1 z2 {cos(θ1 ) cos(θ2 ) − sin(θ1 ) sin(θ2 ) + i[sin(θ1 ) cos(θ2 ) + cos(θ1 ) sin(θ2 )]}
= z1 z2 [cos(θ1 + θ2 ) + i sin(θ1 + θ2 )]
Pero z = z1 z2 , por tanto z = z1 z2 [cos(θ1 + θ2 ) + i sin(θ1 + θ2 )], lo que
demuestra lo pedido.

3
z1
2. Consideremos un vector z definido por la ecuación z = z2 (z2 6= 0), siendo
z1 = a + ib y z2 = c + id.
a) Demostrar que la longitud de z es igual al cociente de las longitudes
de z1 y z2 .
b) Demostrar que el ángulo comprendido entre z y el eje x, es la resta
que forma por separado z1 y z2 con el eje x.

Solución

a)
√ Se hallan las longitudes
√ de los vectores individuales, resultando: ||z1 || =
a2 + b2 y ||z2 || = c2 + d2 6= 0, y su cociente es,
√ r
||z1 || a2 + b2 a2 + b2
= √ =
||z2 || c2 + d2 c2 + d2

Pero z = a+ib c−id


c+id . Se multiplica el cociente por c−id , resultando
  
a + ib c − id (a + ib)(c − id) (ac + ibc) + (−iad + bd)
z= = =
c + id c − id (c + id)(c − id) c2 − (id)2
(ac + bd) + i(bc − ad) 1
= = 2 [(ac + bd) + i(bc − ad)]
c2 + d2 c + d2

cuya longitud es,


1 p
||z|| = 2 (ac + bd)2 + (bc − ad)2
c + d2
1 p 2 2
= 2 (a c + b2 d2 + 2acbd) + (b2 c2 + a2 d2 − 2bcad)
c + d2
1 p 2 2 2 d2 + b2 c2 + a2 d2 =
1 p 2
= 2 a c + b (a + b2 )(c2 + d2 )
c + d2 c2 + d2
p s r
(a2 + b2 )(c2 + d2 ) (a2 + b2 )(c2 + d2 ) a2 + b2 ||z1 ||
= 2 2
= 2 2 2
= =
c +d (c + d ) c2 + d2 ||z2 ||
||z1 ||
Demostrando que ||z|| = ||z2 || .

b) Usando el hecho de que a = z1 cos(θ1 ), b = z1 sin(θ1 ), c = z2 cos(θ2 ) y


que d = z2 sin(θ2 ), entonces:
z1 = z1 cos(θ1 ) + iz1 sin(θ1 ) = z1 [cos(θ1 ) + i sin(θ1 )]
z2 = z2 cos(θ2 ) + iz2 sin(θ2 ) = z2 [cos(θ2 ) + i sin(θ2 )]
z1
y por tanto (con z = z2 ),
 
z1 [cos(θ1 ) + i sin(θ1 )] z1 cos(θ1 ) + i sin(θ1 )
z= =
z2 [cos(θ2 ) + i sin(θ2 )] z2 cos(θ2 ) + i sin(θ2 )

4
cos(θ2 )−i sin(θ2 )
Multiplicando por cos(θ2 )−i sin(θ2 ) , se llega a
  
z1 cos(θ1 ) + i sin(θ1 ) cos(θ2 ) − i sin(θ2 )
z=
z2 cos(θ2 ) + i sin(θ2 ) cos(θ2 ) − i sin(θ2 )
 
z1 [cos(θ1 ) + i sin(θ1 )][cos(θ2 ) − i sin(θ2 )]
=
z2 [cos(θ2 ) + i sin(θ2 )][cos(θ2 ) − i sin(θ2 )]
 

 

 
z1 [cos(θ1 ) cos(θ2 ) + i sin(θ1 ) cos(θ2 )] + [−i cos(θ1 ) sin(θ2 ) + sin(θ1 ) sin(θ2 )] 

=
z2 

 cos2 (θ2 ) + sin2 (θ2 ) 


 | {z } 
=1
z1
= {[cos(θ1 ) cos(θ2 ) + sin(θ1 ) sin(θ2 )] + i[sin(θ1 ) cos(θ2 ) − cos(θ1 ) sin(θ2 )]}
z2
z1
Por tanto, z = z2 [cos(θ1 − θ2 ) + i[sin(θ1 − θ2 )]], lo que demuestra lo pe-
dido.
3. Demostrar que la multilicación de cualquier número complejo z por eiθ
puede describirse, en términos geométricos, como una rotación positiva en
el ángulo θ del vector representado por z sin alteración de su longitud.

Solución

Primer método

El vector z = x + iy se puede escribir como z = A cos(α) + iA sin(α). Al


multiplicarlo por eiθ que por la ecuación de Euler es eiθ = cos(θ)+i sin(θ),
queda:

zeiθ = [A cos(α) + iA sin(α)][cos(θ) + i sin(θ)]


= A cos(α) cos(θ) + iA sin(α) cos(θ) + iA cos(α) sin(θ) − A sin(α) sin(θ)
= A[cos(α) cos(θ) − sin(α) sin(θ)] + iA[sin(α) cos(θ) + cos(α) sin(θ)]
= A cos(α + θ) + iA sin(α + θ) = A[cos(α + θ) + i sin(α + θ)] = Aei(α+θ)

Donde se observa que se rota de forma positiva con un ángulo θ y la lon-


gitud A se mantiene.

Segundo método

Se escribe z = Aeiα , que al multiplicarlo por eiθ queda:

zeiθ = Aeiα eiθ = Aei(α+θ)

Demostrando lo pedido.
4. a) Si z = Aeiθ , deducir que dz = izdθ y explicar el significado de esta
relación en un diagrama vectorial.

5
√ √
b) Hallar los valores y direcciones de los vectores (2+i 3) y (2−i 3)2 .

Solución

a) Se calcula el diferencial de z, resultando dz = Adθeiθ = idθ Ae
|{z} y
z
por tanto dz = izdθ. Se observa que cuando se multiplica a z = x + iy por
i resulta:
iz = i(x + iy) = ix − y = −y + ix
Esto significa (ver el texto) que hay una rotación positiva de π2 (en contra
de las manecillas del reloj). Al estar multiplicado por un diferencial del
ángulo dθ, su magnitud se reduce a un diferencial (ver figura 1).

Figura 1: Figura problema 1.4a

√ √
b) Para el primer vector, xq 1 + iy1 = 2 + i 3 y entonces x1 = 2 y y1 = 3.
√ √ √
2 2
Su magnitud es ||z1 || = 2 + ( 3) = 4 + 3 = 7. Su dirección es
√ √ 
tan(θ1 ) = xy11 = 23 y entonces, θ1 = arctan 23 = 40.89o con el eje
horizontal. √
Para
√ el segundo
√ vector (2 −√i 3)2 , primero
√ se expande resultando: 2√ 2
+
2
(i 3) −2(2)i 3 = 4−3−i4 3 = 1−i4 3; y entonces x 2 = 1 y y2 = 4 3.
q √ p √
Su magnitud es: ||z2 || = 12 + (−4 3)2 = 1 + 16(3) = 1 + 48 = 7.
√ √ 
Su dirección es: tan(θ2 ) = xy22 = −41 3 y entonces, θ2 = arctan −4 3 =
−81.79o = 98.21o con el eje horizontal.
5. Para tomar las derivadas sucesivas de eiθ respecto a θ, basta multiplicar
por i:
d
Aeiθ = iAeiθ


6
Demostrar que esta propiedad sigue siendo válida si se utiliza la represen-
tación sinusoidal eiθ = cos(θ) + i sin(θ).

Solución

Se deriva eiθ = cos(θ) + i sin(θ), resultando:

d d
Aeiθ =

[A cos(θ) + iA sin(θ)] = −A sin(θ) + iA cos(θ)
dθ dθ
= i2 A sin(θ) + iA cos(θ) = iA[i sin(θ) + cos(θ)]
= iA[cos(θ) + i sin(θ)] = iAeiθ

Que demuestra lo pedido.


6. Dada la relación de Euler eiθ = cos(θ) + i sin(θ), hallar
a) La representación geométrica de e−iθ .
b) La representación exponencial de cos(θ).
c) La representación exponencial de sin(θ).

Solución

a) Teniendo en cuenta que eiθ = cos(θ)+i sin(θ), entonces: e−iθ = cos(−θ)+


i sin(−θ) = cos(θ) − i sin(θ). Esto hace el valor en el eje vertical cambie de
signo; su gráfica es:

Figura 2: Figura problema 1.6a

7
b) Teniendo en cuenta la relación de Euler:

e+iθ = cos(θ) + i sin(θ)


e−iθ = cos(θ) − i sin(θ)

Se suman las dos ecuaciones resultando,

e+iθ + e−iθ = cos(θ) + i sin(θ) + cos(θ) − i sin(θ) = 2 cos(θ)

Despejando se llega a la representación exponencial de la función coseno:

e+iθ + e−iθ
cos(θ) =
2

c) Teniendo en cuenta la relación de Euler:

e+iθ = cos(θ) + i sin(θ)


e−iθ = cos(θ) − i sin(θ)

Se restan las dos ecuaciones resultando,

e+iθ − e−iθ = cos(θ) + i sin(θ) − [cos(θ) − i sin(θ)] = 2i sin(θ)

Despejando se llega a la representación exponencial de la función seno:

e+iθ − e−iθ
sin(θ) =
2i
−iθ
+iθ
e+iθ −e−iθ
7. a) Justificar las fórmulas cos(θ) = e +e 2 y sin(θ) = 2i ,
utilizando los desarrollos en serie correspondientes.
b) Describir geometricamente las relaciones anteriores mediante dia-
gramas vectorial en el plano xy.

Solución

a) Teniendo en cuenta que:

(iθ)0 (iθ)1 (iθ)2 (iθ)3 (iθ)4 (iθ)5 (iθ)6


eiθ = + + + + + + + ···
0! 1! 2! 3! 4! 5! 6!
y que,

(−iθ)0 (−iθ)1 (−iθ)2 (−iθ)3 (−iθ)4 (−iθ)5 (−iθ)6


e−iθ = + + + + + + + ···
0! 1! 2! 3! 4! 5! 6!
(iθ)0 (iθ)1 (iθ)2 (iθ)3 (iθ)4 (iθ)5 (iθ)6
= − + − + − + + ···
0! 1! 2! 3! 4! 5! 6!

8
Al sumar las dos exponenciales,

(iθ)0 (iθ)1 (iθ)2 (iθ)3 (iθ)4 (iθ)5 (iθ)6


eiθ + e−iθ = + + + + + + + ···
0! 1! 2! 3! 4! 5! 6!
(iθ)0 (iθ)1 (iθ)2 (iθ)3 (iθ)4 (iθ)5 (iθ)6
+ − + − + − + + ···
0! 1! 2! 3! 4! 5! 6!
Se observa que se cancelan los términos con potencias impares y se dupli-
can los de potencias pares resultando,

(iθ)0 (iθ)2 (iθ)4 (iθ)6


eiθ + e−iθ = 2 +2 +2 +2 + ···
0! 2! 4! 6!
e−iθ + e−iθ (iθ)0 (iθ)2 (iθ)4 (iθ)6
= + + + + ···
2 0! 2! 4! 6!
Entonces,

eiθ + e−iθ (θ)0 (θ)2 (θ)4 (θ)6


= − + − + ···
2 | 0! 2! {z4! 6! }
cos(θ)

Resultando,
eiθ + e−iθ
= cos(θ).
2
Si se restan las dos exponenciales,

(iθ)0 (iθ)1 (iθ)2 (iθ)3 (iθ)4 (iθ)5 (iθ)6


eiθ − e−iθ = + + + + + + + ···
0! 1! 2! 3! 4! 5! 6!
(iθ)0 (iθ)1 (iθ)2 (iθ)3 (iθ)4 (iθ)5 (iθ)6
− + − + − + − + ···
0! 1! 2! 3! 4! 5! 6!
Se observa que se cancelan los términos con potencias pares y se duplican
los de potencias impares resultando,

(iθ)1 (iθ)3 (iθ)5


eiθ − e−iθ = 2 +2 +2 + ···
1! 3! 5!
eiθ − e−iθ (iθ)1 (iθ)3 (iθ)5
= + + + ···
2 1! 3! 5!
También,
eiθ − e−iθ (θ)1 (θ)3 (θ)5
=i −i +i + ···
2 1! 3! 5!
Entonces,
eiθ − e−iθ (θ)1 (θ)3 (θ)5
= − + + ···
2i | 1! 3! {z 5! }
sin(θ)

9
Resultando,
eiθ − e−iθ
= sin(θ)
2i
Que demuestra lo pedido.

b) Se observa en la figura 3 que los vectores en el eje complejo (i) se


cancelan, y los vectores en el eje real (horizontal) se suman. Lo que de-
muestra la primera relación.

Figura 3: Figura problema 1.7b

Se observa en la figura 4 que los vectores en el eje real (horizontal) se


cancelan, y los vectores en el eje complejo (i) se suman. Lo que demuestra
la segunda relación.

Figura 4: Figura problema 1.7b

8. Utilizando las representaciones vectoriales de sin(θ) y cos(θ), comprobar


las siguientes identidades trigonométricas:

10
a) sin2 (θ) + cos2 (θ) = 1.
b) cos2 (θ) − sin2 (θ) = cos(2θ).
c) 2 sin(θ) cos(θ) = sin(2θ).

Solución
−iθ

eiθ −e−iθ
a) Usando cos(θ) = e +e 2 y sin(θ) = 2i para elevarlas al cua-
drado y sumarlas se tiene que,
2  iθ 2
eiθ − e−iθ e + e−iθ

2 2
sin (θ) + cos (θ) = +
2i 2
2 2 2 2
eiθ − e−iθ eiθ + e−iθ eiθ + e−iθ − eiθ − e−iθ
 
=− + =
4 4 4
ei2θ + e−i2θ + 2eiθ e−iθ − e2iθ − e−2iθ + 2eiθ e−iθ
=
4
4eiθ e−iθ 4
= = =1
4 4
que demuestra lo pedido.
−iθ

eiθ −e−iθ
b) Usando cos(θ) = e +e 2 y sin(θ) = 2i para elevarlas al cua-
drado y restarlas se tiene,
2  iθ 2
eiθ + e−iθ e − e−iθ

cos2 (θ) − sin2 (θ) = −
2 2i
−iθ 2 −iθ 2
2 2
eiθ + e−iθ + eiθ − e−iθ

 iθ

e +e e −e
= + =
4 4 4
ei2θ + e−i2θ + 2eiθ e−iθ + e2iθ + e−2iθ − 2eiθ e−iθ
=
4
2 cos(2θ)
z }| {
2ei2θ + 2e−i2θ ei2θ + e−i2θ
= =
4 2
Llegando a:
cos2 (θ) − sin2 (θ) = cos(2θ)
que demuestra lo pedido.

11
eiθ +e−iθ eiθ −e−iθ
c) Multiplicando el doble de cos(θ) = 2 y sin(θ) = 2i ,
Diferencia de cuadrados
z }| {
eiθ + e −iθ
eiθ − e −iθ
(eiθ + e−iθ )(eiθ − e−iθ )
  
2 sin(θ) cos(θ) = 2 =
2 2i 2i
2i sin(2θ)
z }| {
e2iθ − e−2iθ
= = sin(2θ)
2i
Que demuestra lo pedido.
9. ¿Se puede pagar 20 céntimos de un objeto valorado por un matemático
con ii pesetas? (Recuérdese que eiθ = cos(θ) + i sin(θ).)

Solución

Se debe buscar un ángulo tal que el valor de eiθ sea i, para ası́ poderlo
elevar a la i. Esto es, el valor de θ debe ser tal que cos(θ) = 0 y sin(θ) = 1.
Por tanto usando θ = π2 queda,
π 2π π
ei 2 = i y por tanto, ii = ei 2 = e− 2 = 0.208

El resultado es 20.8 céntimos y por tanto no se puede pagar con 20 cénti-


mos de peseta.
2
d y 2
10. Comprobar que la ecuación diferencial dx 2 = −k y tiene por solución

y = A cos(kx) + B sin(kx) siendo A y B constantes arbitrarias. Demostrar


también que esta solución puede escribirse de la forma

y = C cos(kx + α) = C<[ei(kx+α) ] = <[(Ceiα )eikx ]

y expresar C y α en función de A y B.

Solución

Se deriva dos veces y = A cos(kx) + B sin(kx):


dy
= −kA sin(kx) + kB cos(kx)
dx
d2 y
= −k 2 A cos(kx) − k 2 B sin(kx) = −k 2 [A cos(kx) + B sin(kx)]] = −k 2 y
dx2 | {z }
y

Teniendo en cuenta el coseno del ángulo suma, cos(a + b) = cos(a) cos(b) −


sin(a) sin(b), se define A = C cos(α) y B = −C sin(α) resultando:

y = A cos(kx) + B sin(kx) = C cos(α) cos(kx) − C sin(α) sin(kx)


= C[cos(kx) cos(α) − sin(kx) sin(α)] = C cos(kx + α)

12
que es lo que se pedı́a demostrar.
Adicionalmente se observa que, A2 + B 2 = C 2 cos2 (α) + (−C)2 sin2 (α) =
C 2 [cos2 (α) + sin2 (α)] = C 2 .
Por tanto los valores de A, B y C son:

A = C cos(α),
B = −C sin(α),
p
C = A2 + B 2
−C sin(α)
Para hallar el valor de α se divide B entre A, BA = C cos(α) = − tan(α),
que lleva a:  
B
α = arctan − .
A

Por último teniendo en cuenta que z = eiβ = cos(β) + i sin(β) y que la


parte real de z es <[z] = cos(β) y la imaginaria es =[z] = sin(β), entonces
z = <[z] + i=[z]. Realizando la correspondencia con el resultado anterior,
se llega a que:

y = C<[ei(kx+α) ] = <[Cei(kx+α) ] = <[Ceikx eiα ] = < Ceiα eikx


  

Que demuestra lo pedido.


11. Una masa en el extremo de un muelle oscila con una amplitud de 5 cm y
una frecuencia de 1 Hz (ciclos por segundo). Para t = 0, la masa está en
su posición de equilibrio (x = 0).
a) Hallar las ecuaciones posibles que describen la posición de la masa
en función del tiempo, en la forma x = A cos(ωt + α), dando los valores
numéricos de A, ω y α.
dx d2 x 8
b) ¿Cuáles son los valores numéricos de x, dt y dt2 para t = 3 s?

Solución

a) La frecuencia que da el enunciado es la lineal ν y no la angular ω;


por tanto ω = 2πν = 2π rad s . La amplitud es A = 5 cm. Con estos dos
valores se construye la ecuación:

x = 5 cos(2πt + α)

qe con la condición inicial x(0) = 0, queda,

0 = 5 cos(α) 0 = cos(α)

Para que esta ecuación se cumpla, α debe tener valores impares de π2 ; esto
es α = (2n − 1) π2 = nπ − π2 para n = 1, 2, 3, · · · Ası́, las ecuaciones posibles
son: h  π i
xn = 5 cos 2πt + nπ − n = 1, 2, 3, · · ·
2

13
b) Se hallan la velocidad y la aceleración, resultando:
h  π i
xn (t) = 5 cos 2πt + nπ −
2
dxn (t) h  π i
= −10π cos 2πt + nπ −
dt 2
d2 xn (t) 2
h  π i
= −20π cos 2πt + nπ −
dt2 2
8
Para t = 3 s:
      
8 π 16π  π
xn |t= 8 = 5 cos 2π + nπ − = 5 cos + nπ −
3 3 2 3 2
      
dxn 8 π  16π  π
= −10π cos 2π + nπ − = −10π cos + nπ −
dt t= 8 3 2 3 2
3

d2 xn
     
2 8  π 
2 16π  π
= −20π cos 2π + nπ − = −20π cos + nπ −
dt2 t= 8 3 2 3 2
3

n = 1, 2, 3, · · ·

12. Un punto se mueve en una circunferencia con una velocidad constante de


50 cm
s . El perı́odo de una vuelta completa es 6 s. Para t = 0 la recta que
va del punto al centro de la circunferencia forma un ángulo de 30o = π6 rad
con el jeje de las x.
a) Obtener la ecuación de la coordenada x del punto en función del
tiempo, en la forma x = A cos(ωt + α), conocidos los valores numéricos de
A, ω y α.
dx d2 x
b) ¿Cuáles son los valores numéricos de x, dt y dt2 para t = 2 s?

Solución

a) La figura del estado de la partı́cula P en t = 0 es (ver figura 5). Donde


se observa que el ángulo que se forma con el eje de las x es en t = 0 y
por tanto es la constante de fase α = π6 rad. Con el perı́odo τ = 6 s, se
construye la frecuencia angular:
2π 2π π rad
ω= = =
τ 6 3 s
Con lo anterior se construye la ecuación,
π π
x = A cos t+
3 6
Derivando temporalmente para halla la velocidad:
dx π π π
= − A sin t+
dt 3 3 6

14
Figura 5: Figura problema 1.12

Pero en el enunciado se informa que la velocidad tiene un valor constante


de 50 cms , por tanto:
π π π
50 = − A sin t+
3 3 6
Para hallar el valor de la amplitud A, se tiene en cuenta que es cuando la
función seno toma su máximo valor. Esto es la ±1. Por tanto,
π 150
50 = ± A donde, A = ± cm
3 π
Donde se halla la ecuación de posición:
150 π π
x(t) = cos t+
π 3 6

b) Derivando para hallar la velocidad y la aceleración:


150 π π
x(t) = cos t+
π 3 6
dx(t) 150  π  π π π π
=− sin t+ = −50 sin t+
dt π 3 3 6 3 6
d2 x(t) 150  π 2 π π 150π π π
= − cos t + = − cos t +
dt2 π 3 3 6 9 3 6
Los valores son:
   
150 2π π 150 5π
x|t=2 = cos + = cos
π 3 6 π 6
   
dx
2π π 5π
= −50 sin + = −50 sin
dt t=2 3 6 6
2
   
d x 150π 2π π 150π 5π
= − cos + = − cos
dt2 t=2 9 3 6 9 6

15
2. Superposición de movimientos
 
1. Escribir las expresiones siguientes en la forma z = < Aei(ωt+α)
a) z = sin(ωt) + cos(ωt).
b) z = cos ωt − π3 − cos(ωt).


c) z = 2 sin(ωt) + 3 cos(ωt).
π

d) z = sin(ωt) − 2 cos ωt − 4 + cos(ωt).

Solución

Como lo que se busca es hallar la parte real de la exponencial, enton-


ces hay que reducir las expresiones a la función trigonométrica coseno,
porque: z = Aei(ωt+α) = A cos(ωt + α) +i A sin(ωt + α).
| {z } | {z }
<[z] =[z]

a) Para z = sin(ωt) + cos(ωt) se observa que los coeficientes son uni-


tarios; esto es: z = (1) sin(ωt) + (1) cos(ωt). Como se desea convertir a
coseno, entonces para convertir la expresión en cos(a+b) = cos(a)
 cos(b)−
sin(a) sin(b) se definen los unos como: √22 cos π4 y √22 sin π4 , donde se

 √
recuerda que, cos π4 = sin π4 = 22 . Entonces,


2 π 2 π
z = sin(ωt) + cos(ωt) = √ sin sin(ωt) + √ cos cos(ωt)
2 4 2 4
2 h  π   π i
= √ cos(ωt) cos + sin(ωt) sin
2 4 4
 
2  π 2 i(ωt− π4 )
= √ cos ωt − =< √ e
2 4 2

Donde A = √22 , (o racionalizando A = 2) y α = − π4 .

π

b) Para z = cos ωt − 3 − cos(ωt), se expande el binomio resultando:
π π
z = cos(ωt) cos + sin(ωt) sin − cos(ωt)
| {z 3 } | {z3 }

1 3
2 2

1 3
= cos(ωt) + sin(ωt) − cos(ωt)
2 2
Factorizando, √
1 3
z = − cos(ωt) + sin(ωt).
2 2

Entonces − 12 = A cos(α) y 23 = A sin(α) con el objetivo de formar el co-
seno suma. Resolviendo estas ecuaciones, primero se suman sus cuadrados

16
para hallar la amplitud:
 2 √ !2
1 3 2 2
− + = [A cos(α)] + [A sin(α)] → 1 = A2
2 2

Resultando A = 1. Finalmente dividiendo las ecuaciones entre si para


encontrar la constante de fase:

2
3
A sin(α) √ √ π
= → − 3 = tan(α) → α = arctan[− 3] = −
− 12 A cos(α) 3

c) Para z = 2 sin(ωt)+3 cos(ωt), se hace la correspondencia −A sin(α) = 2


y A cos(α) = 3. Resolviendo,

[−A sin(α)]2 + [A cos(α)]2 = 23 + 32 → A2 = 13



Y por tanto, A = 13. Dividiendo las ecuaciones,
 
−A sin(α) 2 2 2
= → − tan(α) = → α = − arctan
A cos(α) 3 3 3

d) Para z = sin(ωt) − 2 cos ωt − π4 + cos(ωt), se expande el trinomio




resultando,
 π
z = sin(ωt) − 2 cos ωt − + cos(ωt)
4 
π π
= sin(ωt) − 2 cos(ωt) cos − 2 sin(ωt) sin + cos(ωt)
4 4
√ ! √ !
2 2
= sin(ωt) − 2 cos(ωt) − 2 sin(ωt) + cos(ωt)
2 2
√ √
= sin(ωt) − 2 cos(ωt) − 2 sin(ωt) + cos(ωt)

Factorizando,
 √   √ 
z = 1 − 2 cos(ωt) + 1 − 2 sin(ωt)
| {z } | {z }
A cos(α) −A sin(α)

Sumando los cuadrados de las ecuaciones,


 √ 2 √ 2
1− 2 + 2 − 1 = [A cos(α)]2 + [A sin(α)]2 = A2

Se obtiene
√ 2 √ √  √
2 2−1 = A2 → A= 2 2−1 =2− 2

17
Se dividen las dos ecuaciones,

2−1 A sin(α)
√ = −1 = = tan(α) → tan(α) = −1
1− 2 A cos(α)

Y entonces α = arctan(−1) = − π4 = − π4 + π = 3π
4

2. Una partı́cula está sometida simultaneamente a tres movimientos armóni-


cos simples de la misma frecuencia y en dirección x. Si las amplitudes
son 0.25, 0.20 y 0.15 mm, respectivamente, y la diferencia de fase entre el
primero y el segundo es 45o , y entre el segundo y el tercero es 30o , hallar
la amplitud del desplazamiento resultante y su fase relativa respecto al
primer componente (de amplitud 0.25 mm).

Solución

El diagrama es:

Figura 6: Figura problema 2.2

Cómo se observa se puede solucionar a grandes rasgos de dos maneras.


Una forma vectorial o una escalar.
a) Forma vectorial. Se construyen los vectores A, B y C con las
amplitudes respectivas 0.25, 0.20 y 0.15 mm:

A = 0.25x̂
B = 0.20 cos(45o )x̂ + 0.20 sin(45o )ŷ
C = 0.15 cos(75o )x̂ + 0.15 sin(75o )ŷ

Se calcula la resultante haciendo la suma vectorial D = A + B + C,

D = [0.25 + 0.20 cos(45o ) + 0.15 cos(75o )] x̂ + [0.20 sin(45o ) + 0.15 sin(75o )] ŷ


= 0.430x̂ + 0.286ŷ

Cuya magnitud es:


p
D= 0.4302 + 0.2862 = 0.516 mm

18
Su orientación es:
0.286
tan(δ) = = 0.665
0.430
→ δ = arctan(0.665) = 33.63o respecto a la primer componente.

b) Forma escalar. Para hacerlo escalarmente, se construye un para-


lelogramo transladando la segunda componente a a y la primera compo-
nente a b, que da a lugar a una resultante R1. El diagrama es:

Figura 7: Figura problema 2.2. Resultante R1

Se usa la ley de Cosenos:

R12 = a2 + b2 + 2ab cos(45o )


s √
2
R1 = 0.202 + 0.252 + 2(0.20)(0.25) = 0.416 mm
2

La componente y de R1 es (Ver figura 7):

R1y = 0.20 sin(45o ) = 0.141 mm

La componente x de R1 es (Ver figura 7):

R1x = 0.25 + 0.20 cos(45o ) = 0.391 mm

Por tanto,
0.141
tan(α1) = = 0.361
0.391
Y de esta manera el ángulo α1 se halla con el valor:

α1 = arctan(0.361) = 19.83o

Como último paso se translada la tercera componente a c y se forma


una nueva resultante con R1, la cual se denomina R2 (ver Figura 8).
Se observa que el ángulo formado entre R1 y la tercera componente es:
75.00o − 19.83o = 55.17o . Con estos datos se puede calcular el valor de R2

19
Figura 8: Figura problema 2.2. Resultante R2

usando la Ley de cosenos:

R22 = R12 + c2 + 2(R1)(c) cos(55.17o )


p
R2 = 0.4162 + 0.1502 + 2(0.416)(0.150) cos(55.17o ) = 0.516 mm

La componente y de R2 es (ver figura 8):

R2y = 0.141 + 0.15 sin(75o ) = 0.286 mm

La componente x de R2 es (ver figura 8):

R2x = 0.391 + 0.15 cos(75o ) = 0.430 mm

Ası́, el valor de α2 es:


 
0.286
α2 = δ = arctan = 33.63o
0.430
Dando los mismos valores que se obtuvieron con el método vectorial.
3. Dos vibraciones sobre la misma recta vienen descritas por la ecuaciones

y1 = A cos(10πt)
y2 = A cos(12πt)

Hallar el perı́odo de batido y dibujar un esquema cuidadoso de la pertur-


bación resultante durante un perı́odo τ de la pulsación.

Solución

El problema es de una superposición paralela de osciladores (los dos están


en el eje y) con diferente fecuencia. Por tanto, se tienen dos frecuencias

20
angulares ω1 = 10π y ω2 = 12π. El perı́odo de batido se calcula con la
frecuencia de la envolvente, que es la resta de las dos frecuencias angulares
(ver texto). Por tanto,
ω2 − ω1 12π − 10π 2π
= =π → =π → τ =2s
2
| {z } 2 τ

τ

La figura de la perturbación resultante durante un perı́odo τ es:

Figura 9: Figura problema 2.3.

Donde usando una amplitud unitaria, la superposición está dada por la


función f (t) = 2 cos(πt) cos(11πt), y la envolvente está dada por 2 cos(πt).
4. Hallar la frecuencia ν del movimiento combinado en cada una de las si-
guientes vibraciones:

a) sin(2πt − 2) + cos(2πt)
 π
b) sin(12πt) + cos 13πt −
4
c) sin(3t) − cos(πt)

Primero que todo, la superposición de estos sociladores es paralela, debi-


do a que se suman directamente. Debido a que en el texto se superponen
cosenos, primero setransformará el seno a coseno usando el hecho de que,
sin(a) = cos a − π2 .

21
√ √
a) sin(2πt − 2) + cos(2πt) = cos 2πt − 2 − π2 + cos(2πt). Por tan-


to se desea superponer,
h √ π i
cos 2πt − 2+ + cos(2πt)
2
que son dos osciladores armónicos simples con igual frecuencia ω = 2π.
Teniendo en cuenta que ω = 2πν, entonces: ω = 2π = 2πν, resultando que
ν = 1 s−1 .

b) sin(12πt)+cos 13πt − π4 = cos 12πt − π2 +cos 13πt − π4 , que como


  

se observa son dos osciladores armónicos simples con diferente frecuencia.


En el texto se observa claramente que la superposición está dada por (sin
tener en cuenta las constantes de fase):
   
ω1 − ω2 ω1 + ω2
y = 2A cos t cos t
2 2

Reemplazando valores:
π   
25π
y = 2 cos t cos t = 2 cos (0.5πt) cos (12.5πt)
2 2

Donde la frecuencia de batido es de 0.5π = 2πνb dando

νb = 0.25 s−1

y la frecuencia del sistema es 12.5π = 2πνs dando

νs = 6.25 s−1 .

c) sin(3t) − cos(πt) = cos 3t − π2 − cos(πt), que como se observa son dos




osciladores armónicos simples con diferente frecuencia. La superposición


es (sin tener en cuenta las constantes de fase),
   
π−3 π+3
y = 2 cos t cos t
2 2
π−3
Donde la frecuencia de batido es de 2 = 2πνb dando

π−3
νb = = 0.01 s−1

π+3
y la frecuencia del sistema es 2 = 2πνs dando

π+3
νs = = 0.49 s−1 .

22
5. Dos vibraciones perpendiculares vienen descritas por las ecuaciones

x = 10 cos(5πt)
 π
y = 10 cos 10πt +
3
Construir la figura de Lissajous del movimiento combinado.

Solución

Describir en este solucionario el método gráfico es muy complicado. Se


ofrece la ecuación y su gráfica en los paquetes Mathematica y Geogebra.
La función es:

Mathematica: ParametricPlot[{10 Cos[5 Pi t], 10 Cos[10 Pi t + Pi/3]},


{t, 0 , Pi}]
Geogebra: Curve(10 cos(5πu), 10 sin(10πu + π/3), u, 0, π)

Figura 10: Figura problema 2.5.

6. Construir las figuras de Lissajous de los movimientos siguientes:

a) x = cos(2ωt), y = sin(2ωt).
 π
b) x = cos(2ωt), y = cos 2ωt − .
4
c) x = cos(2ωt), y = cos(ωt).

Solución

De nuevo se usan los paquetes Mathematica y Geogebra debido a no poder


mostrar el método gráfico acá. Se usará el valor ω = 1 rad
s .

23
a) Estas dos ecuaciones son las ecuaciones paramétricas de una circun-
ferencia. Por tanto la función es:

Mathematica: ParametricPlot[{Cos[2 t], Sin[2 t]}, {t, 0 , 2 Pi}]


Geogebra: Curve(cos(2u), sin(2u), u, 0, π)

Figura 11: Figura problema 2.6a.

b) La función en Mathematica es:

Mathematica: ParametricPlot[{Cos[2 t], Cos[2 t - Pi/4]}, {t, 0 , 2 Pi}]


Geogebra: Curve(cos(2u), sin(2u − π/4), u, 0, π)

Figura 12: Figura problema 2.6b.

c) La función en Mathematica es:

Mathematica: ParametricPlot[{Cos[2 t], Cos[t]}, {t, 0 , 2 Pi}]


Geogebra: Curve(cos(2u), sin(u), u, 0, π)

24
Figura 13: Figura problema 2.6c.

3. Vibraciones libres de los sistemas fı́sicos


1. Se cuelga de un muelle o resorte (se seguirá usando resorte en este solu-
cionario) un objeto de 1 g de masa y se le deja oscilar. Para t = 0, el
desplazamiento era 43.785 cm y la aceleración −1.7514 cm s2 . ¿Cuál es la
constante del resorte?

Solución

Como la descripción es la de la elongación de un resorte, entonces se usa


la Ley de Hooke F = −kx. Entonces,
F max (1)(−1.7514) Din
k=− =− =− = 0.04
x x 43.785 cm
Cuyo resultado no coincide con el del texto.
2. Una masa m cuelga de un resorte uniforme de constante k:
a) ¿Cuál es el perı́odo de las oscilaciones del sistema?
b) ¿Cuál serı́a el perı́odo si la masa m colgase de modo que:
1) Estuviese sujeta a dos resortes idénticos situados uno junto al
otro?
2) Estuviese sujeta al extremo inferior de dos resortes idénticos co-
nectados uno a continuación del otro? (Véase figura 14).

Solución


a) Teniendo en cuenta que la frecuencia angular se define como: ω = τ ,
k
y ω2 = m (ver texto), entonces,
r
k 2π
=
m τ

25
Figura 14: Figura problema 3.2.

y por tanto el perı́odo fundamental (en segundos) es:


r
2π m
τ0 = q = 2π
k k
m

b.1) En este caso los resortes se elongan lo mismo x1 = x2 = x, pero


las fuerzas son diferentes F1 = −kx1 y F2 = −kx2 . Por tanto la fuerza
equivalente es la suma de las dos fuerzas:

FE = −kE x = F1 + F2 = −kx1 − kx2

Con las dos elongaciones iguales se halla la constante equivalente,

−kE x = −kx − kx = −2kx ∴ kE = 2k

Y por tanto el perı́odo equivalente es:


r r
m m
τE = 2π = 2π
kE 2k

que en función del perı́odo fundamental es:


r
1 m τ0
τE = √ 2π =√
2 k 2
Resultando el perı́odo equivalente de menor magnitud que el perı́odo fun-
damental.

b.2) En este caso a los resortes se les aplica la misma fuerza F1 = F2 = F ,

26
pero tienen elongaciones diferentes x1 y x2 . Por tanto la elongación equi-
valente es la suma de las dos elongaciones:
F F1 F2
xE = − = x1 + x2 = − −
kE k k
Con las dos fuerzas iguales se halla la constante equivalente,
F F F F 1 2 k
− =− − = −2 ∴ = ⇒ kE =
kE k k k kE k 2
Y por tanto el perı́odo equivalente es:
r s r
m m 2m
τE = 2π = 2π k = 2π
kE 2
k

que en función del perı́odo fundamental es:

2m √ √
r  r 
m
τE = 2π = 2 2π = 2τ0
k k
Resultando el perı́odo equivalente de mayor magnitud que el perı́odo fun-
damental.
3. Una plataforma está realizando un movimiento armónico simple (MAS)
en dirección vertical con una amplitud de 5 cm y una frecuencia de
10 vibraciones
π s . En el punto más bajo de su trayectoria se coloca un cuerpo
sobre la plataforma.
a) ¿En qué punto se separará el cuerpo de la plataforma?
b) ¿A qué altura ascenderá el cuerpo por encima del punto más alto
alcanzado por la plataforma?

Solución

vib
a) Primero que todo la frecuencia angular está dada en unidades s y
debe convertirse a rad
s . Pero una vibración tiene 2π rad; por tanto,
 
10 vib 2π rad rad
ω= = 20
π s 1 vib s
Y como es un MAS, entonces este obedece la ecuación de la forma ma-
temática: y = A cos(ωt + α):
y = 5 cos(20t + α)
Ahora las condiciones iniciales dicen que: ”En el punto más bajo de su
trayectoria se coloca un cuerpo sobre la plataforma.”. Esto es en t = 0 s,
y = −5 cm. Por tanto:
y(0) = 5 cos(α) = −5 ⇒ cos(α) = −1 ∴ α = π.

27
Finalmente la ecuación de movimiento es:

y(t) = 5 cos(20t + π) = −5 cos(20t)

Hallando sus derivadas se obtienen la velocidad y la aceleración:

y(t) = −5 cos(20t) cm
cm
vy (t) = 100 sin(20t)
s
cm
ay (t) = 2000 cos(20t)
s2
Se observa para la aceleración que su valor máximo en magnitud es de
2000 cms2 (debido a que el valor máximo que toma coseno es unitario);
este valor es mucho mayor que el valor de la gravedad que es de 980 cm s2 .
Este hecho nos indica que como en el punto de equilibrio y = 0 cm, la
aceleración es nula ay = 0 cms2 , entonces cuando la plataforma vaya de
2000 cms2 a 0 cm
s2 , pasa por dicha magnitud, lo que implica que ahı́ se
separa el cuerpo de la plataforma, porque es como si estuviera en caı́da
libre. Reemplazando este valor en la ecuación de la aceleración, se halla el
tiempo para que la plataforma alcance el valor de la gravedad; entonces,
 
1 980
980 = 2000 cos(20t) → t = arc cos = 3.033 s
20 2000
Reemplazando este valor en la ecuación de posición:

y = −5 cos[20(3.033)] = −2.45 cm

El valor es negativo porque está por debajo de y = 0.

b) Como el cuerpo ya está separado de la plataforma, el movimiento no


es armónico simple, sino solo vertical bajo la influencia de la gravedad.
Por tanto la altura alcanzada se halla con las ecuaciones de caı́da libre.
Primero se debe halla la velocidad de despegue. Reemplazando el valor
hallado del tiempo en vy , se halla la velocidad vertical inicial v0y :
cm
v0y = 100 sin[20(3.033)] = 87.17
s
Se usa la ecuación vy2 = v0y
2
− 2gy para vy = 0 con el fin de hallar la altura
máxima H:

vy2 = v0y
2
− 2gy → vy2 = 0 = 87.172 − 2(980)H

Finalmente,
87.172
H= = 3.88 cm
2(980)
Pero este resultado es la altura desde y0 = 2.45 cm, que fue el punto donde
se despegó el cuerpo de la plataforma (se necesita referenciar el valor para

28
el problema que se analiza). Por tanto, desde la base de la plataforma la
distancia es: 2.45 + 3.88 = 6.33 cm. Si se referencia la altura desde y = 0,
entonces esta es de 6.33 − 5.00 = 1.33 cm.
4. Un cilindro de diámetro d flota manteniendo la parte l de su longitud
sumergida. La altura total es L. Admı́tase que no hay amortiguamiento.
En el instante t = 0 se empuja el cilindro hacia abajo una distancia B y
se suelta.
a) ¿Cuál es la frecuencia de la oscilación?
b) Dibujar un gráfico de la velocidad en función del tiempo desde
t = 0 hasta t = 1 perı́odo. Deberá incluirse la amplitud y fase correctas.

Solución

a) El cilindro flota, y por tanto está en equilibrio su peso y el empuje gene-


rado por el principio de Arquı́medes. El empuje es de la forma: E = ρf gV
(ρf es la densidad del fluido); donde el volumen V , es el producto entre la
sección transversal α, y la altura que se desplaza y: V = αy. Ası́ la fuerza
de restitución está dada por: −ρf gαy = F = may y la aceleración es:

ω2
z }| {
ρf gα
ay = − y
m
Por tanto la frecuencia de oscilación es:
r
2 ρf gα ρf gα
ω = → ω=
m m

Pero el cilindro tiene un volumen dado por V = αL, entonces,


r r
ρf gαL ρf gV
ω= =
mL mL
ρ
Y la relación mf a pesar que ρf es la densidad del fluido, resulta ser el
recı́proco del volumen del cilindro: V −1 . Entonces:
r r
V −1 gV g
ω= resultando ω =
L L

que es la frecuencia angular de oscilación del cilindro que flota.


ρ gα
b) Para hallar la velocidad se soluciona la ecuación: ay = − fm y. Es-
ta ecuación es una ecuación diferencial de segundo grado de la forma:

d2 y ρf gα
+ y=0
dt2 m

29
Cuya solución es (ver texto):
r 
ρf gα
y = B cos t+φ
m

Cuando se sumerge el cilindro es t = 0 y la posición es y = −B; entonces


en t = 0 s:

−B = B cos(φ) → cos(φ) = −1 ⇒ φ=π

La ecuación queda:
r  r 
ρf gα ρf gα
y = B cos t+π = −B cos t
m m

La velocidad es:
r r  r 
ρf gα ρf gα ρf gα
vy = B sin t = A sin t
m m m
| {z }
A
q
ρf gα
Donde, A = B es la amplitud de la velocidad, o la velocidad máxi-
m
q
ρf gα
ma en magnitud. El perı́odo se halla usando ω = 2π
τ = m :

m
r
τ = 2π
ρf gα

Donde resulta que la gráfica es una función sinusoidal completa en un


perı́odo.
5. Una varilla uniforme de longitud L se sujeta por un clavo a un poste de
modo que dos tercios de su longitud están por debajo del clavo. ¿Cuál es
el perı́odo de las oscilaciones pequeñas de la varilla?

Solución

En la figura (ver figura 15), se puede observar junto con la lectura del
enunciado que la varilla tiene solamente una dimensión debido a que no se
menciona que posee sección transversal (varilla con volumen, 3D), o ancho
(varilla con área, 2D).

30
Figura 15: Figura problema 3.5.

Se tratará como un péndulo simple. Las fuerzas involucradas en la varilla


se observan en la figura 16.

Figura 16: Figura problema 3.5.

Los torques que actúan sobre la varilla delgada alrededor del punto O son
(ver figura 16):

L
τm = sin(θ)mg
6
L
τM = − sin(θ)M g
3

31
Y por tanto, el torque total es:
L L L 2L
τ= sin(θ)mg − sin(θ)M g = sin(θ)mg − sin(θ)M g
6 3 6 6
L
= sin(θ)(m − 2M )g
6
Pero el torque es igual al producto entre la aceleración angular α y el
momento de inercia I: τ = Iα, y la densidad lineal de masa es µ = MLT ,
que para cada pedazo de varilla su masa es, m = Lµ 2Lµ
3 , y M = 3 ; entonces:

d2 θ
  
L Lµ 2Lµ
Iα = I 2 = sin(θ) −2 g
dt 6 3 3
  
L 1 2
= sin(θ) −2 gLµ
6 3 3

donde se verifica que:

d2 θ gL2 µ
I 2
=− sin(θ)
dt 6
El momento de inercia es fuera del centro de masa CM (ver figura 16),
entonces se usa el teorema de Steiner para calcularlo (I = ICM + md2 ):
 2
1 2 2 L
I = ICM + MT d = MT L + MT
12 6
2
1 L 3 L2
= MT L2 + MT = MT L2 + MT
12 36 36 36
1 2
= MT L
9
Reemplazando,

1 d2 θ gL2 µ
MT L2 2 =− sin(θ)
9 dt 6
1 d2 θ gµ
MT 2 =− sin(θ)
3 dt 2
1 d2 θ gµ
µL 2 =− sin(θ)
3 dt 2
d2 θ g
= − 2L  sin(θ)
dt2 3

Para linealizar la ecuación se hace la aproximación sin(θ) ∼ θ para llegar


a:
d2 θ g
= − 2L  θ
dt2 3
| {z }
ω2

32
que finalmente da la frecuencia del sistema:
s r s
g 2π 3g 2L
ω= 2L
 ó = → τ = 2π s
3
τ 2L 3g

6. Un arco circular de diámetro d se cuelga de un clavo. ¿Cuál es el perı́odo


de sus oscilaciones cuando las amplitudes son pequeñas?

Solución

La figura que describe el sistema es:

Figura 17: Figura problema 3.6.

La localización del centro de masa es CM 0, 2r



π (ver figura 18):

Figura 18: Figura problema 3.6.

Su cálculo se realiza con las ecuaciones:


R
xdm
xCM = Rλ
dm

ydm
yCM = Rλ
λ
dm

33
Se convierten estas en polares (ver figura 18), y se usa la densidad lineal
de masa µ = dm ds para escribir el diferencial de masa en función de la
longitud:
R
r cos(θ)µds
xCM = λ R
λ
µds
R
r sin(θ)µds
yCM = λ R
λ
µds

donde ds es un diferencial de arco; este tiene el valor ds = rdθ. Entonces,


R
r cos(θ)µrdθ
xCM = λ R
λ
µrdθ
R
r sin(θ)µrdθ
yCM = λ R
λ
µrdθ

Con r y µ constantes, las integrales quedan como:

r2 µ λ cos(θ)dθ
R R
r λ cos(θ)dθ
xCM = R = R
rµ λ dθ dθ
2
R R λ
r µ λ sin(θ)dθ r λ sin(θ)dθ
yCM = R = R
rµ λ dθ λ

Los lı́mites de integración son entre 0 y π por ser una semicircunferencia.


Por tanto,

r 0 cos(θ)dθ r[sin(π) − sin(0)]
xCM = Rπ = =0
0
dθ π−0

r 0 sin(θ)dθ r[cos(π) − cos(0)] 2r d
yCM = Rπ = = =
0
dθ π − 0 π π

Por tanto el centro de masa de la figura se localiza en el punto CM 0, − πd




(ver figura 19).

Figura 19: Figura problema 3.6.

34
Figura 20: Figura problema 3.6.

Al desequilibrarlo un poco para que oscile (ver figura 20), se observa que
el centro de masa queda por fuera de la vertical, lo que genera un torque
que hace oscilar la semicircunferencia. Las fuerzas que actúan se observan
en la figura. La magnitud del vector RCM es r − πd = d2 − πd , por tanto el
torque es:  
1 1
τ = −d − sin(θ)mg
2 π
pero τ = Iα,
d2 θ
 
1 1
I 2 = −mgd − sin(θ)
dt 2 π
Para oscilaciones pequeñas sin(θ) ∼ θ. Por tanto:
d2 θ d2 θ
   
1 1 mgd 1 1
I 2 = −mgd − θ → = − − θ
dt 2 π dt2 I 2 π
El momento de inercia alrededor del centro de masa de la semicircunfe-
rencia con densidad lineal uniforme µ es:
Z π2 Z π2 Z π2
ICM = r2 dm = r2 µds = r2 µrdθ
0 0 0
m
π z }| {
mr2
Z
3
2
3π 2 2πrµ
=r µ dθ = r µ = r =
0 2 4 4
Y por tanto el monento de inercia alrededor del punto O, usando el teo-
rema de Steiner es I = ICM + mRCM 2
. Donde RCM = r − πd = d2 − πd .
Entonces,
2 2 " 2 #
mr2 md2
  
d d 2 1 1 2 1 1 1
I= +m − = +md − = md + −
4 2 π 16 2 π 16 2 π

35
    
1 1 2 1 5 1 1
= md2 + − + = md2 − +
16 4 2π π 2 16 π π 2
Por tanto el momento de inercia es:
 
2 5 π−1
I = md −
16 π2

Reemplazando en la ecuación de la aceleración:

d2 θ
   
mgd 1 1 g 1 1
= − 5 − θ=− − θ
− π−1 5 π−1
 
dt2 md2 16 π2
2 π d 16 − π2
2 π
| {z }
ω2

Entonces,
s
1
− π1 1
− π1
 
2 g 2 2π g 2
ω = 5 → ω= =
− π−1 5
− π−1
 
d 16 π2
τ d 16 π2

Entonces,
s  s 
5
− π−1 d (5π 2 − 16π + 16)2π
 
d 16 π2
τ = 2π 1 1 = 2π
g 2 − π
g (π − 2)16π 2
s  s
d 5π 2 − 16π + 16 d 5π 2 − 16π + 16
  
= 2π = 2π
g 8π 2 − 16π 8g π 2 − 2π
s s
5π 2 − 16π + 16

d

2g π 2 − 2π
| {z }
2.05

Y por tanto el perı́odo es:


s r
d r
τ = 2.05π = 2.05π
2g g

Que no coincide con el resultado del texto.


7. Un alambre de longitud l0 se alarga en 10−3 l0 , cuando se cuelga de su
extremo inferior una cierta masa. Si se conecta este mismo alambre entre
dos puntos A y B, alejados l0 y situados en el mismo plano horizontal y
de su punto medio se cuelga la misma masa, como se ve en la figura, ¿cuál
es la depresión y en dicho punto y cuál es la tensión del alambre?

Solución

El alambre inicialmente se configura como se observa en la figura 22.

36
Figura 21: Figura problema 3.7.

Figura 22: Figura problema 3.7.

Se observa que todo el alambre se elonga una milésima de su longitud


inicial. Por tanto si se observa la figura con la configuración final (ver
figura 21), la elongación a lado y lado del punto medio es: 21 10−3 l0 . De esta
forma cuando se considera la figura 21, existen dos fuerzas de restitución
F = −mω 2 y (una para cada alambre a lado y lado del vértice) que se
oponen al peso mg (ver figura 23). Se observa acá que la ecuación de
dinámica es:
 
2
y  = − q 2mω y
Fy = −2F sin(θ) = −2mω 2  q 2 2
y 2 + l20 y 2 + l20

37
Figura 23: Figura problema 3.7.

Para hallar y, se usa la geometrı́a de la figura:


s  2
l0 1
1 + 10−3 l0

y2 + =
2 2

Elevando al cuadrado:
 2  2
2 l0 2
 
l0 1 2 l0
y2 + = 1 + 10−3 l02 → y2 + = 1 + 10−3
2 4 2 2
h 2 i  l 2  2
l0
0
y2 = 1 + 10−3 = 2 × 10−3 + 10−6

−1
2 2
 
p l0 l0
y= 2 × 10−3 + 10−6 =
| {z } 2 40
1
0.045∼ 20

Como se trabajó con media figura, entonces al usar la figura completa y


por ser simétrica, se duplica el resultado anterior resultando:
l0
y=
20
Reemplazando este valor en la ecuación de dinámica,
l0 1
 
2mω 2 20 2mω 2 20 2mω 2
Fy = − q = − q = − q
l0 2
2 1 2
2 2
+ l20 + 12 1 + 20
 
20 20 2
2mω 2 1
= −√ = −0.199mω 2 ∼ −0.2mω 2 = (−mω 2 )
1 + 10 2 5

Entonces
5Fy = −mω 2 = T
Pero Fy = mg. Por tanto:
T = 5mg

38
8. a) Un objeto de 0.5 Kg de masa se cuelga del extremo de un alambre de
acero de 2 m de longitud y 0.5 mm de diámetro (módulo de Young =
N
2 × 1011 m 2 ). ¿Cuál es el alargamiento del alambre?

b) El objeto se levanta en una distancia h (de modo que el alambre deja


de estar tirante) y luego se deja caer de modo que el alambre recibe un
N
tirón súbito. La carga de rotura del acero es de 1.1 × 109 m 2 . ¿Cuál es el

valor posible de h, que resiste el alambre sin romperse?

Solución

a) Teniendo en cuenta que (ver el texto):


F L0 mgL mgL
Y = → x= =  2
σ∆L Y πr2
Y π φ2

donde σ es la sección transversal que es circular porque se supone que el


alambre es cilı́ndrico con diámetro φ. Reemplazando valores:
4mgL 4(0.5)(9.8)2 8(9.8)
x= = =
Y πφ2 (2 × 1011 )π(0.5 × 10−3 )2 π × 105
= 0.00025 m = 0.25 mm

b) Para el tirón súbito y analizar el efecto de la carga sobre el alambre,


sigue usándose la caracterı́stica de rigidez de este. Hallando el trabajo
realizado se tiene que:
Z s
Y πr2 (2 × 1011 )π(0.5 × 10−3 )2 s
Z
W = xdx = xdx
0 L 2 0

Donde s es la elongación máxima. Para calcularla, se halla la fuerza máxi-


ma soportada por el alambre. Esta es (usando la deformación normal so-
L0
lamente; esto es cuando ∆L → 1):

FM ax YR πφ2 (1.1 × 109 )π(0.5 × 10−3 )2


= YR → FM ax = =
πr2 4 4
Lo que da una fuerza máxima de FM ax = 215.98 N . Este valor se substi-
tuye en la ecuación de rigidez resultando la elongación máxima s:
4FM ax L 4(215.98)2 32(215.98)
s= 2
= 11 −3 2
= = 0.011 m
Y πφ (2 × 10 )π(0.5 × 10 ) (2 × 105 )π
Reemplazando en la ecuación hallada anteriormente, se calcula el trabajo:

(2 × 1011 )π(0.5 × 10−3 )2 0.011


Z
W = xdx = 1.19 J
2 0

39
Pero ese trabajo es igual a la energı́a potencial. Por tanto,
1.19 1.19
1.19 = mgh → h= = = 0.24 m
mg 0.5(9.8)
Finalmente la distancia es: h = 0.24 m.
9. a) Se cuelga una bola de acero maciza del extremo de un alambre de acero
de 2 m de longitud y radio 1 mm. La carga de rotura del acero es de
N
1.1 × 109 m 2 . ¿Cuáles son el radio y la masa de la bola de mayor tamaño

que pueda soportar el alambre?


b) ¿Cuál es el perı́odo de oscilaciones de torsión de este sistema? (Módulo
N
de cizalladura del acero = 8 × 1010 m 2 . Momento de inercia de la esfera
2
respecto a un eje que pasa por el centro = 2M5R .)

Solución

a) Usando,
(1.1 × 109 )π(1 × 10−3 )2
FM ax = mg = YR πr2 → m= = 352.63 Kg
9.8
Kg
La densidad media del acero es de 7850 m3 ; por tanto,
352.63
m = 352.63 = ρV → V = πr3 =
7850
Ası́, el radio es:
  31
352.63
r= = 0.24 m = 24 cm
7850π

b) La masa de la varilla es m = ρV = 7850 π(1 × 10−3 )2 (2) = 0.05 Kg


 

y su peso es WV = 0.05(9.8) = 0.48 N . El peso de la esfera es WE =


352.63(9.8) = 3455.77 N . Con las masas se calcula el centro de masa del
sistema a partir de la varilla:
mV yV = mE yE
y = yV + yE = 1.12 m
0.05yV = 352.63(1.12 − yV ) = 394.95 − 352.63yV
394.95
352.68yV = 394.95 → yV = = 1.1199 ∼ 1.12 m
352.68
que es la posición del centro de masa de la esfera. Por tanto desde el punto
de vista de centros de masa, el sistema se comporta como si la varilla no
existiera; esto es, como si estuviera oscilando en torsión alrededor de su
centro. Usando la ecuación (ver texto):
r
I
τ = 2π
c

40
Donde I es el momento de inercia de la esfera de acero alrededor de un eje
que pasa por el centro geométrico de esta y c es el módulo de cizalladura
del acero. Reemplazando los valores dados:
s s
2M R2
5 2(352.68)(0.24)2
τ = 2π = 2π = 0.000063 s = 63 µs
8 × 1010 5(8 × 1010 )

10. Una varilla metálica de 0.5 m de larga tiene una sección recta rectangular
de 2 mm de área.
a) Puesta vertical la varilla y teniendo colgada una masa de 60 Kg en
su extremo inferior, seproduce un alargamiento de 0.25 mm. ¿Cuál es el
N
módulo de Young m 2 de material de la varilla?
b) Se sujeta firmemente la varilla por su parte inferior, como se indica en
el esquema, y en su parte superior se aplica una fuerza F en dirección y,
como está indicado (paralela a la arista de longitud b). El resultado es una
flexión elástica dada por:
4L3
y= F
Y ab3

Figura 24: Figura problema 3.10.

41
Si se suprime la fuerza F y se sujeta a la parte superior de la varilla una
masa m, mucho mayor que la masa de la varilla, ¿Cuál es el cociente de
las masas de las frecuencias de vibración en las direcciones y y x (es decir
paralelas a las aristas de longitud b y a)?
c) Se empuja la masa lateralmente en una determinada dirección trans-
versal y luego esta se suelta, con lo que se describe una trayectoria como
la de la figura 25. ¿Cuál es el cociente entre a y b?

Figura 25: Figura problema 3.10.

Solución

a) Con L0 = 0.5 m, σ = 2 × 10−6 m2 , F = mg = 60(9.8) y ∆L =


0.25 × 10−3 m, se reemplaza en la ecuación:
F L0
Y =
σ∆L
Resultando,
60(9.8)(0.5) N
Y = = 5.88 × 1011 2
2 × 10−6 (0.25 × 10−3 ) m

4L3
b) La flexión elástica y = Y ab3 F
debido a la fuerza, se reemplaza por:

4L3 d2 y
 
y=− m 2
Y ab3 dt
El signo negativo es debido a que la masa genera una fuerza de restitución.
Por tanto reorganizando se llega a:
d2 y Y ab3
= − y
dt2 |4mL
3
{z }
ωy2

42
donde r
Y ab3
ωy =
4mL3
La flexión en x es:
4L3 d2 x
 
x=− 3 m 2
Ya b dt
Reorganizando,
d2 x Y a3 b
2
=− 3
x
dt |4mL
{z }
2
ωx

donde r
Y a3 b
ωx =
4mL3
El cociente de las vibraciones es:
√ √
q
Y ab3
ωy 4mL3 ab3 b2 b
=q =√ =√ =
ωx 3
Ya b 3
a b a 2 a
4mL 3

c) Se observa que el movimiento es una superposición perpendicular de


osciladores con diferentes frecuencias. Esto es la gráfica que genera este
sistema es una figura de Lissajous. Si se observa el texto en el capı́tulo
anterior la relación de frecuencias da 1.5

11. a) Hallar la frecuencia de vibración en condiciones adiabáticas de una co-


lumna de gas encerrada en un tubo cilı́ndrico, cerrado por un extremo y
con un pistón de masa m bien ajustado pero que puede moverse libremen-
te.
b) Una bola de acero de 2 cm de diámetro oscila verticalmente en un tubo
de vidrio con un orificio de precisión montado sobre un frasco de vidrio
de 12 litros que contiene aire a la presión atmosférica. Comprobar que el
perı́odo de oscilación deberá ser de 1 segundo aproximadamente. (Admitir
que las variaciones de presión son adiabáticas, con γ = 1.4. Densidad del
acero 7600 Kg
m3 .)

Solución

a) En condiciones adiabáticas se verifica que (ver texto), pV γ = constante.


Al aplicar logaritmo natural a miembro y miembro y luego derivar, se ob-
tiene (donde V es el volumen):

1 dp γ
+ =0
p dV V

43
F
Pero p = σ donde σ es la sección transversal. Reemplazando en la presión,
 
σ 1 dF γ 1 dF γ
+ =0 → + =0
F σ dV V F dV V
En el volumen V = σl la sección transversal es constante. Por tanto,
1 dF γ 1 dF γ
+ =0 → + =0
σF dl σl F dl l
Entonces,
dF γ
+ F =0
dl l
La fuerza varı́a respecto a la posición, pero la masa del pistón es constante.
Entonces con la segunda Ley de Newton para masa constante F = ma:
da γ da γ
m + F =0 → + F =0
dl l dl ml
Con dl = vdt y F = pσ,
1 da γ da γpσ
+ pσ = 0 → + v=0
v dt ml dt ml
Finalmente en función de la velocidad, la ecuación diferencial queda como:
d2 v γpσ
+ v=0
dt2 ml
Analizando dimensionalmente la ecuación se observa que,
γpσ
ω2 =
ml
donde la frecuencia pedida tiene un valor de,
r
γpσ
ω=
ml

b) El perı́odo es:
s s
4

ρ 3
ml 3 πr l
τ = 2π = 2π
γpσ γpσ

La esfera tiene una peso de:


   
4 3 4 −2 3
mg = ρV = 7600 πr g = 7600 π(1 × 10 ) 9.8 = 0.312 N
3 3
La presión ejercida por la esfera es de:
W 0.312 N
pE = = = 993.127 2
σE π(1 × 10−2 )2 m

44
Reemplazando los valores, el área de la esfera 4πr2 = 4π(1 × 10−2 )2 m2 ,
N
presión atmosférica patm = 1.01 × 105 m 2 y la densidad del acero ρ =
Kg
7600 m3 ,
s
7600 34 π(1 × 10−2 )3 l √
 
τ = 2π 5 −2 2
= 0.084 l
1.4(1.01 × 10 + 993.127)2π(1 × 10 )
Por último, la longitud l equivale al volumen sobre el área, donde el volu-
men es la diferencia entre el volumen del cilindro y el volumen de la esfera
(ver corte del pistón en la figura 26):

Figura 26: Figura problema 3.11.

V 12 − 43 πr3 12 − 43 π(1 × 10−2 )3


l= = = = 9549.29 m
σ 4πr2 4π(1 × 10−2 )2
entonces: √
τ = 0.084 9549.29 = 16.33 s????
12. El movimiento de un oscilador lineal puede representarse mediante un
gráfico en el que se muestra x en las abscisas y dx
dt en las ordenadas. La
historia del oscilador resulta ser entonces una curva.
a) Demostrar que esa curva es una elipse en el caso un oscilador no
amortiguado.
b) Demostrar (cualitativamente al menos) que si se incluye un término
de amortiguamiento se obtiene una curva en espiral hacia el origen.

Solución

a) Para un oscilador no amortiguado su ecuación y solución son:


d2 x
+ ω02 x = 0 → x(t) = A cos(ω0 t + α)
dt2

45
La velocidad alcanzada por el objeto que oscila es:
dx(t)
= −ω0 A sin(ω0 t + α)
dt
Como se observa la velocidad depende de una función trigonométrica con
una diferencia de fase de π2 respecto a la función trigonométrica de la cual
depende la posición. Esto hace que los dos vectores sean perpendiculares.
La función coseno es postiva en los intervalos 0 ≤ ω0 t + α ≤ π2 , 3π 2 ≤
ω0 t + α ≤ 2π y negativa en el intervalo π2 < ω0 t + α < 3π
2 . La función seno
es positiva en el intervalo 0 ≤ ω0 t + α ≤ π2 y negativa en el intervalo π2 <
ω0 t+α < 2π. Se analiza que debido a que la velocidad va en las ordenadas,
la mitad de la gráfica es negativa y la otra mitad positiva. Pero debido
a que las abscisas comienzan con su valor máximo (cuando las ordenadas
valen cero) y terminan con su valor máximo (cuando las ordenadas valen
cero), la gráfica es cerrada. Finalmente se puede ver claramente que con
los cambios de signo de las abscisas, se genera una elipse.
Analı́ticamente se observa que las dos funciones son:

x(t) = A cos(ω0 t + α)
dx(t)
y(t) = = −ω0 A sin(ω0 t + α)
dt
Entonces,
x(t)
= cos(ω0 t + α)
A
y(t)
= sin(ω0 t + α)
−ω0 A
Al elevarlas al cuadrado,
 2
x(t)
= cos2 (ω0 t + α)
A
 2
y(t)
= sin2 (ω0 t + α)
−ω0 A
y al sumarlas,
 2  2
x(t) y(t)
+ = cos2 (ω0 t + α) + sin2 (ω0 t + α) = 1
A −ω0 A
Finalmente con B = −ω0 A se llega a:

x2 (t) y 2 (t)
+ =1
A2 B2
Que es la ecuación de una elipse, lo cual demuestra lo pedido. x(t) =
A cos(ω0 t + α) y y(t) = −ω0 A sin(ω0 t + α) son las ecuaciones paramétricas

46
de la elipse.

b) Para un oscilador amortiguado su ecuación y solución son:

d2 x dx γ

2
+γ + ω02 x = 0 → x(t) = Ae− 2 t cos(ω 0 t + α)
dt dt
q 2
donde ω 0 = ω02 − γ2 . Su velocidad es:

dx(t) γ γ γ
y(t) = = − Ae− 2 t cos(ω 0 t + α) − ω 0 Ae− 2 t sin(ω 0 t + α)
dt 2
Cuando se introduce el término de amortiguamento (que es una expo-
nencial con potencia negativa temporalmente dependiente) se obliga al
sistema a que su amplitud tienda a cero. Se observa que las ecuaciones pa-
ramétricas de la elipse x(t) = A cos(ω0 t + α) y y(t) = −ω0 A sin(ω0 t + α),
al ser multiplicadas por un término de amortiguamiento quedan como:
γ γ
x(t) = Ae− 2 t cos(ω0 t + α) y y(t) = −ω0 Ae− 2 t sin(ω0 t + α)

que son idénticos matemáticamente a los términos:


γ
x(t) = Ae− 2 t cos(ω 0 t + α)

y al segundo término de la velocidad del sistema amortiguado:


γ
−ω 0 Ae− 2 t sin(ω 0 t + α)

El primer término de la velocidad del sistema amortiguado:


γ γ
− Ae− 2 t cos(ω 0 t + α)
2
es proporcional a x(t). Entonces teniendo en cuenta que las ecuaciones
paramétricas de una espiral circular respecto al origen son:

x(u) = eu cos(u) y y(u) = eu sin(u),

el sistema de ecuaciones que se presenta en un oscilador amortiguado es


un sistema espiral elı́ptico respecto al origen. Para ver la gráfica de estas
ecuaciones, se usa el programa Mathematica con la instrucción:
 s  s 
 2 2  2  2  2 2
 π2 π π − π
t π
ParametricPlot  e− 20 t cos  π 2 − t , − te 20 cos  π 2 − t
 20 20 20
s s  
 2 2  2   2 2 
π − π
t π
− π2 − e 20 sin  π 2 − t , {t, 0, 2π}
20 20 

Se observa la figura 27.

47
Figura 27: Figura problema 3.12.

13. Comprobar que x(t) = Aeαt cos(ωt) es una posible solución de la ecuación
d2 x dx
+γ + ω02 x = 0
dt2 dt
y hallar α y ω en función de γ y ω0 .

Solución

Derivando temporalmente la solución propuesta y multiplicándola por γ


se tiene que:
dx
γ = γAαeαt cos(ωt) − γAωeαt sin(ωt)
dt
Derivando temporalmente la anterior derivada se llega a:
d2 x
= Aα2 eαt cos(ωt) − Aαωeαt sin(ωt)
dt2
− Aωαeαt sin(ωt) − Aω 2 eαt cos(ωt)
= (α2 − ω 2 )Aeαt cos(ωt) − 2ωαAeαt sin(ωt)
Sumando los dos resultados junto con ω02 x(t):
d2 x dx
2
+γ + ω02 x = (α2 − ω 2 )Aeαt cos(ωt) − 2ωAαeαt sin(ωt)
dt dt
+ γαAeαt cos(ωt) − γωAeαt sin(ωt) + ω02 Aeαt cos(ωt) = 0

48
Reagrupando términos,

d2 x dx
2
+γ + ω02 x = (α2 − ω 2 + ω02 + γα)Aeαt cos(ωt)
dt dt
− (2ωα + γω)Aeαt sin(ωt) = 0

Como esta ecuación diferencial de segundo orden vale cero, entonces cada
término se anula por separado. Debido a que la amplitud A no vale cero,
tampoco los valores de α y t tienden a −∞ y los valores de las funciones
trigonométricas no son nulos todo el tiempo, entonces los valores entre los
paréntesis son nulos; esto es:

(α2 − ω 2 + ω02 + γα) = 0


(2ωα + γω) = 0 → ω(2α + γ ) = 0
| {z }
0

Resolviendo la segunda ecuación,


γ
2α + γ = 0 ∴ α=−
2
Reemplazando este valor en la primera ecuación,
 γ 2  γ
− − ω 2 + ω02 + γ − =0
2 2
entonces,
 γ 2  γ  γ2 γ2
ω2 = − + ω02 + γ − = + ω02 −
2 2 4 2
2 2
γ − 2γ  γ  2
= + ω02 = − + ω02
4 2
Por tanto, r  γ 2
ω= ω02 −
2
Resumiendo: r
γ  γ 2
α=− y ω = ω02 − .
2 2
y por tanto la solución del oscilador amortiguado es:
"r #
 γ 2
− γ2 t 2
x(t) = Ae cos ω0 − t
2

14. Se cuelga un objeto de masa 0.2 Kg de un resorte cuya constantes es


N
80 m . Se somete el objeto a una fuerza resistente dada por −bv, siendo v
su velocidad en m
s .

49
a) Plantear la ecuación diferencial de movimiento en el caso de osci-
laciones libres del sistema.

b) Si la frecuencia de amortiguamiento es 23 de la frecuencia sin
amortiguamiento, ¿cuál es la frecuencia de la constante b?
c) ¿Cuál es el valor Q del sistema, y en qué factor se reducirá la am-
plitud del sistema después de 10 ciclos completos?

Solución
q q
k 80 rad
a) La frecuencia fundamental del sistema es ω0 = m = 0.2 = 20 s ;
b b rad
la frecuencia de amortiguamiento es de γ = m = 0.2 = 5b s . La ecuación
de dinámica del sistema es:

FN = FR + FA

donde FN = Fuerza Neta, FR = Fuerza de restitución y FA = Fuerza de


amortiguamiento. Entonces,
d2 x dx d2 x k b dx
m 2
= −kx − b → 2
=− x−
dt dt dt m
|{z} m dt
|{z}
ω02 γ

Reemplazando los valores hallados y despejando,


d2 x dx
+ 5b + 400x = 0
dt2 dt
Usando el resultado del problema anterior, se llega a la solución:
r !
− 5b t 25b2
x(t) = Ae 2 cos 400 − t
4

b) Siguiendo el enunciado, en el sistema amortiguado hay tres frecuen-


cias: la frecuencia fundamental ω0 , la frecuencia de amortiguamiento γ y
la frecuencia del sistema ω 0 . La que se requiere en este problema es la del
sistema. por tanto,
√ r √
0 3 25b2 3
ω = ω0 → 400 − = (20)
2 4 2
elevando al cuadrado,
25b2 3 25b2
400 − = (400) → 400 − 300 =
4 4 4
r
2 400 N ·s
400 = 25b → b= =4
25 m

50
Con esto la solución queda
 √ 
x(t) = Ae−10t cos 10 3t

c) Como el factor de calidad es Q = ωγ0 , entonces reemplazando los valores


20
hallados, este toma la magnitud de: Q = 20 = 4. Para hallar el factor de
reducción de la amplitud se trabaja solo con la parte de la solución que
tiene esta. Por tanto, con n como el factor de reducción,
nA = Ae−10t
el valor del perı́odo después de 10 ciclos es
 

τ10 = 10 =π
20
Entonces reemplazando, el factor toma la magnitud de:
n = e−10π

15. Muchos sistemas oscilantes, aunque el mecanismo de pérdida o disipación


no sea análogo al amortiguamiento viscoso, muestran una disminución
exponencial con el tiempo de su energı́a almacenada media, E = E 0 e−γt .
Puede definirse una Q para ellos utilizando la definición Q = ωγ0 , siendo
ω0 la frecuencia angular natural.
a) Cuando se pulsa en el piano la nota ”do”, su energı́a de oscilación
disminuye a la mitad de su valor inicial en 1 segundo aproximadamente.
La frecuencia de dicha nota es 256 Hz. ¿Cuál es la Q del sistema?
b) Si la nota correspondiente a una octava más alta (512 Hz) emplea
aproximadamente el mismo tiempo para perder su energı́a, ¿cuál es su Q?
c) Un oscilador armónico libre y amortiguado, compuesto por una
masa m = 0.1 Kg, que se mueve en un lı́quido viscoso de coeficiente
de amortiguamiento b(Fviscosa = −bv), sujeta a un resorte de constante
N
k = 0.9 m , realiza un movimiento oscilante amortiguado. Su energı́a me-
dia disminuye a 1e de su valor inicial en 4 s. ¿Cuál es la Q del oscilador?
¿Cuál es el valor de b?

Solución

a) Como la energı́a disminuye a la mitad, entonces usando la ecuación


E = E 0 e−γt ,
1 1
E 0 = E 0 e−γ(1) → = e−γ
2 2
Calculando el recı́proco de la ecuación,
2 = eγ ∴ γ = ln(2)

51
La frecuencia fundamental es,

ω0 = 2πν = 2π(256) = 512π

Por tanto la Q0 es,


ω0 512π
Q0 = =
γ ln(2)

b) Se duplica la frecuencia y entonces, ω = 2ω0 y entonces:


ω 2ω0 ω0
Q= = =2 = 2Q0
γ γ γ

c) Primero se calcula la frecuencia fundamental,


r r
k 0.9 rad
ω0 = = =3
m 0.1 s
1
Como su energı́a media disminuye a e de su valor inicial en 4 s, entonces:

1 1
E 0 = E 0 e−γ(4) → = e−4γ
e e
Calculando el recı́proco de la ecuación,
1
e = e4γ ∴ γ=
4
La Q es,
ω0 3
Q= = 1 = 12
γ 4
Y el coeficiente de amortiguamiento es:
 
1 1 N ·s
b = mγ = 0.1 = = 0.025
4 40 m

16. De acuerdo con la teorı́a electromagnética clásica, un electrón acelerado


2 2 2
radia energı́a en la proporción Kec3a , siendo K = 6 × 109 NC·m
2 , e= carga

del electrón (C), a= aceleración instantánea sm2 y c= velocidad de la luz



m
s .
a) Si un electrón estuviese oscilando a lo largo de una recta con
frecuencia ν Hz y amplitud A, ¿Cuánta energı́a emitirá durante un ci-
clo? (Suponer que el movimiento queda adecuadamente descrito por x =
A sin(2πνt) durante un ciclo cualquiera.)
b) ¿Cuál es la Q de este oscilador?

52
c) ¿Cuántos perı́odos de oscilación deberı́an ocurrir antes que la energı́a
del movimiento disminuyese a la mitad de su valor inicial?
d) Sustituyendo ν por una frecuencia óptica tı́pica (es decir, luz visi-
ble), estimar numéricamente a Q aproximada y la ”vida mitad” del sistema
radiante.

Solución

a) Para hallar la energı́a emitirá durante un ciclo se usa la ecuación


2 2
E = Kec3a , donde se reemplaza la aceleración
d2 x
= −A(2πν)2 sin(2πνt)
dt2
quedando:
 2
Ke2 −A(2πν)2 sin(2πνt) Ke2 A2 (16π 4 ν 4 ) sin2 (2πνt)
E= =
c3 c3
Para hallar la energı́a emitida en un ciclo, se halla el valor medio de la
energı́a en un perı́odo, multiplicada por el perı́odo porque se debe evaluar
el valor medio en un ciclo. Esto da:
 Z τ 
2 2 4 4 2
1 Ke A (16π ν ) sin (2πνt) 
hEi = τ  dt
τ c3
0

16π 4 ν 4 Ke2 A2 τ
Z
= sin2 (2πνt)dt
c3 0
1−cos(4πνt)
Pero: sin2 (2πνt) = 2 ; entonces,
τ
Z
16π 4 ν 4 Ke2 A2 1 − cos(4πνt)
hEi = dt
c3 2
0 
τ τ
16π 4 ν 4 Ke2 A2 
Z Z

= 3
 dt − cos(4πνt)dt
2c  
| 0{z } | 0 {z }
=τ =0
=1
8π 4 ν 4 A2 Ke2 τ 8π 4 ν 3 ντ A2 Ke2 8π 4 ν 3 A2 Ke2
z}|{
= = =
c3 c3 c3
Haciendo el análisis dimensional del resultado,
 1 3 h 2
i
8π 4 ν 3 A2 Ke2 s [m]2 NC·m 2 [C]2
hEi = →
c3 m 3
 
s
h 2 i
m ·N ·m2 ·C 2  4
m · N · C 2 · s3

s3 ·C 2
=  m3  = = [N · m] = [J]
s3
s3 · C 2 · m3

53
que coincide con una energı́a.
ω0
b) Por definición, Q = γ . La energı́a sobre unidad de masa es,
2
Ke2 A2 (2πν)4 sin2 (2πνt)

E dx
= = vx2 = = A2 (2πν)2 cos2 (2πνt)
m mc3 dt
Simplificando,
Ke2 ω02 sin2 (2πνt) Ke2 ω0 ω0 sin2 (2πνt)
= cos2 (2πνt) → = cos2 (2πνt)
mc3 mc3
mc3
Entonces debido a que Ke2 2πν es adimensional como Q, se despeja esta
cantidad resultando:
ω0 sin2 (2πνt) ω0 mc3
= 2 = Q =
cos2 (2πνt) Ke2 2πν
 
cos(2πνt)
sin(2πνt)
| {z }
γ

Por tanto,
mc3
Q=
2πνKe2
Como es el movimiento de un electrón en el eje x con una energı́a determi-
nada, entonces al elegir un sentido de los dos (+ ó -), se divide el resultado
entre dos, quedando:
mc3
Q=
4πνKe2

c) La energı́a para un sistema amortiguado está dada por (ver texto):


ω0
E = E0 e− 2Q t . Por tanto para que la energı́a disminuya a la mitad se
requiere que E = 12 E0 ; la ecuación queda:
1 ω0 1 ω0
E0 = E0 e− 2Q τ → = e− 2Q τ
2 2
Calculando el recı́proco de la ecuación,
ω0 ω0 2Q
2 = e 2Q τ → ln(2) = τ → ln(2) = τ
2Q ω0
Reorganizando se llega a:
2Q ln(2)
τ=
ω0
Si se hace un análisis dimensional, se llega a:
Adimensional
z }| {
2Q ln(2) 2Q ln(2)
τ= → = [s]
ω0 [s−1 ]

54
que corresponden con las unidades de un perı́odo.
3
mc
d) La Q del sistema es: 4πνKe2 . Reemplazando los valores K = 6 ×
2
9 N ·m −31
10 C 2 , m = 9.1094 × 10 Kg, e = −1.602176 × 10−19 C, c =
m
299792458 s ,

mc3 (9.1094 × 10−31 )(299792458)3


Q= =
4πνKe2 4πν(6 × 109 )(−1.602176 × 10−19 )2
Un valor óptico tı́pico para la luz visible está en el color verde, cuyo valor
es alrededor de, 530 × 10−9 m. Reemplazando este valor se llega a:
(9.1094 × 10−31 )(299792458)3
Q= w 2.4 × 1028
4π(530 × 10−9 )(6 × 109 )(−1.602176 × 10−19 )2
El tiempo de semidesintegración del sistema, que es un sistema de elec-
trones, es:
τ Q ln(2) (2.4 × 1028 ) ln(2)
τ 21 = = = = 4.99 × 1033 s
2 2πν 2π(530 × 10−9 )
Que corresponde a un tiempo de desintegración adecuado para un electrón
calculado en la literatura (6.6 × 1028 años). Esto es,
 
33 1 año
τ 12 = 5 × 10 s = 5.70 × 1028 años
86400 s

17. Un tubo en U tiene brazos verticales de radio r y 2r, unidos por un tubo
horizontal de longitud l cuyo radio aumenta linealmente de r a 2r. El tubo
en U contiene lı́quido hasta una altura h en cada brazo. Se pone a oscilar
el lı́quido y en un momento dado el lı́quido en el brazo más estrecho está
a una altura y sobre el nivel de equilibrio.
a) Demostrar que la energı́a potencial del lı́quido viene dada por U =
5 2 2
8 gρπr y .
b) Demostrar que la energı́a cinética de una pequeña porción cilı́ndri-
ca de lı́quido en el brazo horizontal (véase diagrama 28) viene dada por
 2
1 πr2 dx dy
dK = ρ
2 1 + x 2 dt

l

[Obsérvese que si el lı́quido no se acumula en ninguna parte, el producto


(velocidad × sección recta) deberá tener el mismo valor en todo punto del
tubo.]
c) Utilizando el resultado de la parte b), demostrar que la energı́a
cinética total de todo el lı́quido móvil viene dada por
   2
1 2 5 dy
K = ρπr l + h
4 2 dt

55
Figura 28: Figura problema 3.17.

(Despreciar cualquier efecto debido a los codos.)


5h
d) A partir de a) y b), calcular el perı́odo de oscilación si l = 2 .

Solución

a) Para solucionarlo se analiza la figura 29, donde se observa que los dos
extremos del tubo tienen radios diferentes y por tanto masas de lı́quido
diferentes.

Figura 29: Figura problema 3.17a.

Esto es, usando la Ley de Pascal, se analiza que las presiones en los extre-
mos del tubo en forma de U son iguales y por tanto:
Fr mr g F2r m2r g m2r g
Pr = = y P2r = = =
Ar πr2 A2r π(2r)2 4πr2

donde se reemplazó la fuerza por el peso del lı́quido, y el área de la sección

56
transversal del cilı́ndro. Debido a que las dos presiones son iguales:
mr g m2r g 1
Pr = P2r → = → mr = m2r
πr2 4πr2 4
Entonces la energı́a potencial es:

U = (mr + m2r )gy

Usando el resultado anterior (ver figura 29) para hallar la energı́a potencial
en función del tramo izquierdo (con radio r) debido a que es la que tiene
la columna más alta,
y y 5
ρπr2 yr gy

U = (mr + 4mr )g = 5mr g =
2 2 2
El producto entre la presión y el volumen es constante. Entonces:

Pr Vr = P2r V2r
mr g 2 m2r g 4mr g
πr yr = π(2r)2 y2r = 4πr2 y2r
πr2 4πr2 4πr2
yr = 4y2r
Por tanto se observa que la distancia que sube la columna en el tubo de
radio r es cuatro veces mayor a la distancia que baja la columna en el
tubo de radio 2r. Esto hace que la energı́a potencial sea en relación a la
distancia que empuja la columna para que suba la otra. Esto es:
1
yr = y
4
Reemplazando,
  
5 1 5
U= ρπr2 y gy = gρπr2 y 2
2 4 8

que es lo que se querı́a demostrar.

b) La energı́a cinética diferencial, por definición es:


 2
1 dy
dK = dm
2 dt

Usando la densidad volumétrica de masa:


 2
1 dy
dK = ρdV
2 dt

El diferencial de volumen puede ser calculado en la figura 30.

57
Figura 30: Figura problema 3.17b.

En esta figura se han aproximado los triángulos a las partes medias de los
tubos verticales y se obvian los efectos de borde. Se observan dos triángulos
congruentes. Sus tangentes son:
2r − R r
y
l−x l
Y son iguales (ver la figura 30); entonces,

2r − R r r
= → 2r − R = (l − x)
l−x l l
Y por tanto,
r rx rx
R = 2r − (l − x) = 2r − r + → R=r+
l l l
Finalmente el valor de R es:
 x
R=r 1+
l
Se observa claramente que cuando x = 0, entonces R = r y cuando x = l
entonces R = 2r. Tal cual como se ve en la figura.
El área de la sección es:
 x 2
πR2 = πr2 1 +
l
Y entonces esta sección es para una figura estática; pero es una sección de
un lı́quido que se mueve oscilando y por tanto tiene velocidad; por tanto
el factor que hace que varı́ e la sección transversal se define como una
variación entre los radios. Esto es,
dx  x 2
= 1+
dl l
Despejando dl que es lo que nos interesa para observar como varı́a esta
longitud con el radio,
dx
dl = 2
1 + xl

58
Entonces el elemento diferencial de volumen es:
πr2 dx
dV = πr2 dl = 2
1 + xl
Reemplazando este en la ecuación de la energı́a cinética diferencial,
 2
1 πr2 dx dy
dK = ρ
2 1 + x 2 dt

l

que demuestra lo pedido.

c) La energı́a cinética total es (integrada sobre x):


Z l"  2 #  2
Z l
1 πr2 dx dy 1 2 dy dx
Kx = ρ = ρπr
2 1 + x 2 dt 2 dt x 2
 
0 l 0 1+ l

Con u = 1 + xl , entonces du = dx
l , y ası́:
 2 Z 2  2 2  2
1 2 dy ldu l 2 dy 1 l 2 dy
Kx = ρπr 2
= − ρπr = ρπr
2 dt 1 u 2 dt u 1 4 dt
Pero la velocidad varı́a y como dice el enunciado: [Obsérvese que si el
lı́quido no se acumula en ninguna parte, el producto (velocidad × sección
recta) deberátener el mismo valor en todo punto del tubo.]. Entonces el
factor 1 + xl tiene que  tenerse en cuenta para este resultado, pero en el
y
eje y. Ası́, K = 1 + l Kx :
   2  2
l 2 y(h) dy 1 2 dy
K = ρπr 1 + = ρπr [l + y(h)]
4 l dt 4 dt
En la figura 28 se observa que cuando el lı́quido llega a la parte superior
su velocidad es nula y cuando llega a la parte inferior también. Entonces
este factor es el que modula la velocidad en y y debe depender solo de h,
y(h).
Observando el resultado de  la parte a) se analiza la ecuación de la energı́a
potencial: U = 25 ρπr2 yr gy, se encuentra que hay un factor de modu-
lación de la altura alcanzada que es 52 . Este factor es justamente el que
modula h y por tanto y(h) = 25 h, y la energı́a cinética queda:
   2
1 2 5 dy
K = ρπr l + h
4 2 dt
que era lo que se querı́a demostrar.

d) Usando los resultados de a) y b) (c)), la energı́a mecánica total es:


   2
1 2 5 dy 5
E = ρπr l + h + gρπr2 y 2
4 2 dt 8

59
Para l = 52 h,
 2
5 dy 5
E= ρπr2 h + gρπr2 y 2
4 dt 8
Multiplicando miembro a miembro por:
4
5ρπr2 h
la ecuación queda como:
 2  
4E dy 4 5
= + gρπr2 y 2
5ρπr2 h dt 5ρπr2 h 8
 2
4E dy  g 
= + y2
5ρπr2 h dt 2h
Y por tanto,
g 2π  g  21
ω2 = → ω= =
2h τ 2h
Finalmente el perı́odo es:
  12
2h
τ = 2π .
g

18. Este problema es mucho más ambicioso que los problemas normales en el
sentido de que exige reunir mayor número de partes. No obstante, si se
resuelven las diversas partes como se sugiere, se verá que aisladamente no
poseen ninguna dificultad especial y que el problema completo sirve de
ejemplo de la potencia del método de la conservación de energı́a para el
análisis de problemas de oscilaciones.

Figura 31: Figura problema 3.18.

Sin duda se está familiarizado con el fenómeno de oscilaciones del agua en


una bañera. El movimiento más sencillo es, con cierta aproximación, aquel

60
en que la superficie del agua está inclinada como se ve en la figura, pero
más o menos lisa. Un fenómeno semejante se presenta en los lagos. Imagi-
nemos un lago de sección recta rectangular, como se indica, de longitud L
y con una profundidad del agua h(<< L). El problema recuerda al de un
péndulo simple, en el que la energı́a cinética se debe casi por completo a
un flujo horizontal del agua, mientras que la energı́a potencial depende de
variaciones de nivel del agua muy pequeñas. El programa para calcular,
aproximadamente, el perı́odo de las oscilaciones es este:
a) Imaginar que en un instante dado el nivel del agua en los extre-
mos es ±y0 respecto al nivel normal. Demostrar que la energı́a potencial
gravitatoria de la masa de agua completa viene dada por
1
U= bρgLy02
6
siendo b la anchura del lago. Para obtener este resultado basta con hallar
el incremento de energı́a potencial de un elemento situado a una distancia
x del centro e integrar.
b) Admitiendo que el flujo del agua es predominantemente horizontal,
su velocidad v debe variar con x, siendo mayor para x = 0 y cero para
x = ± L2 . Como el agua es incompresible (más o menos) podemos relacionar
la diferencia de velocidades de flujo en x y x + dx con el cambio por
unidad de tiempo dy dt de la altura de la superficie del agua en x. Esto
es una condición de continuidad. El agua fluye hacia x a una velocidad
vhb y hacia x + dx a una velocidad (v + dv)hb. (Estamos
  admitiendo que
y0 << h.) La diferencia debe ser igual a (bdx) dydt , que representa el
aumento por unidad de tiempo del volumen de agua contenido entre x y
x + dx. Utilizando esta condición demostrar que
1 2 dy0
v(x) = v(0) − x
hL dt
en donde
L dy0
v(0) =
4h dt
c) A partir de aquı́ demostrar que en un instante cualquiera la energı́a
cinética total asociada con el movimiento horizontal del agua viene expre-
sada por
2
1 bρL3 dy0

K=
60 h dt
para escribir este resultado, se debe calcular la energı́a cinética del ele-
mento de agua comprendido entre x y x + dx (con volumen igual a bhdx)
que se mueve con velocidad v(x) e integrar entre los lı́mites x = ± L2 .
d) Escribir ahora
K + U = const.

61
que es una ecuación de la forma
 2
dy0
A + By02 = const.
dt

y define un MAS de un perı́odo determinado. Puede verse que este perı́odo


depende solo de la longitud L, de la profundidad h y de g. [Nota: Esta
teorı́a no es realmente correcta. La superficie del agua es realmente una
porción de onda sinusoidal y no plana. Pero nuestra fórmula da un error
inferior al 1 %. (La respuesta correcta es τ = √2L
gh
).]
e) El lago de Ginebra puede considerarse aproximadamente como un
depósito rectangular de agua de unos 70 km de longitud y una profundi-
dad media de 150 m. El perı́odo de sus oscilaciones se observa que vale 73
minutos aproximadamente. Comparar este resultado con la fórmula obte-
nida.

Solución

a) Para observar lo que se debe hacer analizamos la gráfica 32:

Figura 32: Figura problema 3.18a.

Realizando el cálculo desde la mitad del lago donde se puso el origen:

d(dU ) = dmgdy = ρdV dy = ρgbydydx


L
Z Z y0   2
2 L y0 1
U = ρgb dx ydy = ρgb = ρgbLy02
0 0 2 2 4

19. Una masa m descansa sobre una mesa horizontal sin rozamiento y está
unida a unos soportes rı́gidos mediante dos resortes idénticos de longitud
sin deformar l0 y constante k (véase figura 33). Ambos resortes se esti-
ran una longitud l considerablemente mayor que l0 . Los desplazamientos

62
horizontales de m respecto a su posición de equilibrio se denominarán x
(sobre AB) e y (perpendicular a AB).

Figura 33: Figura problema 3.19.

a) Escribir la ecuación diferencial del movimiento (es decir, la ley de


Newton) que rige las oscilaciones pequeñas en la dirección x.
b) Escribir la ecuación diferencial del movimiento que rige las oscila-
ciones pequeñas en la dirección y (admitir que y << x).
c) Calcular el cociente entre los perı́odos de oscilaciones sobre x e y
en función de l y l0 .
d) Si para t = 0 se deja libre la masa m desde el punto x = y = A0 con
velocidad nula, ¿cuáles son sus coordenadas x e y en un instante posterior
t?
e) Dibujar un gráfico de la trayectoria de m resultante bajo las con-
diciones de la parte d) si l = 95 l0 .

Solución

Cuando el resorte tiene una longitud inicial l0 , al estar horizontal no hay


fuerzas sobre el. Cuando se elonga a una longitud l >> l0 se genera una
tensión en el resorte de valor Tx0, aplicada = k(l − l0 ), donde esta fuerza
es la que actúa en el resorte en estado de equilibrio. Cuando se desqui-
libra el sistema, la tensión en el resorte es de restitución y por tanto,
Tx = −k[s − (l − l0 )] = max , donde s es la longitud al ser desequilibrado
el resorte. Entonces,
d2 x
m = −k [s − (l − l0 )] = −kx
dt2 | {z }
x

63
a) Tomando un punto P en desequilibrio, esto es un punto donde se loca-
lizarı́a la masa m, se observan sobre el cuerpo las fuerzas (ver figura 34),
que serı́an las de restitución de los resortes idénticos 1 y 2. Como se puede
ver en la figura 34, las dos fuerzas se dirigen hacia el punto de equilibrio
(el origen) para restablecerlo.
Estas fuerzas son iguales debido a que los resortes son idénticos. Por tanto

Figura 34: Figura problema 3.19a

realizando las condiciones de equilibrio de Newton para el eje x se llega a


FN = F1 + F2 , pero F1 = F2 = F y son fuerzas restitutivas que obedecen
la ley de Hooke (el signo negativo es porque son de restitución y no tiene
que ver con el sentido de estas. El mismo resultado serı́a si el punto P
estuviera en la porción a la izquierda del punto de equilibrio). Por tanto

d2 x
m = −2kx
dt2
k
Dividiendo miembro a miembro entre m y usando: ω02 = m,

d2 x
+ 2ω02 x = 0
dt2
Que es la ecuación del oscilador armónico simple con solución (ver texto):
√  √ 
x(t) = A cos 2ω0 t + αA + B sin 2ω0 t + αB

Cuyo perı́odo de oscilación es:



2π 2π
τx = √ =
2ω0 ω 0

b) En este problema se desplaza una distancia y el cuerpo de masa m


(ver figura 35), generándose dos fuerzas idénticas F de restitución parale-
las a cada uno de los resortes (ver figura 35). Estas fuerzas producen una

64
elongación igual a la diferencia entre la longitud del resorte s y la longitud
inicial l de este. Donde s es:
p
s = y 2 + l2

Como l >> y, entonces:

Figura 35: Figura problema 3.19b

r 
y 2
s=l +1
l
Calculando la elongación del resorte da:
r  "r #
y 2  y 2
s−l =l +1−l =l 1+ −1
l l

La condición de equilibrio de Newton genera dos ecuaciones:

Eje x: Fx = −F cos(α) + F cos(α) = 0


Eje y: Fy = −F sin(α) − F sin(α) = −2F sin(α)

Pero
y
sin(α) = p y F = k(s − l)
y 2 + l2
entonces,
y
Fy = −2k(s − l) p
y2 + l2
y por tanto:
"r #
 y 2 y
Fy = −2kl 1+ −1 p
l y2 + l2
"r #
 y 2 y
= −2kl 1+ −1 q
l y 2

l 1+ l

65
Simplificando,
 
 y 2 − 12
(  )
1
Fy = −2k 1 − q  y = −2k 1− 1+ y
y 2
 l
1+ l

Debido a que el problema exige que y << l, entonces se debe aproximar


el resultado y para lograr esto se usa el teorema del Binomio,
1 n 0 n n−1 1 n(n − 1) n−2 2 n(n − 1)(n − 2) n−3 3
(a+b)n = a b + a b + a b + a b +· · ·
0! 1! 2! 3!
h 2 i− 12
para la expresión, 1 + yl . Entonces haciendo la correspondencia
y 2 1

a = 1, b = l y n = − 2 , resulta:
  y 2 − 12 
1 − 1  y 2
0
− 12 (− 1 )−1  y 2 1
  
1+ = 1 2 + 1 2
l 0! l 1! l
1
− 2 − 1 (− 1 )−2  y 2 2
  1 
−2
 
+ 1 2
2! l
1
  1   1    3
−2 − 2 − 1 − 2 − 2 (− 1 )−3 y 2
+ 1 2 + ···
3! l

  y 2 − 21 1  y 2 − 12 − 12 − 1  y 4

1+ =1− +
l 2 l 2 l
− 2 − 2 − 1 − 12 − 2  y 6
1 1
 
+ + ···
6 l
Finalmente los cuatro primeros términos de la serie infinita son:
  y 2 − 12 1  y 2 3  y 4 15  y 6
1+ =1− + − + ···
l 2 l 8 l 48 l
Debido a que el cociente yl es pequeño en magnitud, entonces se corta la
serie desde el segundo término que está elevando este a la cuarta potencia,
quedando:
  y 2 − 12 1  y 2
1+ ≈1−
l 2 l
Reemplazando en la ecuación de la fuerza en el eje y:
    
1  y 2 1  y 2
Fy = −2k 1 − 1 − y = −2k 1 − 1 + y
2 l 2 l
Entonces,
d2 y
   
1 y 2
Fy = m 2 = −2k y
dt 2 l

66
Resultando la ecuación:
d2 y k  y 2
+ y=0
dt2 |m {zl }
ωy2

donde r  
k  y 2 k y
ωy2 = → ωy =
m l m l
y por tanto el perı́odo es:
r    
m l 2π l
τy = 2π → τy =
k y ω0 y

c) √

r√
τx ω0 2y l0
=  = =? 1 −
τy 2π l 2l l
ω0 y

4. Vibraciones forzadas y resonancia


1. Construya la tabla a su elección.
2. Considerar el procedimiento de resolver el movimiento de estado esta-
cionario de un oscilador forzado si la fuerza impulsora es de la forma
F = F0 sin(ωt) en lugar de F = F0 cos(ωt)

Solución

Si se tiene la ecuación del oscilador armónico forzado:


d2 x F0
+ ω02 x = G(ω)
dt2 m
donde G(ω) es una función oscilatoria. Desde el punto de vista matemático,
se soluciona la ecuación de una forma superpuesta, donde se superpones
dos soluciones; la de la ecuación homogenea llamada solución complemen-
taria (xc ) y la de la ecuación no homogenea, llamada particular (xp ). Esto
es x = xc + xp . La ecuación complementaria es:

d2 xc
+ ω02 xc = 0
dt2
que no es nada más y nada menos que el oscilador armónico simple cuya
solución es:
xc = AO(ω0 )

67
donde A es la amplitud y O(ω0 ) es una función de la frecuencia fundamen-
tal del sistema. Para la ecuación particular, se usa un método llamado de
suposición, donde como su nombre lo indica, uno supone una solución par-
ticular de acuerdo con las caracterı́sticas fı́sico matemáticas del sistema.
Entonces se escribe la ecuación particular:
d2 xp F0
+ ω02 xp = G(ω)
dt2 m
y se supone una función particular que sea oscilatoria y continua entre
−2π y 2π, las cuales son seno y coseno. Por tanto se supone

xp = B sin(ωt) + C cos(ωt)

Al derivarla temporalmente dos veces,


d2 xp
= −Bω 2 sin(ωt) − Cω 2 cos(ωt)
dt2
Reemplazando las dos ecuaciones anteriores en la particular, esta queda
como:
F0
−Bω 2 sin(ωt) − Cω 2 cos(ωt) + ω02 [B sin(ωt) + C cos(ωt)] = G(ω)
m
factorizando las funciones:
 2 F0
−ω + ω02 B sin(ωt) + −ω 2 + ω02 C cos(ωt) =
  
G(ω)
m
Luego se igualan los coeficientes de las funciones seno y coseno a miembro
y miembro de la ecuación y por tanto G(ω) puede tener las siguientes
posibilidades:

a) Ser solo seno: G(ω) = EGs (ω)


b) Ser solo coseno: G(ω) = HGc (ω)
c) Ser una combinación lineal de seno y coseno: G(ω) = EGs (ω)+HGc (ω)

donde E y H son amplitudes. Resultando


 2
−ω + ω02 B = E y
 2
−ω + ω02 C = 0
 
a)
 2
−ω + ω02 B = 0 y
 2
−ω + ω02 C = H
 
b)
 2
−ω + ω02 B = E y
 2
−ω + ω02 C = H
 
c)
Las anteriores ecuaciones dan los posibles valores de las amplitudes B y
C desconocidas:
E
a) B = y C=0
−ω 2 + ω02
H
b) B = 0 y C=
−ω 2 + ω02

68
E H
c) B= y C=
−ω 2+ ω02 −ω 2 + ω02
y ası́ las posibles soluciones a la ecuación particular xp = B sin(ω) +
C cos(ω) son:
E sin(ωt)
a) xp =
−ω 2 + ω02
H cos(ωt)
b) xp =
−ω 2 + ω02
E sin(ωt) H cos(ωt) E sin(ωt) + H cos(ωt)
c) xp = + =
−ω 2 + ω02 −ω 2 + ω02 −ω 2 + ω02
Finalmente las soluciones con AO(ω0 ) = Ac cos(ω0 t) + As sin(ω0 t) más
generales son (x = xc + xp ):

E sin(ωt)
a) x = Ac cos(ω0 t) + As sin(ω0 t) +
−ω 2 + ω02

H cos(ωt)
b) x = Ac cos(ω0 t) + As sin(ω0 t) +
−ω 2 + ω02
E sin(ωt) + H cos(ωt)
c) x = Ac cos(ω0 t) + As sin(ω0 t) +
−ω 2 + ω02
Como los estados estacionarios no dependen de las amplitudes, entonces
supongamos que Ac = As = A y E = H = Fm0 , y como lo pide el enunciado
solo se tomará F = F0 sin(ωt):
F0
sin(ωt)
m
x = A[cos(ω0 t) + sin(ω0 t)] +
−ω 2 + ω02

que como ya se sabe (ver problema 2.1(a)) esta queda como:


√  π F0 sin(ωt)
x= 2A cos ω0 t − +
4 m(−ω 2 + ω02 )

Un estado estacionario es cuando la forma de la oscilación no varı́a en el


tiempo y por tanto si graficamos (ver Figura 36):
se observa que es un estado estacionario. Nada especial, las combinaciones
de las soluciones complementarias no tienen relevancia.
3. Un objeto de masa 0.2 Kg se cuelga de un resorte cuya constante es de
N
80 m . El cuerpo se somete a una fuerza resistente dada por −bv = −b dx
dt
siendo v su velocidad m N s

s y b = 4 m .
a) Plantear la ecuación diferencial del movimiento en el caso de
oscilaciones libres del sistema y hallar su perı́odo.
b) Se somete el objeto a una fuerza impulsora sinusoidal dada por
F (t) = F0 sin(ωt), siendo F0 = 2 N y ω = 30 s−1 . En estado estacionario

69
Figura 36: Figura problema 4.2

¿cuál es la amplitud de la oscilación forzada?

Solución

a) La ecuación diferencial de movimiento en el caso de oscilaciones libres


del sistema, es la que no posee fuerza impulsora. Por tanto esta es:

d2 x d2 x
   
dx b dx k
m 2 = −kx − b → 2
+ + x=0
dt dt dt m dt m

Reemplazando los valores dados,

d2 x d2 x
   
4 dx 80 dx
+ + x=0 → + 20 + 400x = 0
dt2 0.2 dt 0.2 dt 2 dt

La solución de esta ecuación como ya se sabe (ver el texto) es:


 
r  2 
2− γ
γ
x = A0 e 2 t cos 

ω0 t
 2 
| {z }
ω0

y por tanto hay tres frecuencias angulares: a) frecuencia fundamental (ω0 ),


b) frecuencia de amortiguamiento (γ) y c) frecuencia del sistema (ω 0 ). La
que nos interesa hallar es la frecuencia del sistema. Por tanto esta es:
r s  2
 γ 2 20
0 2
ω = ω0 − = 400 −
2 2

y entonces
√ 2π 2π 2π
300 = → τ=√ = √
τ 300 10 3
Finalmete el perı́odo es de:
π
τ= √ s
5 3

70
b) Con una fuerza impulsora dada por F (t) = 2 sin(30t), la solución for-
zada está dada por (ver el texto):
F0
cos(ωt − δ)
x = p m2
(ω0 − ω 2 )2 + (γω)2
γω
con tan(δ) = ω02 −ω 2
. Por tanto la amplitud es:

F0
m
A(ω) = p 2
(ω0 − ω 2 )2 + (γω)2

Reemplazando los valores dados,


2
0.2
A(ω) = p = 0.0128 ∼ 0.013 m
(400 − (30)2 )2 + [(20)(30)]2

Finalmente la amplitud es de 1.3 cm.

4. Se conecta un bloque de masa m a un resorte cuyo otro extremo se mantie-


ne fijo. Existe también un mecanismo de amortiguamiento viscoso. Sobre
este sistema se han realizado las siguientes observaciones:
(1) Si se empuja horizontalmente el bloque con una fuerza igual a
mg, la compresión estática del resorte es igual a h.
(2) La fuerza resistente viscosa es igual a mg si el bloque se mueve
con una cierta velocidad conocida u.
(a) Para este sistema completo (en el que se incluyen tanto el re-
sorte como el amortiguador) escribir la ecuación diferencial que rige las
oscilaciones horizontales de la masa en función de m, g, √
h y u.
Responder las siguientes cuestiones en el caso que u = 3 gh:
(b) ¿Cuál es la frecuencia angular de las oscilaciones amortiguadas?
(c) ¿Qué tiempo ha de transcurrir, expresado en forma de un múlti-
h 1
plo de g , para que la energı́a descienda en un factor de e ?
(d) ¿Cuál es el valor Q de ese oscilador?
(e) Este oscilador, inicialmente en su posición de reposo, se pone
en movimiento repentinamente cuando t = 0 mediante un proyectil de
masa despreciable, pero cantidad de movimiento no nula, que se mueve
en sentido positivo de las x. Hallar el valor del ángulo de fase δ en la
γt
ecuación x = Ae− 2 cos(ωt − δ) que describe el movimiento subsiguiente,
y representar x en función de t para los primeros ciclos.
(f)
qSi el oscilador se impulsa con una fuerza igual a mg cos(ωt), sien-
do ω = 2g h ¿Cuál es la amplitud de la respuesta del estado estacionario?

Solución

71
a) Al aplicarle una fuerza de compresión con magnitud mg, el resorte se
comprime una distancia h, y todo esto es antes que comience a moverse.
Por tanto,
d2 x
m 2 = −k(x − h) − bu + mg
dt
que queda como:
d2 x
 
b
+ u + ω02 (x − h) = g
dt2 m
Siendo esta la ecuación diferencial pedida.
d2  d2 x
Con  = x − h y por tanto d dx
dt = dt = u y dt2 = dt2 ,

d2 
 
b
2
+ u + ω02  = g
dt m

donde b d
dt es la fuerza resistente. Si esta es igual a mg, entonces quedarı́a
solo es oscilador armónico simple.

b) Con
d2 
 
b d
+ + ω02  = g
dt2 m dt
cuya solución es:
r !
b b 2
 = A0 e 2m t cos ω02 − t
4m2
√ √
Para u = 3 gh, entonces la fuerza resistente es: mg = b(3 gh), donde
mg
b = 3√ gh
. La frecuencia de amortiguamiento es:
v
u  2
r u mg

s
b2 t 3 gh m2 g 2
ω0 = ω02 − = ω02 − = ω02 −
4m2 4m2 9gh(4m2 )
r
k g
= −
m 36h
Cuando se aplicó la fuerza de magnitud mg, el resorte se comprimió una
distancia h. Usando la Ley de Hooke:
mg
mg = kh ∴ k=
h
Reemplazando,
r r r
g g 36g g 35g
ω0 = − = − =
h 36h 36h 36h 36h

72
Realmente esta no es la frecuencia angular de amortiguamiento γ, sino la
frecuencia angular del sistema amortiguado ω 0 .

c) La energı́a de un sistema amortiguado es:


1 b
E(t) = A0 e− m t
2
Para que descienda a un factor de 1e , se requiere que:
1 b A0 b 1 b
E(t) = A0 e − m t = → e− m t = → emt = e
2 2e e
b
t = ln(e) = 1
m
mg
y entonces usando b = √
3 gh
,
√ s
m 3 gh h
t= = =3 s
b g g

ω0 mg
d) Teniendo en cuenta que Q = γ . Reemplazando los valores de k = h
mg
y b = 3√ gh
:
r r r
k mg g b mg g
ω0 = = = y γ= = √ = √
m mh h m m3 gh 3 gh
Entonces, pg pg √
ω0 h h 3 gh
Q= = = =3
γ √g g
3 gh

γt
e) Usando la ecuación x = Ae− 2 cos(ω 0 t − δ) (recuerden que la frecuencia
angular de la función coseno es la del sistema o sea ω 0 ), se observa que el
enunciado dice el oscilador inicialmente está en reposo por unn proyectil
que viaja en el eje x positvo y todo esto en t = 0 s. Por tanto derivando
temporalmente la ecuación se obtiene:
dx γA − γt γt
=− e 2 cos(ω 0 t − δ) − Aω 0 e− 2 sin(ω 0 t − δ)
dt 2
en t = 0,
γA
− cos(−δ) − Aω 0 sin(−δ) = 0 → −γ cos(−δ) − 2ω 0 sin(−δ) = 0
2
usando la caracterı́stica par de la función coseno y la impar de la función
seno, se llega a:
−γ cos(δ) + 2ω 0 sin(δ) = 0 → γ cos(δ) = 2ω 0 sin(δ)

73
entonces,
γ
tan(δ) =
2ω 0
Pero en el enunciado se observa que x(0) = 0 y por tanto h → 0. ası́:

√g g √1
3 gh h 2
lı́m tan(δ) = lı́m q = q3 lı́m
1 =√
h→0 h→0 35g 35g h→0 √ 35
36h 36 h

resultando
δ = 0.3381 rad = 18.68o
También en t = 0 el resorte está en su posición de reposo y por tanto,
π
x(0) = A cos(−δ) = 0 ∴ δ = rad = 90o
2
La no coincidencia de los resultados se debe a que según el enunciado no
es posible que el sistema esté en la posición de equilibrio con velocidad
nula en t = 0. Según el texto el resultado es δ = π2 , pero es claro en el
enunciado que en t = 0 la posición es x(0) = −h.

f) Siqel oscilador se impulsa con una fuerza igual a mg cos(ωt), siendo


ω = 2g h entonces en general, la amplitud está dada por (ver texto):

F0
A= p
m (ω02 − ω 2 )2 + (γω)2
Reemplazando los valores, donde ω02 = hg , δ = π g
p
2 yγ= 9h :
mg
A= v"
2 2
#
p g q 2g 2
u  q    
u
g 2g
m h − +
t
h 9h h

g g
=q =q
2g 2
2 g 2
g g 2g
− hg 2
 
h − h + 9h h
+ 9 h
r
g h 9
= q =q = h = 0.905h
g
1+ 2 11 11
h 9 9

5. Un péndulo simple tiene una longitud l de un metro. En vibración libre la


amplitud de su oscilador disminuye en un factor e en 50 oscilaciones. El
péndulo se coloca en vibración forzada moviendo su punto de suspensión
horizontalmente con un movimiento armónico simple de amplitud 1 mm.
(a) Demostrar que si el desplazamiento horizontal de la lenteja del
péndulo es x, y el desplazamiento horizontal del soporte es ξ, la ecuación
de movimiento de dicha lenteja en el caso de oscilaciones pequeñas es
d2 x dx g g
+γ + x= ξ
dt2 dt l l

74
Resolver esta ecuación para
 el caso de movimientos estacionarios si ξ =
ξ0 cos(ωt) poner ω02 = gl .
(b) En la resonancia exacta, ¿cuál es la amplitud del movimiento
de la lenteja? (En primer lugar utilizar la información dada para hallar
Q).
(c) ¿Para qué frecuencias angulares es la amplitud la mitad de su
valor de resonancia?

Solución

a) Cuando se mueve horizontalmente el soporte del péndulo en movi-


miento armónico simple, es lo mismo que realizar una fuerza impulsora
oscilatoria. Por tanto en oscilaciones pequeñas el movimiento de la lenteja
es solamente en el eje x, y cuando se impulsa horizontalmente, resulta
una posición relativa η = x − ξ. Entonces para un movimiento oscilatorio
amortiguado se verifica la ecuación de dinámica,

Ftotal = Frestitución + Famortiguamiento

resultando
d2 x dx dx
m 2
= −mω02 η − b = −mω02 (x − ξ) − b
dt dt dt
Dividiendo miembro a miembro entre la masa,
d2 x dx d2 x dx
= −ω02 (x − ξ) − γ → +γ + ω02 x = ω02 ξ
dt2 dt dt2 dt
Finalmente como es un péndulo simple entonces ω02 = gl :

d2 x dx g g
+γ + x= ξ
dt2 dt l l
demostrando lo pedido.

b) El enunciado dice que en vibración libre la amplitud de su oscilador


disminuye en un factor e en 50 oscilaciones. Por tanto usando el resultado
de un oscilador armónico simple amortiguado es
"r #
 γ 2
− γ2 t 2
x = A0 e cos ω0 − t
| {z } 2
A(t)

γ
Usando solo la amplitud A(t) = A0 e− 2 t y reemplazando los valores del
t
enunciado, donde en 50 oscilaciones el perı́odo es: τ = 50 . Pero
r s
g 2π l 2π
ω0 = = → τ = 2π =√
l τ g g

75
entonces
− γ2 100π

A0 e = A0 e g

Resolviendo,
− γ2 100π
√ γ 100π
e=e g
⇒ 1=− √
2 g
Finalmente,

√ g
2 g = −100γπ ⇒ γ= = −0.0199 ∼ −0.02
−50π
es de anotar que 0 ≤ γ ≤ ∞, y por tanto la frecuencia de amortiguamiento
no puede tener valores negativos; por tanto se tendrá en cuenta su valor
absoluto. El próximo paso es usar la ecuación del oscilador armónico simple
amortiguado forzado:
d2 x dx g g
+γ + x = ξ0 cos(ωt)
dt2 dt l l
cuya solución es (la que tiene solo la fuerza impulsora):
gξ0 cos(ωt − δ) γω
x= q y tan(δ) =
2 2 ω02 − ω2
l [ω02 − ω 2 ] + (γω)

donde en resonancia exacta (ω = ω0 ),


gξ0 cos(ω0 t − π2 ) gξ0 sin(ω0 t)
xRE = q =
2 lγω0
l (γω0 )

donde la amplitud en resonancia es


gξ0 gξ0
ARE = =
0.02l gl
p
lγω0
m
con l = 1 m y g = 9.8 s ,

9.8ξ0
ARE = √ = 156.525ξ0 m
0.02 9.8
Con ξ0 = 10−3 m,

ARE = 0.156525 m = 15.6525 cm ∼ 15.7 cm

c) La amplitud de magnitud la mitad del valor de la de resonancia es


(en unidades MKS),
gξ0 0.0098
A= q =q = 0.0775
2 2 2 2
l [ω02 − ω 2 ] + (γω) [ω02 − ω 2 ] + (γω)

76
 2
2 2 2 0.0098
ω02
 
−ω + (γω) = = 0.0159
0.0775
ω 4 + (γ 2 − 2ω02 )ω 2 + (ω04 − 0.0159) = 0

r
2 1 2 2 1 2
ω = − (γ − 2ω0 ) ± (γ − 2ω02 )2 − (ω04 − 0.0159)
2 4
q
= (ω02 − 0.0002) ± (0.012 − ω02 )2 − ω04 + 0.0159
q
= ω02 ± −0.0002ω02 + 0.0159 − 0.0002

Haciendo la aproximación
q √
−0.0002ω02 + 0.0159 ∼ 0.0159 = 0.1261

Entonces
ω 2 = ω02 ± 0.1261 − 0.0002
Aproximando de nuevo, finalmente se llega a

ω 2 = ω02 ± 0.1261 s−2

Debe dar ω = ω0 ± 0.017 s−1 ???


6. Imaginemos un sismógrafo sencillo compuesto por una masa M colgada
mediante un resorte de un montaje rı́gido sujeto a la Tierra, tal como
se indica en la figura 37. La fuerza del resorte y la fuerza amortiguadora
dependen del desplazamiento y de la velocidad relativa de la masa respecto
a la superficie de la Tierra, pero la aceleración que tiene un significado
dinámico es la aceleración de M relativa a las estrellas fijas.
a) Utilizando y para denominar el desplazamiento de M respecto a
la Tierra y η para designar el desplazamiento de la propia tierra, demostrar
que la ecuación del movimiento es

d2 y dy d2 η
2
+γ + ω02 y = − 2
dt dt dt

b) Hallar el valor de y (vibración de estado estacionario) si η =


C cos(ωt).
c) Dibujar un esquema de la amplitud A del desplazamiento y en
función de ω (suponiendo que C es el mismo para todos los valores de ω).
d) Un sismógrafo tı́pico de perı́odo largo tiene un perı́odo de unos
30 s y un Q de 2 aproximadamente. Como resultado de un terremoto
violento la superficie de la Tierra puede oscilar con un perı́odo de unos

77
Figura 37: Figura problema 4.6

20 min y con una amplitud tal que la aceleración máxima sea aproxima-
damente 10−9 sm2 . ¿Cuál será el menos valor de A que sea observable si ha
de ser detectado por el sismógrafo?

Solución

a) Cuando se toman datos de posición, velocidad, etc, se necesita un sis-


tema de referencia y que cumpla las Leyes de Newton. Este sistema se
denomina sistema de referencia inercial, que requiere que se esté estático
o se mueva en movimiento recitlineo a velocidad constante. Se observa
que no existe un sistema con tales requerimiento en el universo, pero se
considera la superficie de la tierra como uno lo suficientemente bueno pa-
ra definir las variables cinemáticas y dinámicas necesarias para solucionar
problemas en general. Se usa para esto las transformaciones de Galileo
(Ver figura 38).
Un sistema no primado está estático y otro primado se mueve en mo-

Figura 38: Figura problema 4.6

78
vimiento rectilineo (eje x) a velocidad constante ~u = ux . Para que las
medidas de los dos coincidan se debe verificar que (ver figura 38):
 0

 x = x − ux t
y 0 = y

~r0 = ~r − ~ut resultando


 z0 = z
0
t =t

donde la última ecuación marca la homogeneidad del tiempo. Si se derivan


temporalmente estas, entonces:

0
vx = vx − ux

0
~v = ~v − ~u resultando vy0 = vy
 0

vz = vz

volviendo a derivar temporalmente,



0
ax = ax

~a0 = ~a resultando a0y = ay
 0

az = az

donde se concluye que la dinámica es la misma para los dos observadores


sin necesidad de una transformación. Estas ecuaciones tomando la tierra
como un sistema de referencia inercial. En este caso el sistema primado es
el que está en cuerpo de masa M , por tanto se usa (en el eje y),
dy dη
vy = vy0 + uy → vy = +
dt dt
Pero el cuerpo de masa M se mueve oscilatoriamente, de forma tal que su
dinámica no es igual a la del sistema no primado. Por tanto,
d2 y d2 η
ay = + 2.
dt2 dt
Por tanto la ecuación de dinámica del sistema es:
 2
d y d2 η

dy
M ay = Frestitución +Famortiguam → M 2
+ 2 = −b −ky = 0
dt dt dt
dividiendo miembro a miembro entre M y despejando,
d2 y dy 2 d2 η
+ γ + ω 0 y = −
dt2 dt dt2

d2 η
b) Con η = C cos(ωt) entonces dt2 = −Cω 2 cos(ωt):

d2 y dy
+γ + ω02 y = Cω 2 cos(ωt)
dt2 dt

79
La solución es (ver el texto):
Cω 2 cos(ωt − δ) γω
y=p con tan(δ) =
(ω02 − ω 2 )2 + (γω)2 (ω02 − ω 2 )

c) La amplitud es,
Cω 2
A= p 2
(ω0 − ω 2 )2 + (γω)2
Si ω tiende a cero,
A→0
Si ω tiene a un número mayor que ω0 y que tiene al infinito,
Cω 2 C
A→ p =q =C
(−ω 2 )2 + (ω)2 1+ 1
ω2

Si ω es igual a ω0 ,
Cω 2 Cω0
AR = p 0 =
(γω0 ) 2 γ
La gráfica es,

Figura 39: Figura problema 4.6

d) Un perı́odo de 30 s, y una Q = 2, lleva a ω0 = 2π π


30 = 15 s y tam-
ω0
bién Q = γ que lleva a γ = 15(2) = 30 s−1 . Estos datos llevan a una
π π

amplitud
Cω 2
A = rh i2   
π 2 π 2

− ω 2 + 30 ω
15

80
En un terremoto se tiene un perı́odo de 20 min = 1200 s que lleva a

ω = 1200 π
= 600 s−1 ; entonces,

π 2

C 600
A = rh
π 2
 π
 2 i2  π
 π
2
15 − 600 + 30 600
C
6002
=q 2 2
1 1 1
152 − 6002 + 18000

y con una amplitud tal que la aceleración máxima sea aproximadamente


10−9 sm2 , entonces

d2 η  π 2 h π i
= Cω 2 cos(ωt) → 10−9 = C cos (1200)
dt2 600 600
donde se obtiene el valor de C
10−9
C= 2  = 0.0000364756
π
 π
600 cos 600 (1200)

Con este valor se halla A,


0.0000364756
6002
A= q 2 2 = 2.28097 × 10−8
1 1 1
152 − 600 2 + 18000

Esto es,
A = 228.097 × 10−10 m = 228.097 Å

7. Consideremos un sistema con una fuerza amortiguadora que sufre unas


oscilaciones forzadas con frecuencia angular ω.
(a) ¿Cuál es la energı́a cinética instantánea del sistema?
(b) ¿Cuál es la energı́a potencial instantánea del sistema?
(c) ¿Cuál es el cociente entre la energı́a cinética media y la energı́a
potencial media? Expresar la respuesta en función del cociente ωω0 .
(d) ¿Para qué valor o valores de ω son iguales la energı́a cinética
media y la energı́a potencial media? ¿Cuál es la energı́a total del sistema
en estas condiciones?
(e) ¿Cómo varı́a la energı́a total del sistema con el tiempo para
un valor arbitrario de ω? ¿Para qué valor o valores de ω es constante la
energı́a total en el tiempo?

Solución

81
(a) Un sistema con una fuerza amortiguadora que sufre unas oscilacio-
nes forzadas con frecuencia angular ω está descrito por la ecuación
d2 x dx F0
+γ + ω02 x = cos ωt
dt2 dt m
cuya solución es,
F cos (ωt + δ)
x(t) = A cos(ω0 t + α) + q 0
2 2
m (ω02 − ω 2 ) + (γω)
| {z }
xp

donde  
γω
δ = arctan 2
ω0 − ω 2
Pero la parte de la solución x(t) que se usa para hallar la energı́a cinética
es la que tiene las propiedades de amortiguamiento e impulso. Esto es,
F cos (ωt + δ)
xp = q 0
2 2
m (ω02 − ω 2 ) + (γω)

El sistema tiene masa constante y la velocidad varı́a respecto al tiempo


como (velocidad instantánea):
dxp F0 ω sin (ωt + δ)
vx = =− q
dt 2 2
m (ω02 − ω 2 ) + (γω)

cuyo cuadrado es,


F02 ω 2 sin2 (ωt + δ)
vx2 = − h
2
i
2
m2 (ω02 − ω 2 ) + (γω)

y por tanto la energı́a cinética instantánea es,


1 F02 ω 2 sin2 (ωt + δ)
K= mvx2 = − h i
2 2
2m (ω 2 − ω 2 ) + (γω)
2
0

Pero debido a que esta energı́a no puede ser negativa, se toma su valor
absoluto resultando,
F02 ω 2 sin2 (ωt + δ)
K= h
2
i
2
2m (ω02 − ω 2 ) + (γω)

(b) La energı́a potencial instantánea es,


Z
U = F~  d~r

82
Hay que incluir la fuerza en esta ecuación; pero es claro que la fuerza
debe ser originada por un potencial como: F~ = −∇U~ , suponiendo que se
conserva la energı́a. Con esto se supone una energı́a potencial U :
1
U= mω02 x2p
2
Entonces la energı́a potencial instantánea es:
 2
1 F0 cos (ωt + δ)
U = mω02  q 
2 2 2
m (ω02 − ω 2 ) + (γω)
F02 ω02 cos2 (ωt + δ)
= h
2
i
2
2m (ω02 − ω 2 ) + (γω)

(c) La energı́a cinética media da (usando el teorema del valor medio):


 Z τ
F02 ω 2

1 2
K= h i sin (ωt + δ) dt
2
2m (ω02 − ω 2 ) + (γω)
2 τ 0

Usando sin2 (φ) = 21 [1 − cos(2φ)], la integral da:

1 τ
Z Z τ
1
cos2 (ωt + δ) dt = [1 − cos (2ωt + 2δ)]dt
τ 0 2τ 0
 
Z τ Z τ
1 
= dt − cos (2ωt + 2δ) dt



0 0 | {z }
=(β)
"Z #
τ Z2(ωτ +δ)
1 1
= dt − cos (β) dβ
2τ 0 2ω 2δ
Z τ Z 2(ωτ +δ)
1 1 1 1 2(ωτ +δ)
= dt − cos (β) dβ = − sin (β)|2δ
2τ 0 4ωτ 2δ 2 4ωτ
entonces,
1 1
= − {sin[2(ωτ + δ)] − sin(2δ)}
2 4ωτ
1 1
= − {sin(2ωτ ) cos(2δ) + cos(2ωτ ) sin(2δ) − sin(2δ)}
2 4ωτ
Pero 2ωτ = 2 2π

τ τ = 4π y entonces usandon sin(4π) = 0 y cos(4π) = 1,
la integral queda:
1 1 1
− [sin(2δ) − sin(2δ)] =
2 8π 2

83
Finalmente el valor medio de la energı́a cinética es,

F02 ω 2
K= h
2
i
2
4m (ω02 − ω 2 ) + (γω)

El valor medio de la energı́a potencial es:


 Z τ
F02 ω0

1 2
U= h i cos (ωt + δ) dt
2
2m (ω02 − ω 2 ) + (γω)
2 2τ 0

Realizando el mismo procedimiento (pero con cos2 (β) = 21 [cos(2β) + 1]):

F02 ω02
U= h
2
i
2
4m (ω02 − ω 2 ) + (γω)

El cociente entre las energı́as medias es:


F02 ω 2
h 2
i 2
( ω0 −ω 2 +(γω)2
) ω2
2

K 4m ω
= F02 ω02
= 2 =
U h 2
i ω0 ω0
4m ( ω0 −ω 2 +(γω)2
2
)

Entonces se verifica que:


ω02 K = ω 2 U

(d) El valor ω para el cual son iguales la energı́a cinética media y la


energı́a potencial media, es cuando el sistema está en resonancia. Esto es
ω = ω0 .

(e) La energı́a total del sistema es:

F02 ω 2 sin2 (ωt + δ) F02 ω02 cos2 (ωt + δ)


E =K +U = h
2
i+ h
2
i
2 2
2m (ω02 − ω 2 ) + (γω) 2m (ω02 − ω 2 ) + (γω)
F02
i ω 2 sin2 (ωt + δ) + ω02 cos2 (ωt + δ)
 
= h
2
2 2 2
2m (ω0 − ω ) + (γω)

que para un valor arbitrario de ω como ω = ω 0 , esta queda:

F02
i ω 02 sin2 (ω 0 t + δ) + ω02 cos2 (ω 0 t + δ)
 
E= h
2 2
2m (ω02 − ω 02 ) + (γω 0 )

que como se observa es oscilatoria con valores siempre positivos.

84
Para que la energı́a sea constante, se necesita que el sistema esté en reso-
nancia (ω = ω0 ):

ω02 F02  2 F02


sin (ω0 t + δ) + cos2 (ω0 t + δ) =

ER = 2
2m (γω0 ) 2mγ 2

8. Se somete una masa m a una una fuerza resistente −bv que no es una
fuerza restauradora como las que ejercen los resortes.
(a) Demostrar que la forma de su desplazamientoen función del
tiempo es
v0
x = C − e−γt
γ
b
donde γ = m.
(b) Para t = 0 la masa está en reposo en la posición x = 0. En ese
instante se pone en funcionamiento una fuerza impulsora F = F0 cos(ωt).
Hallar los valores de A y de δ en la solución del estado estacionario x =
A cos(ωt − δ).
(c) Escribir la solución general [suma de las partes (a) y (b)] y
hallar los valores de C y de v0 mediante las condiciones x = 0, dx dt = 0
para t = 0. Hacer un esquema de x en función de t.

Solución

(a) La ecuación de dinámica para el sistema es FN = FR . Esto es:

d2 x dx d2 x dx
m 2
= −b → 2
+γ =0
dt dt dt dt
resultando  
d dx dx
+ γx = 0 ∴ + γx = C 0
dt dt dt
entonces,
dx
= C 0 − γx
dt
con: u = C 0 − γx y entonces du
dt = −γ dx
dt ,

1 du du
− =u → = −γu
γ dt dt
usando el concepto de antiderivada, la ecuación diferencial queda:
Z Z
du du
= −γdt → = −γ dt
u u

ln(u) = −γt + C1 → u = e−γt+C1 = eC1 e−γt

85
reemplazando u

C 0 − γx = eC1 e−γt → γx = C 0 − eC1 e−γt

despejando x
C0 eC1 −γt
x= − e
γ γ
C0
Siendo γ una constante, entonces C = γ y debido a que es una ecuación
eC 1
para x, γ debe tener unidades de longitud; de esta manera eC1 tiene
unidades de velocidad pues γ tiene dimensiones de frecuencia. Se define
C1
entonces e γ = vγ0 , resultando

v0 −γt
x(t) = C − e
γ

(b) Se impone la condición inicial x(0) = 0. La ecuación forzada es:

d2 x dx
m = −b + F0 cos(ωt)
dt2 dt
d2 x dx F0
+γ = cos(ωt)
dt2 dt m
La solución para la ecuación homogenea es
v0 −γt
xc (t) = C − e
γ
La ecuación particular es

d2 xp dxp F0
+γ = cos(ωt)
dt2 dt m
cuya solución se supone como: xp = A cos(ωt) + B sin(ωt). Reemplazando,

F0
−ω 2 A cos(ωt) − ω 2 B sin(ωt) + γ [−ωA sin(ωt) + ωB cos(ωt)] = cos(ωt)
m
F0
−ω 2 A + γωB cos(ωt) − ω 2 B − γωA sin(ωt) =
 
cos(ωt)
m
donde igualando coeficientes,
F0
−ω 2 A + γωB = y − ω 2 B − γωA = 0
m
solucionando el sistema,
γ
B=− A
ω

86
y por tanto reemplazando en la primera ecuación se halla el valor de A,
 γ  F F0
0
−ω 2 A + γω − A = → A=−
ω m m(ω 2 + γ 2 )
y ası́,  
γ F0 γF0
B=− − =
ω m(ω 2 + γ 2 ) mω(ω 2 + γ 2 )
Siendo la solución particular,
F0 cos(ωt) γF0 sin(ωt)
xp = − +
m(ω 2 + γ 2 ) mω(ω 2 + γ 2 )
y la solución general,
v0 −γt F0
x(t) = C − e + [−ω cos(ωt) + γ sin(ωt)]
γ mω(ω 2 + γ 2 )
donde para la parte forzada,
 
 
F − p ω γ
p 0
 
xp = cos(ωt) + p sin(ωt)
mω ω 2 + γ 2 
 ω2 + γ 2 ω2 + γ 2 

| {z } | {z }
cos(δ) sin(δ)

F
xp = p 0 [− cos(δ) cos(ωt) + sin(δ) sin(ωt)]
mω ω 2 + γ 2
esto es,
F
xp = p 0 cos(ωt + δ)
mω ω 2 + γ 2
donde se observa claramente que la amplitud es
F
A= p 0
mω ω 2 + γ 2
y que
γ
tan(δ) = −
ω

(c) la condición inicial x(0) = 0 y dx dt (0) = 0 no tiene sentido desde el


punto de vista fı́sico porque me está informando que en t = 0 el sistema
está en el estado equilibrio (esto es con aceleración nula) y no se mueve.
O sea está estático, lo que no concuerda con un sistema oscilatorio. La
solución general es
v0 −γt F
x(t) = C − e + p 0 cos(ωt + δ)
γ mω ω 2 + γ 2

87
que para t = 0 da:
v0 F
x(0) = C − + p 0 cos(δ) = 0
γ mω ω 2 + γ 2
despejando C,
v0 F
C= − p 0 cos(δ)
γ mω ω 2 + γ 2
La velocidad es,
dx ωF
= v0 e−γt − p 0 sin(ωt + δ)
dt mω ω 2 + γ 2
que para t = 0 da:

dx ωF
= v0 − p 0 sin(δ) = 0
dt t=0 mω ω 2 + γ 2
despejando v0 ,
ωF
v0 = p 0 sin(δ)
mω ω 2 + γ 2
y reemplazando en C,
ωF F
C= p 0 sin(δ) − p 0 cos(δ)
2
mωγ ω + γ 2 mω ω 2 + γ 2
ωF γF
= p 0 sin(δ) − p 0 cos(δ)
2
mωγ ω + γ 2 mωγ ω 2 + γ 2
Finalmente el valor de C es:
F
C= p0 [ω sin(δ) − γ cos(δ)]
mωγ ω 2 + γ 2
y la solución usando los valores de C y v0 da:

F
x(t) = p0 [ω sin(δ) − γ cos(δ)]
mωγ ω 2 + γ 2
ωF0 sin(δ) F
− p e−γt + p 0 cos(ωt + δ)
mωγ ω + γ 2 2 mω ω 2 + γ 2
que agrupando términos queda:
F
p0 ω sin(δ) 1 − e−γt + γ [cos(ωt + δ) − cos(δ)]
 
x(t) =
mωγ ω 2 + γ 2
Si nos apegamos a la fı́sica entonces la velocidad en t = 0 es diferente de
cero (tomamos un valor conocido u), y entonces:

dx ωF
= v0 − p 0 sin(δ) = u
dt t=0
mω ω 2 + γ 2

88
con lo que
ωF
v0 = u + p 0 sin(δ)
mω ω2 + γ 2
dando un valor de C:
u ωF F
C= + p 0 sin(δ) − p 0 cos(δ)
γ 2
mωγ ω + γ 2 mω ω 2 + γ 2

que resulta para el desplazamiento,

u ωF γF
x(t) = + p 0 sin(δ) − p 0 cos(δ)
γ mωγ ω 2 + γ 2 mωγ ω 2 + γ 2
" #
u ωF0 γF
− + p sin(δ) e−γt + p 0 cos(ωt + δ)
γ 2
mωγ ω + γ 2 mωγ ω 2 + γ 2

agrupando, se llega a la ecuación


del desplazamiento x(t) con las condi-
ciones iniciales x(0) = 0 y dx
dt t=0 = u:

u F
1 − e−γt + p0 ω sin(δ) 1 − e−γt − γ [cos(ωt + δ) − cos(δ)]
  
x(t) =
γ mωγ ω 2 + γ 2

9. (a) Un oscilador amortiguado forzado de masa m tiene un desplazamiento


variable con el tiempo dado por x = A sin(ωt). La fuerza resistente es −bv.
A partir de esta información calcular cuánto trabajo se realiza contra la
fuerza resistente durante un ciclo de oscilación.
(b) En el caso de una frecuencia impulsora ω menor que la frecuencia na-
tural ω0 , dibujar los gráficos correspondientes de la energı́a potencial, la
energı́a cinética y la energı́a total del oscilador en un ciclo completo. Cui-
dar que se señalen los puntos e intersecciones importantes con los valores
de su energı́a y tiempo.

Solución

(a) La fuerza resistente es FR = −b dx


dt con x = A sin(ωt)

FR = −bAω cos(ωt)

El trabajo diferencial es (en contra de la fuerza resistente hace que sea


negativo):

dW = bAω cos(ωt)dx = −bAω cos(ωt)Aω cos(ωt)dt


= bA2 ω 2 cos2 (ωt)dt

89
Por tanto,
τ τ  
cos(2ωt) − 1
Z Z
2 2 2 2 2
W = bA ω cos (ωt)dt = bA ω dt
2
0 0
Z τ Z τ
bA2 ω 2   bA2 ω 2 bA2 ω
=  cos(2ωt)dt − dt = τ= (ωτ )
2 
0 0
 2 2
| {z }
=0
bA2 ω 2π
 
= τ = bA2 ωπ
2 τ

(b) Primera posibilidad. En el caso de una frecuencia impulsora ω me-


nor que la frecuencia natural ω0 (esto es un oscilador lejano de la zona de
resonancia), y usando el desplazamiento variable con el tiempo dado por
x = A sin(ωt), donde se observa que la constante de fase δ es nula, lo que
significa:  
γω
δ = arctan =0
ω02 − ω 2
lo que implica que ω02  ω 2 , y por tanto el sistema es quasi-libre (la fuerza
impulsora no tiene mucho efecto sobre el cuerpo oscilante). Ası́ la energı́a
es aproximadamente la de un oscilador armónico simple amortiguado. Para
la velocidad se tiene que,
(ωt)2 (ωt)4
 
dx
= Aω cos(ωt) = A 1 − + + ···
dt 2! 4!
 
1
≈ A 1 − (ωt)2 (debido a que ω es pequeño en magnitud)
2
que al elevarla al cuadrado,
 2  2
dx 2 1 2
= A2 1 − (ωt)2 + (ωt)4
 
= A 1 − (ωt)
dt 2
que por la misma razón se aproxima a
 2
dx
= A2 1 − (ωt)2
 
dt
y la energı́a cinética es
1
mA2 1 − (ωt)2
 
K=
2
El cuadrado de la posición es:
2
(ωt)3 (ωt)5

2 2 2
x = [A sin(ωt)] = A (ωt) − + − · · · ≈ A2 (ωt)2
3! 5!

90
por tanto la energı́a potencial es (asumiendo un comportamiento con la
Ley de Hooke),
1 1
U = mω 2 x2 = mω 2 A2 (ωt)2
2 2
La gráfica es (se puede ver que la gráfica decrece indefinidamente para la
energı́a cinética y crece indefinidamente para la potencial de forma tal que
la suma de las dos da constante en el tiempo para la energı́a mecánica),
Donde se usaron valores grandes para ver bien la gráfica. Si se toman

Figura 40: Figura problema 4.9

valores pequeños de ω (como se deberı́a hacer), las paraábolas serı́an tan


abiertas que no se apreciarı́a bien la gráfica. El punto de corte de las dos
gráficas es,
1  1
mA2 1 − (ωt)2 = mω 2 A2 (ωt)2

2 2
1 − (ωt)2 = ω 2 (ωt)2

Esto es cuando
r
1 1
ω 2 (ω + 1)t2 = 1 → t=
ω ω+1
Esto lleva a un problema. La energı́a cinética se vuelve compleja cuando
(ωt)2 > 1, mientras que la potencial es siempre real. Por tanto esta apro-
ximación solo es válida para 0 ≤ (ωt)2 ≤ 1.

91
Segunda posibilidad. Tomar el movimiento como un oscilador armónico
simple amortiguado, pero eso harı́a ω = 0, lo que no estarı́a de acuerdo
con el problema que dice que ω es menor que la frecuencia natural ω0 .
Aunque si δ=0, da la posibilidad de ω = 0. Se observa que
 
r  2 
1
x(t) = Ae− 2 γt cos  2− γ

ω0 t
 2 
| {z }
ω0

y la velocidad es:
 
dx 1 1
= Ae− 2 γt − γ cos (ω 0 t) − ω 0 sin (ω 0 t)
dt 2
dx 2
Usando K = 21 m y U = 12 m(ω 0 )2 x2 , las energı́as son:

dt

1
U= mA2 e−γt (ω 0 )2 cos2 (ω 0 t)
2
γω 0
  
1 γ 2
K= mA2 e−γt cos2 (ω 0 t) + (ω 0 )2 sin2 (ω 0 t) + sin (2ω 0 t)
2 2 2
y
1
mγA2 e−γt γ cos2 (ω 0 t) + ω 0 sin (2ω 0 t)
 
E=
4
Las gráficas son,

Figura 41: Figura problema 4.9

92
10. La potencia de entrada para mantener las vibraciones forzadas puede cal-
cularse observando que esta potencia es la velocidad media con que se hace
el trabajo contra la fuerza resistente −bv.
(a) Comprobar por si mismo que la velocidad instantánea de rea-
lización de trabajo contra esta fuerza es igual a bv 2 .
(b) Utilizando la ecuación x = A cos(ωt − δ), demostrar que la
2 2
velocidad media con que se realiza el trabajo es bω 2A .
(c) Sustituir el valor de A para cualquier frecuencia arbitraria y a
partir de aquı́ obtener la expresión de P según se desprende de la ecuación
F02 ω0 1
P (ω) = 2
2kQ ω0

ω 1
ω − ω0 + Q2

Solución

(a) La fuerza es bv, sin el signo negativo porque es aplicada y no hecha


por el sistema. El trabajo diferencial es,
dW = bvx dx = bvx (vx dt) = bvx2 dt
dw

La velocidad instantánea de este trabajo potencia P = dt es,
dW dt
= bvx2 = bvx2 = bv 2
dt dt

(b) La velocidad media con la que se realiza el trabajo, se halla con el


teorema del valor medio; esto es,
1 τ
Z
dw
= F dv
dt τ 0
Usando F = F0 cos(ωt) y x = A cos(ωt − δ) esto es vx = −ωA sin(ωt − δ),
y por tanto dvx = −ω 2 A cos(ωt − δ)dt:
1 τ
Z
dw
F0 cos(ωt) −ω 2 A cos(ωt − δ)dt
 
=P =
dt τ 0
−ω 2 AF0 τ
Z
= cos(ωt) cos(ωt − δ)dt
τ 0
−ω 2 AF0 τ
Z
= cos(ωt) [cos(ωt) cos(δ) − sin(ω) sin(δ)]
τ 0
resultando,
 Z τ Z τ 
−ωAF0
P = cos(δ) cos2 (ωt)dt − sin(δ) cos(ωt) sin(ωt)dt
τ 0 0
−ωAF0 cos(δ) τ sin(δ) τ
 Z Z 
= [cos(2ωt) + 1] dt − sin(2ωt)dt
τ 2 0 2 0

93
integrando y debido a que 2ωτ = 4π,
 
−ωAF0 cos(δ) ωAF0 sin(δ)
P = τ =
τ 2 2
γω γω
Pero tan(δ) = ω02 −ω 2
y entonces cos(δ) = q
2
reemplazando,
(ω02 −ω2 ) −(γω)2

ωAF0 γω
P = q
2 2
(ω02 − ω 2 ) − (γω)2
bω 2 A F0 bω 2 A2
= q =
2 2 2
m (ω02 − ω 2 ) − (γω)2
| {z }
=A

ω0
(c) Usando Q = γ , entonces

F0 1
A= r
mω0 ω  ω ω
2
1
0
ω − ω0 + Q2

y por tanto la poencia media es

F02 ω0 1
P (ω) = 2
2kQ ω0

ω 1
ω − ω0 + Q2

se usa un valor arbitrario de ω, por ejemplo ω = aω0 , donde a > 0.


Entonces,

F02 ω0 1 F02 ω0 1
P (aω0 ) = 2 = 2
2kQ 2kQ 1 1

ω0
− aω0
+ 1
a −a + Q2
aω0 ω0 Q2

F02 ω0 a2 Q2 F 2 ω0 a2 Q
= 2 = 0
2kQ Q2 (1 − a2 ) + 1 2k Q2 (1 − a2 )2 + 1
a2 F02 ω0 Q
=
2k [Q (1 − a2 )]2 + 1

11. Consideremos un oscilador amortiguado con m = 0.2 Kg, b = 4 Nmṡ y


N
k = 80 m . Supóngase que este oscilador se ve impulsado por una fuerza
F = F0 cos(ωt) siendo F0 = 2 N y ω = 30 rads
(a) ¿Cuáles son los valores de A y δ de la respuesta de estado
estacionario descrita por x = A cos(ωt − δ)?
(b) ¿Cuánta energı́a de ve disipada contra la fuerza resistente en
un ciclo?

94
(c) ¿Cuál es la potencia media de entrada?

Solución

(a) La ecuación de aceleración para este sistema es la de un Oscilador


Armónco Simple Amortiguado Forzado:

d2 x dx F0
2
+γ + ω02 x = cos(ωt)
dt dt m
que con los datos queda,

d2 x dx
+ 20 + 400x = 10 cos(30t)
dt2 dt
Cuya solución particular es (ver el texto):

F cos(ωt − δ)
xp (t) = q 0
2
m (ω02 − ω 2 ) + (γω)2

donde
F0 2
A= q = q
2 2
m (ω02 − ω 2 ) + (γω)2 0.2 (400 − 302 ) + [(20)(30)]2
10
=p = 0.0128 m = 1.28 cm
(400 − 900)2 + (600)2

También,
   
γω (20)(30)
δ = arctan 2 = arctan = −50.194 = 129.81o
ω0 − ω 2 400 − 302

(b) La fuerza resistente es FR = F0 cos(ωt). Por tanto,


 
1 τ 1 τ −F −
Z Z
0 ω sin(ωt δ)dt
E= F0 cos(ωt)dxp = F0 cos(ωt)  q 
τ 0 τ 0 2 2 2 2
(ω0 − ω ) + (γω)
2 Z τ
F0 ω
=− q cos(ωt) sin(ωt − δ)dt
2
τ (ω02 − ω 2 ) + (γω)2 0

Pero cos(ωt) sin(ωt − δ) = 12 sin(2ωt) cos(δ) − cos2 (ωt) sin(δ). Por tanto las
integrales son,

cos(δ) τ
Z
cos(δ) τ cos(δ)
sin(2ωt)dt = |cos(2ωt)|0 = (1 − 1) = 0
2 0 4ω 4ω

95
y
τ
sin(δ) τ
Z Z
2
− sin(δ) cos (ωt)dt = − [cos(2ωt) + 1]dt
0 2 0
   
sin(δ) 1 τ τ sin(δ) 1
=− |sin(2ωt)|0 + |t|0 = − (0 − 0) + τ
2 2ω 2 2ω
Reemplazando y teniendo en cuenta que la energı́a es positiva,

F02 ω F02 ω sin(δ)


 
sin(δ)
E= q τ = q
2 2 2
τ (ω02 − ω 2 ) + (γω)2 2 (ω02 − ω 2 ) + (γω)2

Reemplazando los valores,

22 (30) sin(129.81o ) 60 sin(129.81o )


E= q =q = 0.059 J
2 2
2 (400 − 302 ) + (30)2 (20)2 (400 − 302 ) + (30)2 (20)2

(c) Usando Q = 400 20 = 20 y k = 400(0.2) = 80, la potencia media está


dada por (ver el texto):

F02 ω0 1 22 (20) 1
P (ω) = 4(ω0−ω)2
= 4(20−30)2
= 0.025 W att
2kQ + 1 2(80)(20) + 1
ω02 Q2 400 202

12. Un objeto de masa 2 Kg cuelga de un resorte de masa despreciable. El


resorte se alarga 2.5 cm cuando se le sujeta dicho objeto. El extremo
superior del resorte se hace oscilar hacia arriba y hacia abajo con un
movimiento armónico simple de amplitud 1 mm. La Q del sistema es 15.
(a) ¿Cuál es ω0 para este sistema?
(b) ¿Cuál es la amplitud de oscilación formada para ω = ω0 ?
(c) ¿Cuál es la potencia media de entrada para mantener la oscila-
ción formada a una frecuencia 2 % mayor que ω0 ? [Está justificado el uso
de la fórmula aproximada Ec. (4-26).]

γF02 1
(aproximada) P (ω) = (4 − 26)
2m 4(ω0 − ω)2 + γ 2

Solución

(a) Como se alarga 2.5 cm = 0.025 m con un peso de W = F = mg =


2(9.8) = 19.6 N , entonces F = kx (es positivo debido a que es una fuerza
aplicada y no de restitución):
19.6 N
F = kx → 19.6 = 0.025k → k= = 784
0.025 m

96
Por tanto la frecuencia fundamental para este sistema es,
r
784 rad
ω0 = = 19.8
2 s

(b) Cómo Q = 15, entonces γ = 15


19.8 = 0.76 s−1 . Con este dato, la
amplitud es (ver texto):

F0 ω0
A=   12
2
ω0
kω ω − ωω0 + 1
Q2

Para ω = ω0 , la amplitud es:


F0 ω0 F0 Q F0
A= 1 = = 15
kω0 Q k k

F0
Pero k es la amplitud del movimiento impulsor; esto es 1 mm. Por tanto

A = 15(0.001) = 1.5 cm

(c) Para la potencia media de entrada se usa la ecuación aproximada,


2
γF02 γk 2

1 F0 1
P (ω) = =
2m 4(ω0 − ω)2 + γ 2 2m k 4(ω0 − ω)2 + γ 2
F0
con ω = 1.02ω0 (2 %); esto es ω = 1.02(19.8) = 20.2 s−1 y usando m =
0.001,

0.76(784)2 2 1
P = (0.001) = 0.096 W att
2(2) 4(19.8 − 20.2)2 + 0.762

13. El gráfico 42 muestra la curva de resonancia de potencia de un determinado


sistema mecánico cuando se ve accionado por una fuerza F0 sin(ωt), en
donde F0 =constante y ω es variable.
(a) Hallar los valores numéricos de ω y Q para ese sistema.
(b) Se suprime la fuerza impulsora. ¿Después de cuántos ciclos de
oscilación libre ha descendido la energı́a del sistema a e15 de su valor ini-
cial? (e = 2.718.) (Con una buena aproximación, el perı́odo de la oscilación

libre puede hacerse igual a ω 0
.)

Solución

(a) Se observa en el gráfico 42 (Potencia vs frecuencia angular) que el

97
Figura 42: Figura problema 4.12

valor máximo es de 40 rad


s , lo que significa que es el valor de resonancia,
o sea cuando ω = ω0 . Por tanto ω0 = 40 rad s . Pero también se obser-
va que la potencia máxima va disminuyendo a lado y lado de la gráfica
debido al amoritguamiento del sistema. Por tanto el sistema debe tener
un valor de Q (que es adimensional) relacionado con el valor medio de la
altura de la potencia en resonancia. Por tanto se puede analizar que la
diferencia de valores de ω entre los dos puntos de igual potencia debe ser
aproximadamente la frecuencia de amortiguamiento, debido a que como se
mencionó anteriormente, la caı́da de potencia se debe al amortiguamiento
medio γ = 12 γ . Por tanto,


ω0 40
γ= → Q= 1
Q 2γ

También en la gráfica se oberva que: ω0 − 12 γ = 38 y que ω0 + 21 γ = 42;


esto es:
1 1 1
40 − γ = 38 → γ = 40 − 38 → γ = 2 s−1
2 2 2
o
1 1 1
40 + γ = 42 → γ = 40 − 42 → γ = 2 s−1
2 2 2
Finalmente,
40
Q= = 20
2

(b) Al suprimir la fuerza impulsora, la cinemática del sistema es la de un

98
oscilador armónico simple amortiguado

d2 x dx
2
+γ + ω02 x = 0
dt dt
Cuya solución es,
 γ 2  12
( )
− 12 γt
x = Ae cos ω02 − t
2

Con amplitud amortiguada,


1
AA = Ae− 2 γt

y por tanto con energı́a (ver el texto),


1 1
E= mω02 A2A → E= mω02 A2 e−γt
2 2
| {z }
E0

Por tanto,

E = E0 e−5 = E0 e−γt → e−5 = e−γt → 5 = γt

Hallando el tiempo con γ = 2 s−1 ,


5
t= = 2.5 s
2
2π 2π
Pero ω0 = 40 = τ y ası́ el perı́odo es, τ = 40 = 0.16 s. Por tanto t = N τ
y ası́,
2.5
N= = 15.625 ciclos
0.16
14. La figura 43 muestra la potencia media de entrada P en función de la
frecuencia impulsora en el caso de una masa situada sobre un resorte con
amortiguamiento. (Fuerza impulsora = F0 sin(ωt), siendo F0 constante y
ω variable.) La Q es suficientemente elevada para que la potencia media
de entrada, que es máxima para ω0 , disminuya hasta la mitad del máximo
para las frecuancias 0.98ω0 y 1.02ω0 .

(a) ¿Cuál es el valor numérico de Q?


(b) Si se suprime la fuerza impulsora, la energı́a disminuye de
acuerdo con la ecuación
E = E0 e−γt
¿Cuál es el valor de γ?
(c) Si se elimina la fuerza impulsora, ¿qué fracción de energı́as se
pierde por ciclo?

99
Figura 43: Figura problema 4.14

Se construye un sistema nuevo en el que se duplica la constante del resor-


te, pero se mantiene sin variar la masa y el medio viscoso, y se aplica la
misma fuerza impulsora F0 sin(ωt). En función de las magnitudes corres-
pondientes del sistema original hallar los siguientes valores:
(d) La nueva frecuencia de resonancia ω00 .
(e) El nuevo factor de calidad Q0 .
(f) La potencia de entrada máxima P m .
(g) La energı́a total del sistema en la resonancia, E00 .

Solución

(a) Usando la relación mencionada en el enunciado, se llega a:


1
ω0 + γ = 1.02ω0 → 0.04ω0 = γ
| 2 {z }
γ=∆ω0

por tanto
ω0 1
=Q= = 25
γ 0.04

(b) Sin la fuerza impulsora el sistema es de oscilaciones libres y su potencia


de entrada media varı́a según la gráfica 43. Por tanto, usando la ecuación
de la pregunta anterior, la frecuencia de amortiguamiento está dada por:

γ = 0.04ω0

100
(c) De nuevo como un oscilador armónico simple amortiguado, se tiene que
la frecuencia media de amortiguamiento es la variación de la frecuencia
fundamental; esto es, γ = ∆ω0 . Por tanto,
2π 0.08π
γ = 0.04ω0 = 0.04 =
τ τ
y entonces el perı́odo es
0.08π
τ=
γ
La fracción de energı́a perdida en un ciclo es el logaritmo natural del
cociente de energı́as; esto es porque si tomamos solo el cociente, tendremos
el factor de pérdida de energı́a. Por tanto,
 
E 0.08π
ln = −γτ = −γ = −0.08π
E0 γ

De acá en adelante k 0 = 2k, y por tanto (ω 0 )20 = 2ω02 .


(d) Con esto, la nueva frecuencia de resonancia es,
q q √
(ω 0 )20 = 2ω02 → ω00 = 2ω0

(e) El nuevo factor de calidad Q0 es,


ω00 √ ω0 √
Q0 = = 2 = 2Q
γ γ

(f) La potencia de entrada máxima está justificada con la ecuación apro-


ximada
γF02 1
P 0 (ω) = 0
2m 4(ω0 − ω)2 + γ 2
que en resonancia (para su valor máximo) es:

F02
P 0 (ω) = = P (ω)
2mγ

(g) La energı́a total del sistema en la resonancia, E00 debe se E0 , ya que el


valor de la potencia máxima es invariante.
15. Se observa que las oscilaciones libres de un sistema mecánico tienen una
cierta frecuencia angular ω1 . El mismo sistema cuando se ve accionado por
una fuerza F0 cos(ωt) (donde F0 = constante y ω es variable), tienen una
curva de resonancia de potencia cuya anchura es de la frecuencia angular
a potencia mitad del máximo es ω51 .

101
(a) ¿Para qué frecuencia angular se produce la máxima potencia
de entrada?
(b) ¿Cuál es el valor Q del sistema?
(c) El sistema se compone de una masa m sobre un resorte de
constante k. En función de m y k, ¿cuál es el valor de la constante b en el
término resistivo −bv?
(d) Dibujar la curva de respuesta de la amplitud, marcando algu-
nos puntos caracterı́sticos de la misma.

Solución

(a) Con ω+ = ω1 + 12 γ y ω− = ω1 − 21 γ, de forma tal que

1 1 1
ω+ − ω− = ω1 + γ − (ω1 − γ) = γ ' ω1
2 2 5
donde
ω0
ω1 ' 5γ = 5
Q
y por tanto,
Q
ω0 ' ω1
5
pero hasta acá se puede llegar. No hay datos para continuar.
(b) Teniendo en cuenta que la anchura de la frecuencia angular a potencia
mitad del máximo es aproximadamente ωQ0 , entonces con ω0 = ω1 :

ω1 1 ω1 ω1
= γ ' ω1 → ' → Q'5
Q 5 Q 5
Si se observa el punto (a), con un valor de Q aproximadamente igual a 5,
el valor de ω0 ' ω1 (pero eso sı́ ω0 > ω1 ). De nuevo, solo se puede llegar
hasta acá.
b
(c) γ = m , entonces,

1√ √
r r
b 1 1 k m k
' ω1 = → b' = km = 0.2 km
m 5 5 m 5 m 5
(d) No hay datos para hacerla.
16. En el caso del sistema eléctrico de la figura 44, hallar
(a) La frecuencia de resonancia, ω0 .
(b) La anchura de resonancia, γ.
(c) La potencia absorbida en la resonancia.

Solución

102
Figura 44: Figura problema 4.16

(a) Es un circuito RLC con corriente alterna cuyas caı́das de potencial


son VR = IR, VC = Cq y VL = L dIdt . Usando la segunda ley de Kirchkoff
(suma de voltales=0 → VR + VC + VL = 0),

dI q
L + IR + = V0 cos(ωt)
dt C
con ecuación reducida igual a
dI R 1 V0
+ I+ q= cos(ωt)
dt L LC L
en función de la carga la ecuación es

d2 q R dq 1 V0
+ + q= cos(ωt)
dt2 |{z}
L dt |{z}
LC L
γ ω02

Comparando con el oscilador amortiguado forzado

d2 x dx F0
2
+γ + ω02 x = cos(ωt)
dt dt m
se observa que la frecuencia angular es,
r
1 1
ω02 = → ω0 =
LC LC

(b) De la ecuación la frecuencia de amortiguamiento es

R
γ=
L

103
(c) La potencia absorbida por la resonancia es, usando la ecuación apro-
ximada para el sistema mecánico,

γF02 1
P =
2m 4(ω0 − ω)2 + γ 2
que en resonancia es,
F02 F2
P res = = 0
2mγ 2b
Para el sistema eléctrico se hace la correspondencia F0 → V0 y b → R.
Entonces la potencia es
2
V02 (I0 R) I 2R
P res = = = 0
2R 2R 2

17. El gráfico 45 indica la potencia media absorbida por un oscilador cuando


se ve impulsada por una fuerza de valor constante, y frecuencia angular
variable ω.
(a) En la resonancia justa, ¿cuánto trabajo por ciclo se efectúa
contra la fuerza resistente? (perı́odo = 2π
ω )

Figura 45: Figura problema 4.17

(b) A la resonancia justa, ¿cuál es la energı́a mecánica total E0 del


oscilador?
(c) Si se elimina la fuerza impulsora, ¿cuántos segundos transcu-
rren antes de que la energı́a disminuya a un valor E = E0 e−1 ?

Solución

104
(a) En la resonancia justa ω = ω0 = 1 × 106 s−1 , y por tanto el perı́odo
−6
es, τ = 2π
ω = 2π × 10 s. De esta forma el trabajo hecho en un ciclo, está
dado por el producto entre la potencia en resonancia y el perı́odo; esto es
W = P (ω0 )τ :

W = P (1 × 106 )τ = 10(2π × 10−6 ) = 2π × 10−5 J

(b) Se observa en la gráfica 45 que


1 1
1×106 + γ = 1.005×106 → γ = 0.005×106 → γ = 0.01×106 s−1
2 2
y entonces γ = 1 × 104 s−1 . En la gráfica también se puede analizar que la
lı́nea de gráfica tiene unidades (sin tener
 en cuenta los radianes, tal como
aparece en la gráfica) de: Pω = Ws−1
 att
= [W att · s = J] = E Por tanto se
verifica en la resonancia pura que,

E = 10 1 × 106 = 1 × 107 J


(c) Sin la fuerza impulsora, es un oscilador armónico simple amortiguado


con energı́a E = E0 e−γt . Entonces para hallar el tiempo se verifica la
ecuación

E0 e−1 = E0 e−γt → e−1 = e−γt → 1 = γt → t = γ −1

y con el valor hallado de γ, el tiempo da el valor t = 1 × 10−4 s.

5. Osciladores acoplados y modos normales


1. Hágalo en casa.
2. Se unen dos péndulos idénticos mediante un resorte de acoplamiento ligero.
Cada péndulo tiene una longitud de 0.4 m y están situados en un lugar en
donde g = 9.8 sm2 . Estando conectado el resorte de acople, se sujeta uno
de los dos péndulos y se encuentra que el perı́odo del otro es de 1.25 s
exactamente.
(a) Si ninguno de los dos péndulos está sujeto, ¿cuáles son los
perı́odos de los dos modos normales?
(b) ¿Cuál es el intervalo de tiempo entre dos amplitudes posibles
máximas sucesivas de un péndulo después que uno de ellos se retira late-
ralmente y luego se deja en libertad?

Solución
pg
(a) La frecuencia fundamental del péndulo sin acoplar es ω0 = l =

105
q
9.8
= 4.95 rad
0.4 s . Por tanto el primer modo de oscilación (la oscilación de
ambos es como si fuera al oscilación de uno solo) es:
s
l 2π
τ1 = 2π = = 1.27 s
g 4.95

La frecuencia de oscilación del acople está dada cuando estando conectado


el resorte de acople y se sujeta uno de los dos péndulos. Se encuentra que
el perı́odo del otro es de τc = 1.25 s exactamente; por tanto,
2π rad
ωc = = 5.03
1.25 s
Teniendo en cuenta que el acople es quien transfiere la energı́a de un cuerpo
al otro, se observa que si se sujeta cada uno de los cuerpos va a resultar
una suma de dos frecuencias angulares de acople dadas por 2ωc y esta será
igual a la suma de las dos frecuencias angulares de los modos de oscilación.
Por tanto (donde el modo fundamental es el primer modo ω0 = ω1 ),

2ωc = ω0 + ω2 → ω2 = 2ωc − ω0 = 2(5.03)2 − 4.952 = 26.09


√ 2π
∴ ω2 = 26.09 = 5.11 =
τ2
Finalmente,

τ2 = = 1.23 s
5.11
(b) El sistema acoplado tiene como solución (ver texto):
   
1 ω2 − ω0 ω0 + ω2
x = A0 cos t cos t
2 2 2
| {z }
Aenvolvente

Y por tanto las dos amplitudes posibles máximas sucesivas se hallan con
Aenvolvente . Para sus valores máximos sucesivos (entre +A y −A) hay una
fase de π. Por tanto,
ω2 − ω0 2π 2π 2π
t=π → t= = = = 39.27 s
2 ω2 − ω0 5.11 − 4.95 5.11 − 4.95

3. Una masa m cuelga de un resorte de constante k. En la posición de equili-


brio estático, la longitud del resorte es l. Si se retira lateralmente y luego
se deja en libertad, el movimiento subsiguiente será una combinación de
(a) oscilaciones pendulares y (b) alargamiento y compresión del resorte.
sin utilizar apenas las matemáticas, considérese el comportamiento de este
dispositivo como un sistema aislado.

Solución

106
Las ecuaciones del péndulo simple son:

Fk = T − mg cos(θ)
F⊥ = −mg sin(θ)

Si fuera una cuerda ideal entonces la fuerza paralela a la cuerda se anula


debido a que no hay movimiento y por tanto la acelración es nula. Pero
como ya no es una cuerda sino un resorte esta fuerza es de restitución y
obedece la ley de Hooke; de esta forma la tensión T se convierte en la
fuerza de restitución. Por tanto las ecuaciones son (observe el cambio de
signos cuando hay un resorte en la ecuación paralela),

d2 r
Fk = m = −kr + mg cos(θ) + Fck
dt2
d2 θ
F⊥ = ml 2 = −mg sin(θ) + Fc⊥
dt
donde queda por determinar las fuerzas de acople paralela y perpendicu-
lar. Pero debido a que el movimiento es simultáneo y por tanto acoplado,
debe ser proporcional a la aceleración angular multiplicada por la masa
m, mα = m dr ω para la fuerza perpendicular Fc⊥ y debe ser proporcional
 dt
dθ 2
a ml dt para la fuerza paralela Fck (se pueden verificar las correspon-
dencias de unidades). Finalmente al multiplicar por la masa,
 2

Fck = Cpk ml
dt
dr
Fc⊥ = Cp⊥ m ω
dt
De esta manera quedan las ecuaciones,
2
d2 r


Fk = m 2 = −kr + mg cos(θ) + Cpk ml
dt dt
d2 θ dr
F⊥ = ml 2 = −mg sin(θ) + Cp⊥ m ω
dt dt
donde las constantes de proporcionalidad Cpk y Cp⊥ quedan por determi-
nar. El movimiento de este sistema es acoplado donde primero se mueve
como un sistema masa-resorte y paulatinamente va cambiando a movi-
miento pendular mientras disminuye el movimiento oscilatorio longitudi-
nal.
4. Dos osciladores armónicos A y B, de masa m y constantes kA y kB respec-
tivaente, se acoplan juntos mediante un resorte de constante kc . Hallar las
frecuencias normales ω 0 y ω 00 y describir los modos normales de oscilación
si kc2 = kA kB .

107
Solución

Las ecuaciones de dinámica de los osciladores son Fj = FR,n + Tc,n donde


j = A, B. La fuerza de acople obedece la ley de Hooke; esto es tiene una
forma Fc,A = −kc (xA − xB ), donde es claro que depende de la elongación
relativa por ser de acople y obedece para el oscilador A la ley de Hooke:
Fc,A = −kc xA + kc xB . Las ecuaciones de movimiento quedan como:

d2 xA
m = −kA xA − kc (xA − xB )
dt2
d2 xB
m 2 = −kB xB − kc (xB − xA )
dt
Cuyas ecuaciones reducidas son,
d2 xA 2
+ ωA xA + ωc2 (xA − xB ) = 0
dt2
d2 xB 2
+ ωB xB + ωc2 (xB − xA ) = 0
dt2
Las soluciones son de la forma (ver texto), xj = Aj eiωt 6= 0 donde la fre-
cuencia angular ω es desconocida. Reemplazando se hallan las ecuaciones
caracterı́sticas,

− ω 2 A + ωA
2
A + ωc2 (A − B) = 0
− ω 2 B + ωB
2
B + ωc2 (B − A) = 0

factorizando,

−ω 2 + ωA
2
+ ωc2 A − ωc2 B = 0


− ωc2 A + −ω 2 + ωB
2
+ ωc2 B = 0


construyendo la ecuación matricial,


 2 2
+ ωc2 −ωc2
   
−ω + ωA A 0
=
−ωc2 −ω 2 + ωB
2
+ ωc2 B 0

donde    
A 0
6=
B 0
Por tanto se calcula el determinante
 2 2
+ ωc2 −ωc2

−ω + ωA
det =0
−ωc2 −ω 2 + ωB
2
+ ωc2

Resultando

−ω 2 + ωA
2
+ ωc2 −ω 2 + ωB
2
+ ωc2 − −ωc2 −ωc2 = 0
   

108
ω 4 − ω 2 ωA2 2
+ 2ωc2 + ωA 2 2
+ ωc2 ) + ωc2 (ωB2
+ ωc2 ) − ωc4 = 0

+ ωB (ωB
ω 4 − ωA 2 2
+ 2ωc2 ω 2 + [ωA2 2
ωB + ωc2 (ωA
2 2

+ ωB + ωB )] = 0
4 2 2 2 2 4 2 2 2

ω − ωA + ωB + 2ωc ω + [ωc + ωc (ωA + ωB )] = 0

Resolviendo la ecuación cuadrática para ω 2 ,


 n 2 2 o 21
2 2
ωA + ωB + 2ωc2 ± ωA 2
+ ωB + 2ωc2 − 4[ωc4 + ωc2 (ωA
2 2
+ ωB )]
ω2 =
2
n 2 o 21
2 2
ωA + ωB + 2ωc2 − 4[ωc4 + ωc2 (ωA
2 2
+ ωB )]
2 2 2

ωA + ωB + 2ωc
= ±
2 2
( 2 ) 21
2 2 2 2
 
ωA + ωB ωA + ωB
= + ωc2 ± + ωc2 − [ωc4 + ωc2 (ωA
2 2
+ ωB )]
2 2

expandiendo los términos de la raı́z cuadrada,


 2 2
2
ωA + ωB
+ ωc2 − [ωc4 + ωc2 (ωA 2
+ ωB2
)]
2
 2 2
2
ωA + ωB
= + ωc2 − (ωc2 + ωA 2
+ ωB 2
)ωc2
2
 2 2
2
ωA + ωB 2 2 2
= + ωc − (ωA + ωB )ωc2 − ωc4
2
Expandiendo el binomio,
 2 2
2  2 2

ωA + ωB ωA + ωB
+2 ωc2 + ωc4 − (ωA
2 2
+ ωB )ωc2 − ωc4
2 2
 2 2
2  2 2

ωA + ωB ωA + ωB
= +2 ωc2 − (ωA2 2
+ ωB )ωc2
2 2
 2 2
2
ωA + ωB
=
2
Por tanto la solución cuadrática queda,
 2 2
  2 2

ωA + ωB ωA + ωB
ω2 = + ωc2 ±
2 2
y entonces,
2 2
   2 2

2 ωA + ωB ωA + ωB
ω− = + ωc2 − = ωc2
2 2
 2 2
  2 2

2 ωA + ωB ωA + ωB
ω+ = + ωc2 + = ωA2 2
+ ωB + ωc2
2 2

109
usando ωc2 = ωA ωB :
2
ω− = ωc2
2 2 2
ω+ = ωA + ωB + ωA ωB

resultan los modos de oscilación:



q
ω− = ωA ωB y ω+ = 2 + ω2 + ω ω
ωA B A B

5. Se acoplan dos osciladores idénticos sin amortiguar A y B, cada uno de


ellos de masa m y frecuencia natural(angular)
 ω0 de tal modo que la fuer-
2
za de acople ejercida sobre A es αm ddtx2B , y la fuerza de acople ejercida
 2 
sobre B es αm ddtx2A , siendo α una constante de acople de magnitud
menor que 1. Describir los modos normales del sistema acoplado y hallar
sus frecuencias.

Solución
Las ecuaciones del sistema son de la forma F = FR + Fc :
d2 xA d2 xB
m = −kxA + αm
dt2 dt2
d2 xB d2 xA
m 2 = −kxB + αm 2
dt dt
con ecuaciones reducidas,
d2 xA d2 xB
2
+ ω02 xA − α 2 = 0
dt dt
d2 xB 2 d 2
xA
2
+ ω 0 x B − α =0
dt dt2
Con soluciones xA = Aeiωt 6= 0 y xB = Beiωt 6= 0. Reemplazándolas:

− ω 2 A + ω02 A + αω 2 B = 0
− ω 2 B + ω02 B + αω 2 A = 0

Factorizando,

(ω02 − ω 2 )A + αω 2 B = 0
αω 2 A + (ω02 − ω 2 )B = 0

Despejando el cociente de amplitudes,


A αω 2
=− 2
B (ω0 − ω 2 )
A (ω 2 − ω 2 )
=− 0 2
B αω

110
Igualando los cocientes,

αω 2 (ω 2 − ω 2 )
− =− 0 2
(ω02 −ω )2 αω
entonces,

αω 2 (αω 2 ) = (ω02 − ω 2 )(ω02 − ω 2 ) → α2 ω 4 = (ω02 − ω 2 )2


αω 2 = ω02 − ω 2 → (α + 1)ω 2 = ω02

Despejando,
ω02 ω0
ω2 = ∴ ω= 1
1+α ±(1 + α) 2
Los modos de oscilación son:
1
ω = ±ω0 (1 + α)− 2
1  2
Teniendo en cuenta que (1+α)− 2 = 1− 21 α+ − 12 − 12 + 1 α2! +· · · , pero

1
como α < 1, entonces la serie se corta como (1 + α)− 2 ' 1 − 12 α. También
1
se tiene que (1 − α)− 2 ' 1 + 12 α. Por tanto el signo lo maneja la constante
de acople α. Esto hace que los modos de oscilación sean realmente:
1
ω = ω0 (1 ± α)− 2

Esto es,
1 1
ω+ = ω0 (1 + α)− 2 y ω− = ω0 (1 − α)− 2
o también,
   
1 1
ω+ = ω0 1 − α y ω− = ω0 1+ α
2 2

6. Dos masas iguales situadas sobre una vı́a horizontal de aire sin ningún
rozamiento, se mantienen entre soportes rı́gidos mediante tres resortes
idénticos, como se indica en la figura 46. Los desplazamientos a partir
del equilibrio sobre la lı́nea de los resortes están descritos mediante las
coordenadas xA y xB . Si una de las masas se sujeta, el perı́odo τ = 2π ω
correspondiente a una vibración completa del otro es 3 s.

Figura 46: Figura problema 5.6

111
(a) Si ambas masas están libres, ¿cuáles son los perı́odos de los dos
modos normales del sistema? Dibujar los gráficos de xA y xB en función
de t en cada modo. Para t = 0, la masa A estén su posición de reposo
normal y la masa B se ve empujada lateralmente a una distancia de 5 cm.
Las masas se dejan en libertad estando en reposo en ese instante.
(b) Escribir una ecuación para el desplazamiento subsiguiente de
cada masa en función del tiempo.
(c) ¿Qué duración (en segundos) caracteriza la transferencia pe-
riódica del movimiento de B a A y vuelta de nuevo a B? Después de un
ciclo, ¿se reproduce exactamente la situación que tenı́amos para t = 0?
Explicar.

Solución
(a) Debido a que los tres resortes son idénticos, las ecuaciones de movi-
miento son:
d2 xA
+ ω02 xA + ω02 (xA − xB ) = 0
dt2
y
d2 xB
+ ω02 xB + ω02 (xB − xA ) = 0
dt2
que llevan a
−ω 2 + 2ω02 −ω02
 
det =0
−ω02 −ω 2 + 2ω02
(−ω 2 + 2ω02 )2 = ω04 → ω 2 = 2ω02 ± ω02
y los modos de oscilación son
q √
ω1 = ω0 y ω2 = 3ω02 = 3ω0

Una de las masas sujetas lleva a (ver figura 47):

Figura 47: Figura problema 5.6b

donde se observa que la ecuación de movimento es;


d2 xA 2
+ 2ωA xA = 0
dt2

112
por tanto la frecuencia
√ angular fundamental es ω02 = 2ωA
2
= (2)3 = 6 y
entonces ω0 = 6. Ası́ las frecuencias angulares son:
√ √ √ √
ω1 = 6 y ω2 = 3 6 = 3 2.

Estos resultados usan el valor del enunciado del problema como la fre-
cuencia angular y no como el perı́odo. Usándolo como el enunciado dice,
2
la frecuencia angular fundamental es ω02 = 2ωA 2
= 2 2π3 y entonces

2 2π
ω0 = 3 . Ası́ las frecuencias angulares son:
√ √ ! √
2 2π √ 2 2π 2 6π
ω1 = y ω2 = 3 =
3 3 3

Usando las ecuaciones:


d2 xA d2 xB
+ 2ω02 xA − ω02 xB = 0 y + 2ω02 xB − ω02 xA = 0
dt2 dt2
Sumando las ecuaciones con x+ ≡ xA + xB ,

d2 x+
+ ω02 x+ = 0 ∴ x+ = A+ eiω0 t
dt2
Restando las ecuaciones con x− ≡ xA − xB ,

d2 x−
+ 3ω02 x− = 0 ∴ x− = A− ei3ω0 t
dt2
Entonces usando xA = 12 (x+ + x− ) y xB = 21 (x+ − x− ),

1 1
A+ eiω0 t + A− ei3ω0 t A+ eiω0 t − A− ei3ω0 t
 
xA = y xB =
2 2
Con A = A+ = A− ,
1 1
A eiω0 t + ei3ω0 t A eiω0 t − ei3ω0 t
 
xA = y xB =
2 2
Usando la función coseno,
1 1
xA = A [cos(ω0 t) + cos(3ω0 t)] y xB = A [cos(ω0 t) − cos(3ω0 t)]
2 2
con α + β = ω0 t y α − β = 3ω0 t, y usando los cosenos de suma de ángulos,
las soluciones quedan:
   
ω0 − 3ω0 ω0 + 3ω0
xA = A cos t cos t
2 2
   
ω0 − 3ω0 ω0 + 3ω0
xB = A sin t sin t
2 2

113
simplificando y usando la caracterı́stica de función par de coseno e impar
de seno,

xA = A cos (ω0 t) cos (2ω0 t)


xB = −A sin (ω0 t) sin (2ω0 t)

Usando el valor ω0 = 6, quedan como
√   √ 
xA = A cos 6t cos 2 6t
√   √ 
xB = −A sin 6t sin 2 6t

y sus gráficas son:

Figura 48: Figura problema 5.6

114
donde la lı́nea continua es para xA y la punteada para xB . Las gráficas
que tiene cada función son las envolventes o amplitudes moduladas. adi-
cionalmente se tienen las condiciones iniciales:

dxA dxB
xA (0) = xA,equil , xB (0) = xB,equil ±0.05 m, = 0, =0
dt t=0 dt t=0

(b) Las ecuaciones son:


√   √ 
xA = A cos 6t cos 2 6t
√   √ 
xB = −A sin 6t sin 2 6t

que para t = 0 dan xA = A y xB = 0, lo que no concuerda con las


condiciones iniciales. Entonces deben ser modificadas mediante el uso de
constantes de fase. Se recuerda que esto no afecta las ecuaciones de movi-
miento debido a que son constantes y sus derivadas son nulas, pero sirven
para controlar las condicones iniciales. De esta forma se pueden usar las
soluciones desacopladas:

x+ = Aeiω0 t+φ+ y x− = Aei3ω0 t+φ−

usando xA = 12 (x+ + x− ) y xB = 12 (x+ − x− ),


1 1
A eiω0 t+φ+ + ei3ω0 t+φ− A eiω0 t+φ+ − ei3ω0 t+φ−
 
xA = y xB =
2 2
Pero como se observa no es posible resolverla analı́ticamente a no ser que
φ+ = φ− = φ. Por tanto,
1 1
A eiω0 t+φ + ei3ω0 t+φ A eiω0 t+φ − ei3ω0 t+φ
 
xA = y xB =
2 2
entonces,

xA = A cos (ω0 t + φ) cos (2ω0 t + φ)


xB = −A sin (ω0 t + φ) sin (2ω0 t + φ)

con ω0 = 6,
√   √ 
xA (t) = A cos 6t + φ cos 2 6t + φ
√   √ 
xB (t) = −A sin 6t + φ sin 2 6t + φ

Para t = 0:
π
xA (0) = A cos2 (φ) = 0 → φ=
2
reemplazando en xB ,
π
xB (0) = −A sin2 = 0.05 → A = −0.05
2

115
Entonces las ecuaciones son:
√ π  √ π
xA (t) = 0.05 cos 6t + cos 2 6t +
√ 2   √ 2 
π π
xB (t) = −0.05 sin 6t + sin 2 6t +
2 2
Finalmente quedan:
√   √ 
xA (t) = 0.05 sin 6t sin 2 6t
√   √ 
xB (t) = −0.05 cos 6t cos 2 6t

(c) En la figura 49 se observa la transición que se busca en el área som-


breada.

Figura 49: Figura problema 5.6

Comenzando de B, hace que xB sea máximo, y esto es se debe cumplir en

116
√ 
la envolvente que (ver figura 49): sin 6t = 1, su solución da
√  √ π π π π
sin 6t = 1 → 6t = , 5 , 9 , 13 , . . .
2 2 2 2
esto es,
√ π
6t = 2πn + n = 0, 1, 2, 3, ...
2
Finalmente
  √
2n 1 6
t= √ + √ π → t= (4n + 1) π
6 2 6 12
√ 
también se debe cumplir para A que (ver figura 49), cos 2 6t = −1 y
su solución da  √ 
cos 2 6t = −1

Después de un ciclo, ¿se reproduce exactamente la situación que tenı́amos


para t = 0? R/ Se observa que en un ciclo se ha recorrido 2π rad de fase.
Esto es,
√  √  √  √ 
sin 6t + 2π = sin 6t cos (2π) + cos 6t sin (2π) = sin 6t
 √   √   √   √ 
cos 2 6t + 2π = cos 2 6t cos (2π) − sin 2 6t sin (2π) = cos 2 6t
√  √  √  √ 
cos 6t + 2π = cos 6t cos (2π) − sin 6t sin (2π) = cos 6t
 √   √   √   √ 
sin 2 6t + 2π = sin 2 6t cos (2π) + cos 2 6t sin (2π) = sin 2 6t

Las ecuaciones siempre son las mismas bajao un corrimiento de fase de 2π


o una translación temporal de un perı́odo τ .

7. Se conectan dos objetos, A y B, cada uno de ellos de masa m, mediante


resortes, según se ve en la figura. El resorte de acople tiene una constante
kc , y los otros dos tienen una cosntante k0 . Si se sujeta B, A vibra con
una frecuencia de 1.81 s−1 . La frecuencia ν1 del modo normal inferior es
1.14 s−1 .

Figura 50: Figura problema 5.7

(a) Comprobar personalmente que las ecuaciones de movimiento

117
de A y de B son:
d2 xA
m = −k0 xA − kc (xA − xB )
dt2
d2 xB
m 2 = −k0 xB − kc (xB − xA )
dt
q
(b) Poniendo ω0 = km0 , demostrar que las frecuencias angulares
ω1 y ω2 de los modos normales vienen dadas por:
   12
2 2kc
ω1 = ω0 ω2 = ω0 +
m
y que la frecuencia angular de A cuando se sujeta B(xB = 0 siempre)
viene dada por
   12
2 kc
ωA = ω0 +
m
(c) Utilizando los datos numéricos anteriores, calcular la frecuencia
esperada (ν2 ) del modo normal má alto. (el valor observado fue 2.27 s−1 .)
kc
(d) A partir de estos mismos datos calcular el cociente k0 de las
dos constantes de los resortes.

Solución
(a) Las ecuaciones de dinámica son Fj = FR,j + Fc,j donde j = A, B.
d2 x
Donde la fuerza neta es Fj = m dt2j , la fuerza de restitución está dada
por FR,j = −k0 xj y la fuerza de acople que depende directamente de la
elongación relativa es: FR,j = −kc (xj − xj+1 ). Por tanto:
d2 xA
FA j = FR,A + Fc,A → m = −k0 xA − kc (xA − xB )
dt2
d2 xB
FB = FR,B + Fc,B → m 2 = −k0 xB − kc (xB − xA )
dt
 q 
(b) Las ecuaciones reducidas son poniendo ω0 = km0 ,

d2 xA
 
2 kc
+ ω0 xA + (xA − xB ) = 0
dt2 m
d2 xB
 
kc
+ ω02 xB + (xB − xA ) = 0
dt2 m
Sus soluciones son xj = Aj cos ωt 6= 0. Reemplazando
 
kc
− ω 2 A + ω02 A + (A − B) = 0
m
 
kc
− ω 2 B + ω02 B + (B − A) = 0
m

118
    
kc kc
−ω 2 + ω02 + A− B=0
m m
    
kc kc
−ω 2 + ω02 + B− A=0
m m
Entonces,
kc kc
 
m A m
A=  kc
 B → = 2 kc

−ω 2 + ω02 + m
B −ω + ω02 + m
kc kc
  2 
m A −ω + ω02 + m
B=  kc
 A → = kc

−ω 2 + ω02 + m
B m

igualando el cociente de las amplitudes,


kc kc
  2 
m −ω + ω02 + m
=
−ω 2 + ω02 + kmc kc
  
m

solucionando,
  2  2  
kc kc kc kc
−ω 2 + ω02 + = → −ω 2 + ω02 + =±
m m m m
Finalmente los modos de oscilación son,
(
ω1 = ω0
   
2 kc kc
ω = ω02 + ± → q
kc

m m ω2 = ω02 + 2 m

Cuando xB = B cos ωt = 0 y se sujeta siempre, entonces B = 0. La


frecuencia fundamental desaparece y queda solamente un acople con un
soprte fijo. Entonces,
d2 xA d2 xA
    
2 kc 2 kc
2
+ ω0 xA + xA = 2
+ ω0 + xA = 0
dt m dt m
| {z }
2
ωA

y por tanto
   12
2 kc
ωA = ω0 +
m

(c) Con ν2 = 2.27 s−1 , entonces


   12
1 kc
ν2 = 2
ω0 + 2 = 2.27 s−2
2π m
Pero según el enunciado
   12
1 kc rad
νA = 2
ω + = 1.81 s−1 ∴ ωA = 11.37
2π 0 m s

119
q
k0
y también ν1 = ν0 = 1.14 s−1 ; por tanto ω0 = 7.16 rad
s = m y por
tanto, k0 = 51.27m Se llega entonces a:
   12  
2 kc 2 2 kc
11.37 = 7.16 + → 11.37 − 7.16 =
m m
 
kc
= 78.01 ∴ kc = 78.01m
m
También
   12  
2 kc kc
ω0 + 2 = 2π(2.27) → ω02 +2 = 203.43
m m

Por tanto,
   
kc 203.43 − 51.27
kc
7.162 + 2 = 203.43 → = = 76.08
m m 2

donde hay una discrepancia para el valor de kmc debido a las condiciones


adicionales del problema xB = 0.


(d) El cociente de las constantes de los resortes es (usando los dos valores):

kc,1 78.01m kc,2 76.08m


= = 1.52 y = = 1.48
k0 51.27m k0 51.27m

8. (a) Se aplica una fuerza F en el punto A de un péndulo, según está


indicado. ¿Para qué ángulo θ  (1 radián) se presenta la nueva posición
de equilibrio? ¿Qué fuerza F 0 , aplicada a m, producirı́a el mismo resulta-
do?

Figura 51: Figura problema 5.8

Dos péndulos idénticos compuestos de masas iguales montados sobre va-


rillas rı́gidas y sin peso se disponen del modo indicado. El acoplamiento

120
lo proporciona un resorte ligero (no deformado cuando ambas varillas son
verticales y colocado en la situación indicada).
(b) Escribir las ecuaciones diferenciales del movimiento para osci-
laciones de pequeña amplitud en función de θ1 y θ2 . (Despreciar el amor-
tiguamiento.)
(c) Describir el movimiento de los péndulos en cada uno de los
modos normales.
(d) Calcular las frecuencias de los modos normales del sistema.
[indicación; puede explotarse la simetrı́a del sistema con ventaja,
particularmente en las partes (c) y (d), en tanto que las respuestas obte-
nidas de este modo sean comprobadas en las ecuaciones.]

Solución
(a) Para producir el mismo resultado, debe haber una relación lineal entre
la fuerza aplicada y el desplazamiento que debido a que se tratan oscila-
ciones pequeñas, el arco s de trayectoria del cuerpo se convierte en una
distancia x horizontal. Por tanto se verifican las relaciones (ver figura 51),
F F0 a a
= → F0 = F = − kx
a L L L

(b) Teniendo en cuenta que el desplazamiento del cuerpo está a una dis-
tancia L, entonces se desplaza un arco sj ; pero el resorte está más arriba
y describe un desplazamiento
xa xL a a
= → xa = xL ≡ θ
a L L L
Las ecuaciones de movimiento son:
d2 s1
m 2 = −mg sin(θ1 ) − k(sc,1 − sc,2 )
dt
d2 s2
m 2 = −mg sin(θ2 ) − k(sc,2 − sc,1 )
dt
reemplazando lo anteriormente escrito se llega a que para el acople sc,j =
aθ y con − La kx quedan las ecuaciones reducidas
d2 s1 a k 
= −g sin(θ 1 ) − (aθ1 − aθ2 )
dt2 L m
d2 s2 a k 
= −g sin(θ 2 ) − (aθ2 − aθ1 )
dt2 L m
Pero sj = Lθj ; entonces,
d2 θ1
  2
k a
L + g sin(θ 1 ) + (θ1 − θ2 ) = 0
dt2 m L
d2 θ2
  2
k a
L 2 + g sin(θ2 ) + (θ2 − θ1 ) = 0
dt m L

121
Dividiendo miembro a miembro entre L,

d2 θ1  g 
 
k a
2
+ sin(θ1 ) + (θ1 − θ2 ) = 0
dt L m L2
d2 θ2  g 
 
k a
2
+ sin(θ2 ) + (θ2 − θ1 ) = 0
dt L m L2

Debido a que son oscilaciones pequenãs se aproxima sin(θ) ' θ quedando


las ecuaciones de movimiento definidas como:
d2 θ1  g 
  
k a 2
2
+ θ1 + (θ1 − θ2 ) = 0
dt L m L
d2 θ2  g 
  
k a 2
2
+ θ 2 + (θ2 − θ1 ) = 0
dt L m L

(c,d) sumando las dos ecuaciones con θ+ = θ1 + θ2 ,

d2 θ+  g   g  12
+ θ+ = 0 ∴ ω+ =
dt2 L L
cuyo movimiento es → → ó ← ←
Restando las dos ecuaciones con θ− = θ1 − θ2 ,

d2 θ−  g 
  
k a 2
2
+ θ − + 2 θ− = 0
dt L m L
    21
d2 θ−  g 
       
k a 2 g k a 2
+ + 2 θ− = 0 ∴ ω− = +2
dt2 L m L L m L
cuyo movimiento es → ← ó ← →
9. La molécula de CO2 puede semejarse a un sistema constituido por una
masa central m2 unida por resortes iguales de constante k a dos masas m1
y m3 (siendo m1 = m3 )

Figura 52: Figura problema 5.9

(a) Plantear y resolver las ecuaciones para los dos modos normales
en los cuales las masas oscilan a lo largo de la recta que une sus centros.
2
[La ecuación de movimiento para m3 es m3 ddtx23 = −k(x3 − x2 ) y pueden
escribirse ecuaciones semejantes para m1 y m2 .]

122
(b) Haciendo m1 = m3 = 16 unidades, y m2 = 12 unidades, ¿cuál
será el cociente de las frecuencias de ambos modos, admitiendo que fuese
aplicable esta descripción clásica?

Solución

(a) Las ecuaciones de movimiento siguiendo el hecho de que F = FR + Fc


(donde la fuerza de restitución siempre es cero debido a que no hay sopor-
tes para los resortes) son:
d2 x1
m1 = −k(x1 − x2 )
dt2
d2 x2
m2 2 = −k(x2 − x1 ) − k(x2 − x3 )
dt
d2 x3
m3 2 = −k(x3 − x2 )
dt
Sus soluciones son de la forma xj = Aj eiωt 6= 0; por tanto,
−m1 ω 2 A1 = −k(A1 − A2 )
−m2 ω 2 A2 = −k(A2 − A1 ) − k(A2 − A3 )
−m3 ω 2 A3 = −k(A3 − A2 )
factorizando,
(−m1 ω 2 + k)A1 − kA2 = 0
− kA1 + (−m2 ω 2 + 2k)A2 − kA3 = 0
− kA2 + (−m3 ω 2 + k)A3 = 0
la ecuación matricial es
−m1 ω 2 + k
    
−k 0 A1 0
 −k −m2 ω 2 + 2k −k  A2  = 0
0 −k −m3 ω 2 + k A3 0
donde,    
A1 0
A2  6= 0
A3 0
Por tanto para m1 = m3 = mO y m2 = mC ,
−mO ω 2 + k
 
−k 0
det  −k −mC ω 2 + 2k −k =0
2
0 −k −mO ω + k
Solucionando,
(−mO ω 2 + k) (−mC ω 2 + 2k)(−mO ω 2 + k) − (−k)(−k)
 

− (−k)(−k)(−mO ω 2 + k) = 0

123
(−mC ω 2 + 2k)(−mO ω 2 + k)2 − k 2 (−mO ω 2 + k) − k 2 (−mO ω 2 + k) = 0
agrupando,

(−mO ω 2 + k) (−mO ω 2 + k)(−mC ω 2 + 2k) − 2k 2 = 0


 

Resultando las ecuaciones,

−mO ω 2 + k = 0 y (−mO ω 2 + k)(−mC ω 2 + 2k) − 2k 2 = 0

Entonces el primer modo de oscilación es:


r
k
ω1 =
mO
La segunda,

−mC ω 2 (−mO ω 2 + k) + 2k(−mO ω 2 + k) = 2k 2

mC mO ω 4 − kmC ω 2 − 2kmO ω 2 + 2k 2 = 2k 2
mC mO ω 4 − kmC ω 2 − 2kmO ω 2 = 0
Factorizando,

ω 2 (mC mO ω 2 −kmC −2kmO ) = 0 ω 2 mC mO ω 2 − k(mC + 2mO ) = 0


 

resultando las ecuaciones


 
mC + 2mO
ω2 = 0 y ω2 = k
mC mO
y por tanto los otros dos modos de oscilación,
s  
mC + 2mO
ω2 = 0 y ω3 = k
mC mO

Resumiendo los modos de oscilación describen los movimientos,


r
k
ω1 = → · ← ó ← · →
mO
ω2 = 0 → → → ó ← ← ←
s  
mC + 2mO
ω3 = k → → ← ó ← ← →
mC mO

donde en el modo ω2 no hay oscilación sino translación.


(b) Solo es posible un cociente. Este es,
r   v 
k mm C +2mO
 s
u mC +2mO
ω3 C m O
u k mC mO

mC + 2mO

= = = mO
t
k
q
ω1 k
m
mC mO
mO O

124
y entonces r
ω3 mC + 2mO
=
ω1 mC
Usando los valores mO = 16 unidades, y mC = 12 unidades,
r
ω3 12 + 2(16)
= = 1.92
ω1 12

10. Se unen dos masas iguales egún se indica mediante dos resortes de constan-
te k. Considerando solo el movimiento vertical, demostrar que las frecuen-

(3± 5)k
cias angulares de los dos modos normales vienen dads por ω 2 = 2m
y que,√ por tanto, el cociente entre las frecuencias de los modos normales
5+1
vale √5−1 . Hallar el cociente de amplitudes de las masas en cada modo
separado. (Nota: No es necesario considerar las fuerzas gravitatorias que
actúan sobre las masas, porque son independientes de los desplazamientos
y, por tanto, no contribuyen a las fuerzas restauradores que causan las
oscilaciones. Las fuerzas gravitatorias simplemente originan un desplaza-
miento de las posiciones de euqilibrio de las masa y no hay que hallar cuál
es el valor de dicho desplazamiento.)

Figura 53: Figura problema 5.10

Solución

Las ecuaciones de movimiemto son (xs y xi con para el cuerpo superior y

125
el inferior respectivamente):

d2 xs
m = −kxs − k(xs − xi )
dt2
d2 xi
m 2 = −k(xi − xs )
dt
con ecuaciones reducidas,

d2 xs
   
k k
2
+ xs + (xs − xi ) = 0
dt m m
d2 xi
 
k
2
+ (xi − xs ) = 0
dt m

agrupando,

d2 xs
   
k k
+2 xs − xi = 0
dt2 m m
d2 xi
   
k k
+ xi − xs = 0
dt2 m m

Con soluciones dadas por xj = Aj cos(ωt) 6= 0,


   
2 k k
− ω As + 2 As − Ai = 0
m m
   
k k
− ω 2 Ai + Ai − As = 0
m m

factorizando,
    
k k
−ω 2 + 2 As − Ai = 0
m m
    
k k
−ω 2 + Ai − As = 0
m m

Hallando los cocientes entre las amplitudes,


k
 2 
−ω + 2 m Ai
k
 =
m
A s
k

A
 m
k
 = i
−ω 2 + m As

igualando
k k
  
−ω 2 + 2 m m
k
 = 2 k

m −ω + m

126
Entonces,
       2
k k k
−ω 2 + 2 −ω 2 + =
m m m
expandiendo los factores,
     2  2
4 k 2 k 2 k k
ω − ω −2 ω +2 =
m m m m
   2
4 k 2 k
ω −3 ω + =0
m m
Solucionando la ecuación cuadrática,

k
 q k
2 k 2
 k
 q k 2 k 2

3 m ± −3 m − 4 m 3 m ± 9 m −4 m
ω2 = =
2 2
√ √
q
k
 k
 2
± 5 m k k
 
3 m ± 5 m
 
3 m 3± 5 k
= = =
2 2 2 m
Finalmente los modos normales de oscilación son
√ 
2 3± 5 k
ω =
2m
esto es, √  √ 
2 3+ 5 k 2 3− 5 k
ω+
= y ω− =
2m 2m
El cociente entre los dos modos es

2 (3+ 5)k √
ω+ 2m 3+ 5
2 = (3− 5)k =
√ √
ω− 3− 5
2m

11. El esquema (ver figura 54) muestra una masa M1 sobre un plano horizontal
sin rozamiento unida a un soporte O mediante un resorte de rigidez k. La
masa M2 está sujeta a la M1 mediante una cuerda de longitud l.

(a) Utilizando la aproximación de oscilaciones pequeñas,


x2 − x1
sin(θ) ' tan(θ) '
l
y partiendo de F = ma, deducir la ecuación del movimiento de M1 y M2 :
g M2 g
M1 ẍ1 = −kx1 + M2 (x2 − x1 ) M2 ẍ2 = − (x2 − x1 )
l l

(b) Para M1 = M2 = M , utilizar las ecuaciones para obtener las


frecuencias normales del sistema.

127
Figura 54: Figura problema 5.11

(c) ¿Cuáles son los movimientos de los modos normales para M1 =


M2 = M y gl  M k
?

Solución

(a) Las fuerzas en el sistema son (ver figura 55):

Figura 55: Figura problema 5.11a

Para el cuerpo 1,

d2 x1
N − M1 g − T2 cos(θ) = 0 y T2 sin(θ) − T1 = M1
dt2

128
Para el cuerpo 2 (donde s es el arco de giro),
d2 s d2 x2
T3 − M2 g cos(θ) = 0 y − M2 g sin(θ) = M2 = M 2
dt2 dt2
Para oscilaciones pequeñas,
x2 − x1
sin(θ) ' y cos(θ) = 1
l
Por tanto las ecuaciones quedan como (usando θ ' x2 , T1 = kx1 y T2 = T3
como ecuación conectora):
d2 x1
   
T3 x2 − x1 k
N = M1 g + T2 y − x1 =
M1 l M1 dt2
y
d2 x2
 
x2 − x1
T3 = M2 g y − g=
l dt2
por tanto,
d2 x1 d2 x2
     
k M2 x2 − x1 x2 − x1
=− x1 + g =− g
dt2 M1 M1 l dt2 l
Finalmente las ecuaciones quedan
d2 x1
 
k M2  g 
= − x 1 + (x2 − x1 )
dt2 M1 M1 l
d2 x2 g
= − (x2 − x1 )
dt2 l
(b) Para M1 = M2 = M ,
d2 x1
 
k g
+ x 1 − (x2 − x1 ) = 0
dt2 M l
d2 x2  g 
+ (x2 − x1 ) = 0
dt2 l
Con soluciones de la forma xj = Aj eiωt 6= 0, las ecuaciones quedan,
 
2 k g
− ω A1 + A1 − (A2 − A1 ) = 0
M l
g
− ω 2 A2 + (A2 − A1 ) = 0
l
entonces,

2 k g g
− ω A1 + A1 − A2 + A1 = 0
M l l
g g
− ω 2 A2 + A2 − A1 = 0
l l

129
Agrupando,
    
2 k g g
−ω + + A1 − A2 = 0
M l l
g h  g i
− A1 + −ω 2 + A2 = 0
l l
que lleva a la ecuacioón matricial,
k
+ gl − gl 
 2       
−ω + M A1 0
=
− gl −ω 2 + gl

A2 0

donde    
A1 0
6=
A2 0
Por tanto se calcula el determinante,
k
+ gl − gl 
 2    
−ω + M
det =0
− gl −ω 2 + gl


Resultando,
     h
2 k g  g i h  g i h  g i
−ω + + −ω 2 + − − − =0
M l l l l
   g   k   g   g 2  g 2
k g
ω4 − ω2 − ω2 − ω2 + + − =0
M l l M l l l
   g    
4 k 2 k g
ω − +2 ω + =0
M l M l
Solucionando la ecuación cuadrática,
 k g
 q k  g 2
 k
 g

2 M + 2 l ± M +2 l −4 M l
ω =
2
g
 q k 2 2
+ 4 gl + 4 g g
 k k
  k
 
M +2 l ± M M l −4 M l
=
2
g
 q k 2 2
+ 4 gl
 k
M +2 l ± M
=
2
Donde los modos de oscilación son
  " 2 # 21
k  g  k  g 2
2
ω+ = + + +
2M l 2M l
  " 2 # 21
k  g  k  g 2
2
ω− = + − +
2M l 2M l

130
(c) Las ecuaciones para gl  M
k
y por tanto para gl  2M k
se modifican
usando el binomio de Newton para los radicales se realiza la aproximación,
"
k 2
 # 12 k 2

2M 1 2M
1+ g 2
'1+ g 2
2
 
l l

k g g
  
reemplazando, y teniendo en cuenta que también 2M + l ' l :

k 2
g 
2 1 2M
ω+ = +1+ g 2
l 2

l
k 2
g 
2 1 2M
ω− = −1− g 2
l 2

l

g 3 g 2 k 2
  
2 2 l +2 l + 2M
ω+ = g 2

2 l
g 3 g 2 k 2
  
2 2 l −2 l − 2M
ω− = g 2

2 l

Finalmente factorizando se obtienen los modos,


g 2 g k 2
   
2 2 l 1+ l + 2M
ω+ = g 2

2 l
2 k 2
2 gl g
   
2 1− l − 2M
ω− = g 2

2 l

donde no hay claridad del movimiento que sigue el sistema acoplado.

Otra posibilidad de aproximación que no es correcta matemáticamente


es " 2   # 12
k g 2 g
+ '
2M l l
resultando los modos,
g g g
2
ω+ = + =2 Oscilación pendular
 gl   gl  l
2
ω− = − =0 Sin oscilación
l l

12. Se unen dos masa iguales a tres resortes idénticos (constante de resorte k)
sobre una superficie horizontal sin rozamiento (ver figura 56). Un extremo
del sistema está fijo; el otro se impulsa hacia adelante y hacia atrás con

131
Figura 56: Figura problema 5.12

un desplazamiento X = X0 cos(ωt). Hallar y dibujar esquematicamente


gráficos de los desplazamiento resultantes de ambas masas.

Solución

Las oscilaciones de este sistema son longitudinales y es un oscilador aco-


plado forzado entre dos cuerpos, donde hay un resorte fijo a un soporte, un
resorte de acople entre los cuerpos A y B de iguales masas con magnitud m
y un resorte que transmite el impulso de forma oscilatoria y que no entra
en el estudio de las osiclaciones del sistema debido a que no está unido a
otro cuerpo y por tanto no puede servir de acople y tampoco está unido a
un soporte por tanto no puede ejercer una fuerza de restitución; por estar
unido al cuerpo A, la fuerza impulsora está actuando directamente sobre
el cuerpo A solamente en t = 0 s. Las ecuaciones de dinámica son:
d2 xA
m = −k(xA − xB ) + X0 cos(ωt)
dt2
d2 xB
m 2 = −kxB − k(xB − xA )
dt
con ecuaciones reducidas,
d2 xA
   
k k X0
+ xA − xB = cos(ωt)
dt2 m m m
d2 xB
   
k k
− xA + 2 xB = 0
dt2 m m
que no se pueden desacoplar sumando y restándolas porque no son simétri-
cas. La ecuación matricial es
d2 xA
       X 
k 1 −1 xA m
0
cos(ωt)
+ =
dt2 xB m −1 2 xB 0
Se soluciona la homogénea, mediante la ecuación de valores propios:
 
1−λ −1
det =0
−1 2−λ

(1 − λ)(2 − λ) − 1 = 0 → λ2 − 3λ + 1 = 0

132
solucionando, √ √
3± 9−4 3± 5
λ= =
2 2
con soluciones,
√ √
3+ 5 3− 5
λ1 = y λ2 =
2 2
Reemplazando los valores propios λj para j = 1, 2 en:
    
1 − λj −1 xA 0
=
−1 2 − λj xB 0

3+ 5
las ecuaciones quedan para λ1 = 2 ,

1 − 12 (3 + 5)
    
−1 √ xA 0
=
−1 2 − 21 (3 + 5) xB 0

simplificando,
1 √    
2 (−1 − 5) −1√ xA 0
1 =
−1 2 (1 − 5) xB 0

resultando,
1 √
− (1 + 5)xA − xB = 0
2
1 √
− xA + (1 − 5)xB = 0
2
entonces
1 √
xB = − (1 + 5)xA
2

 
1 2
xA = (1 − 5)xB → xB = √ xA
2 1− 5
donde se observa que no se puede construir un vector propio para hallar
la solución complementaria. √
Para el siguiente valor propio: λ1 = 3−2 5 ,

1 − 21 (3 − 5)
    
−1 √ xA 0
1 =
−1 2 − 2 (3 − 5) x B 0

simplificando,
1 √    
2 (−1 + 5) −1√ xA 0
1 =
−1 2 (1 + 5) xB 0

133
por tanto,
1 √
(−1 + 5)xA − xB = 0
2
1 √
− xA + (1 + 5)xB = 0
2
entonces,
1 √
xB = (−1 + 5)xA
2

 
1 2
xA = (1 + 5)xB → xB = √ xA
2 1+ 5
donde se observa de nuevo que no se puede construir un vector propio para
hallar la solución complementaria.

Solución posible

Se puede replantear la gráfica como:


Por tanto las ecuaciones son:

Figura 57: Figura problema 5.12

d2 xA
m + kxA + k(xA − xB ) = X0 cos(ωt)
dt2
d2 xB
m 2 + kxB + k(xA − xB ) = 0
dt
Sumando y restando las ecuaciones se llega a:
d2 x+
 
k X0
2
+ x+ = cos(ωt)
dt m m
d2 x−
   
k k X0
2
+ x − + 2 x− = cos(ωt)
dt m m m
por tanto las dos ecuaciones resultantes son de la forma:
d2 x+
 
k X0
+ x+ = cos(ωt)
dt2 m m
d2 x−
 
k X0
+ 3 x− = cos(ωt)
dt2 m m

134
donde las ecuaciones son osciladores armónicos simples forzados, cuyas
soluciones son:
r !
k X0 cos(ωt)
x+ = A+ cos t+α + k

m m m − ω2
y r !
3k X0 cos(ωt)
x− = A− cos t+α +
m 3k

m m −ω
2

Usando el hecho de que x+ = xA + xB y que x− = xA − xB , entonces:


xA = 21 (x+ − x− ) y xB = 21 (x+ − x− ). Por tanto,
" r ! r !
1 k 3k
xA = A+ cos t + α + A− cos t+α
2 m m
#
X0 cos(ωt) X0 cos(ωt)
+ k
+
m 3k

m m − ω2 m −ω
2

y
" r ! r !
1 k 3k
xB = A+ cos t + α − A− cos t+α
2 m m
#
X0 cos(ωt) X0 cos(ωt)
+ k
−
m 3k

m m −ω 2 2
m −ω

Con gráficas dadas para los valores: A=1, B=2, k=40, m=2, ω=30, α=0
y f =2. Con la instrucción en Mathematica:

Figura 58: Instrucción en Mathematica para la figura del problema 5.12

En la gráfica 59 la lı́nea azul es xA y la amarilla es xB .Se puede analizar


que no existe un comportamiento ideal en esta gráfica acoplada; esto es, no
hay un valor máximo de un cuerpo y en el otro cuerpo valor nulo. Debido
a la clase de impulso impuesto en el sistema, las oscilaciones longitudinales
impulsoras no permiten que el resorte de acople transmita toda la energı́a
y luego la devuelva.

135
Figura 59: Figura del problema 5.12

13. Se sujeta por sus extremos a dos soportes fijos una cuerda de longitud 3l
y masa despreciable. La tensión en la cuerda es T .
(a) Se sujeta una partı́cula de masa m a una distancia l del extremo
de la cuerda, como está indicado. Escribir la ecuación para las oscilaciones
transversales pequeñas de m y hallar el perı́odo.
(b) Se une una partı́cula adicional de masa m a la cuerda como se
ve en la figura, dividiéndola en tres segmentos iguales cada uno de ellos
con tensión T . Dibujar el aspecto de la cuerda y la posición de las masas
en los dos modos normales separados de las oscilaciones transversales.
(c) Calcular ω para aquel modo normal que tenga mayor frecuen-
cia.

Solución

(a) En la figura 61 se pueden observar las fuerzas que actúan sobre el


cuerpo de masa m. Las condicones de equilibrio son:

− T cos(α) + T cos(β) = 0
d2 y
− T sin(α) + T sin(β) = m
dt2
Pero (ver figura 61)
y
sin(α) = p ,
y2 + l2

136
Figura 60: Figura problema 5.13

l
cos(α) = p ,
y + l2
2

y
sin(β) = p ,
y + (2l)2
2

Figura 61: Figura problema 5.13

y
2l
cos(β) = p
y2 + (2l)2
Entonces las ecuaciones de dinámica quedan:
l 2l
−Tp +Tp =0
y 2 + l2 y 2 + (2l)2
d2 y y y
m 2 = −T p −Tp
dt 2
y +l 2 y + (2l)2
2

Las cuales son válidas para cualquier ángulo. Para loscilaciones pequeñas
los ángulos pequeños y como la cuerda está fija en ambos extremos, las

137
oscilaciones longitudinales tienden a cancelarse. Sin embargo el ánalisis
se hará de las oscilaciones transversales pequeñas; esto es las oscilaciones
verticales, con aceleración dada por:

d2 y T y T y
2
+ p + p =0
dt m y +l
2 2 m y + (2l)2
2

para oscilaciones pequeñas l  y. Por tanto se factoriza l y 2l en los


radicales resultando
d2 y T y T y
2
+ q + q =0
dt ml y
 2 2ml y 2

1+ l 1+ 2l

y por tanto,

d2 y T
  y 2 − 12 T
  y 2 − 12
+ y 1+ + y 1+ =0
dt2 ml l 2ml 2l

usando el binomio de Newton se aproxima a

d2 y
   
T 1  y 2 T 1  y 2
+ y 1− + y 1− =0
dt2 ml 2 l 2ml 2 2l

d2 y y2 y2
   
T T
+ y 1− 2 + y 1− 2 =0
dt2 ml 2l 2ml 8l
d2 y 2y 2 y2
 
T
+ y 2 − + 1 − =0
dt2 2ml 2l2 8l2
d2 y 9y 2
 
T
+ 3 − y=0
dt2 2ml 8l2
por tanto,
d2 y
   
3T 9T
+ y− y3 = 0
dt2 2ml 16ml3
Debido al comportamiento lineal se tiene en cuenta el término con y;
3
adicionalmente yl3 → 0. Con lo anterior la frecuencia angular es:
    21
2 3T 2π 3T
ω = → ω= =
2ml τ 2ml

Finalmente el perı́odo es:


  12
2ml
τ = 2π
3T

138
(b) Las gráficas para el primer modo de oscilación (con mayor frecuencia)
son (ver figura 62):

Figura 62: Figura problema 5.13

Las gráficas para el primer modo de oscilación (con menor frecuencia) son
(ver figura 63):

Figura 63: Figura problema 5.13

139
(c) El sistema tiene ecuaciones dadas por

d2 x1
m + mω02 x1 + mω02 (x1 − x2 ) = 0
dt2
d2 x2
m 2 + mω02 x2 + mω02 (x2 − x1 ) = 0
dt

d2 x1
+ 2ω02 x1 − ω02 x2 = 0
dt2
d2 x2
+ 2ω02 x2 − ω02 x1 = 0
dt2
Sumando y restando,
d2 x+
+ ω02 x+ = 0
dt2

d2 x−
+ 3ω02 x− = 0
dt2
T
donde las dimensiones de ml en el SI son:
       
T N kg · m 1
= 2 = ω02
 
= = 2
ml kg · m s · kg · m s
Por tanto,
d2 x+
 
T
+ x+ = 0
dt2 ml
d2 x−
 
3T
+ x− = 0
dt2 ml
Finalmente los modos de oscilación son:
  12   21
3T T
ω1 = y ω2 =
ml ml
| {z }
Mayor modo

14. Para adquirir cierta familiaridad con la ecuación


 
pnπ
Apn = Cn sin
N +1

[Ec. (5-26) del texto], que describe las amplitudes de las partı́culas unidas
en los diversos modos normales, tomar el caso N = 3 y tabular, en un
esquema 3 × 3, los valores numéricos relativos de las amplitudes de las
partı́culas (p = 1, 2, 3) en cada uno de los modos normales (n = 1, 2, 3).

Solución

140
La ecuación para un sistema de tres partı́culas es:
 pnπ 
Apn = Cn sin
4
Para la partı́cula 1:  nπ 
A1n = Cn sin
4
con los modos de oscilación (n = 1, 2, 3), da:
π    
2π π 3π
A11 = Cn sin , A12 = Cn sin = Cn sin , A13 = Cn sin ,
4 4 2 4

resultando,
√ √
2 2
A11 = Cn , A12 = Cn , A13 = Cn
2 2
Para la partı́cula 2:
 
2nπ  nπ 
A2n = Cn sin = Cn sin
4 2

con los modos de oscilación (n = 1, 2, 3), da:


π    
2π 3π
A21 = Cn sin , A22 = Cn sin = Cn sin (π) , A23 = Cn sin ,
2 2 2

cuyos valores son:

A21 = Cn , A22 = 0, A23 = −Cn

Para la partı́cula 2:  
3nπ
A3n = Cn sin
4
con los modos de oscilación (n = 1, 2, 3), da:
       
3π 6π 3π 9π
A31 = Cn sin , A32 = Cn sin = Cn sin , A33 = Cn sin ,
4 4 2 4

Con valores,
√ √
2 2
A31 = Cn , A32 = −Cn , A33 = Cn
2 2
La tabla 3 × 3 es:

141
Apn n

=1 n=2 n

=3
2 2
p=1 2 Cn Cn 2 Cn
p=2 C
√ n
0 −C
√ n
2 2
p=3 2 Cn −Cn 2 Cn

15. Se sujeta a puntos fijos A y B, separados una distancia 4l, una cuerda
elástica de masa despreciable, estirada hasta tener una tensión T , la cual
lleva tres partı́culas de masa m igualmente espacidas, según se indica.
(a) Suponer que las partı́culas tienen desplazamientos transversales
pequeños y1 , y2 e y3 , respectivamente, en un instante dado.Escribir la
ecuación diferencial del movimiento de cada masa.
(b) El aspecto de los modos normales puede hallarse dibujando las
curvas sinusoidales que pasan por A y B. Dibujar estas curvas de modo
que sirvan para hallar los valores relativos y los signos de A1 , A2 y A3 en
cada uno de los posibles modos del sistema.
(c) Poniendo y1 = A1 sin(ωt), y2 = A2 sin(ωt) en las ecuaciones
(a), utilizar los cocientes A1 : A2 : A3 de la parte (b) para hallar las fre-
cuencias angulares de los modos separados.

Figura 64: Figura problema 5.15

Solución

(a,c) En este punto se van a desarrollar los puntos (a) y (c); esto es, plan-
tear las ecuaciones diferenciales y solucionarlas. Como se puede observar
es un sistema coplado de tres masas iguales con acoplamientos iguales. Las
ecuaciones son:
d2 y1
m = −mω02 y1 − mω02 (y1 − y2 )
dt2
d2 y2
m 2 = −mω02 (y2 − y1 ) − mω02 (y2 − y3 )
dt
d2 y3
m 2 = −mω02 y3 − mω02 (y3 − y2 )
dt

142
por tanto,

d2 y1
+ ω02 y1 + ω02 (x1 − x2 ) = 0
dt2
d2 y2
+ ω02 (y2 − y1 ) + ω02 (y2 − y3 ) = 0
dt2
d2 y3
+ ω02 y3 + ω02 (y3 − y2 ) = 0
dt2
factorizando las ecuaciones acopladas quedan como,

d2 y1
+ 2ω02 y1 − ω02 y2 = 0
dt2
d2 y2
+ 2ω02 y2 − ω02 (y1 + y3 ) = 0
dt2
d2 y3
+ 2ω02 y3 − ω02 y2 = 0
dt2
Con soluciones dadas por y1 = A1 sin(ωt), y2 = A2 sin(ωt) y y3 = A3 sin(ωt),
las ecuaciones caracterı́sticas son:

−ω 2 A1 + 2ω02 A1 − ω02 A2 = 0
−ω 2 A2 + 2ω02 A2 − ω02 (A1 + A3 ) = 0
−ω 2 A3 + 2ω02 A3 − ω02 A2 = 0

esto es,

(−ω 2 + 2ω02 )A1 − ω02 A2 = 0


(−ω 2 + 2ω02 )A2 − ω02 (A1 + A3 ) = 0
(−ω 2 + 2ω02 )A3 − ω02 A2 = 0

y por tanto,
 2
−ω + 2ω02 −ω02
   
0 A1 0
 −ω02 −ω 2 + 2ω02 −ω02  A2  = 0
0 −ω02 −ω 2 + 2ω02 A3 0

donde,    
A1 0
A2  6= 0
A3 0
Resultando la ecuación,
 2
−ω + 2ω02 −ω02

0
det  −ω02 −ω 2 + 2ω02 −ω02  = 0
0 −ω02 −ω 2 + 2ω02

143
resolviendo el determinante,

(−ω 2 + 2ω02 )[(−ω 2 + 2ω02 )2 − (−ω02 )(−ω02 )] − (−ω02 )(−ω02 )(−ω 2 + 2ω02 ) = 0

(−ω 2 + 2ω02 )3 − (−ω 2 + 2ω02 )ω04 − (−ω 2 + 2ω02 )ω04 = 0


(−ω 2 + 2ω02 )3 − 2(−ω 2 + 2ω02 )ω04 = 0
(−ω 2 + 2ω02 )(ω 4 − 4ω02 ω 2 + 2ω04 ) = 0
Resultando las ecuaciones

− ω 2 + 2ω02 = 0
ω 4 − 4ω02 ω 2 + 2ω04 = 0

que dan los modos de oscilación,


√ rad
ω1 = 2ω0 = 1.41ω0
s

q
rad
ω2 = 2 + 2ω0 = 1.85ω0
s

q
rad
ω3 = 2 − 2ω0 = 0.77ω0
s
Para observar las relaciones entre las amplitudes, se toman las ecuaciones
anteriores:

(−ω 2 + 2ω02 )A1 − ω02 A2 = 0


(−ω 2 + 2ω02 )A2 − ω02 (A1 + A3 ) = 0
(−ω 2 + 2ω02 )A3 − ω02 A2 = 0

Primero −ω 2 +2ω02 )A1 −ω02 A2 = 0, que queda como (−ω 2 +2ω02 )A1 = ω02 A2
y entonces la relación entre amplitudes es:

A1 ω02
=
A2 −ω + 2ω02
2

Para la tercera ecuación (−ω 2 + 2ω02 )A3 − ω02 A2 = 0, la relación entre


amplitudes queda:
A3 ω02 A1
= 2 2 =
A2 −ω + 2ω0 A2
de donde resulta que A1 = A3 .
Finalmente la segunda ecuación, (−ω 2 + 2ω02 )A2 − ω02 (A1 + A3 ) = 0 se
divide entre A2 ,  
2 2 2 A1 A3
(−ω + 2ω0 ) = ω0 +
A2 A2
de donde se soluciona para los modos de oscilación.
(b) La gráfica preliminar es:

144
Figura 65: Figura problema 5.15

16. Considerando un sistema de N osciladores acoplados asociados a una fre-


cuencia ω < 2ω0 (es decir, y0 = 0, yN +1 = h cos(ωt)). Hallar las am-
plitudes resultantes de los N osciladores. [Indicación: las ecuaciones di-
ferenciales del movimiento son las mismas que en el caso sin impulsar
(solo con diferentes condiciones lı́mites). De aquı́ que pueda ensayarse
Ap = C sin(αp), y determinar ası́ los valores necesarios de α y C. (Nota:
si ω < 2ω0 , α es complejo y las ondas se amortiguan exponencialmente en
el espacio.)]

Solución

Se pide que y0 = 0 y que yN +1 = h cos(ωt) que implica que sea un oscilador


armónico simple de N cuerpos acoplados forzado. Para N + 1,
 

   

 nπ 
yN +1,n (t) = Cn sin [(N + 1)α] cos 2ω0 sin t = h cos(ωt)
| {z } 
 2(N + 1) 
AN +1
| {z } 
ω
h i
y debido a que ω < 2ω0 , entonces: 0 < sin 2(Nnπ+1) < 1, y esto no tiene
que ver algo con las propiedades matemáticas de α como lo sugiere el enun-
ciado. Además α está en el argumento de Ap ; esto es, Ap = C sin(αp), por
tanto no puede ser complejo porque las funciones trigonométricas seno y
coseno no están definidas para calcularlas con complejos. Adicionalmente
el movimiento impulsado no tiene que ver con la relación entre ω y ω0 .
Como se observa hay muchas inconsistencias en el enunciado.

Para
n que seh cumplai yoN +1 = h cos(ωt), entonces se debe verificar que
cos 2ω0 sin 2(Nnπ+1) t = 1. Esto es,
 

2ω0 sin t = 2sπ donde, s = 1, 2, 3, . . .
2(N + 1)

145
entonces,
2sπ
t= h i
2ω0 sin 2(Nnπ+1)

por tanto para ese valor temporal yN +1 = h cos(ωt). Esto es AN +1 =


h cos(ωt), quedando:
 
 ω  2sπ 
AN +1 = h cos h i
 2ω0 sin nπ 
2(N +1)

ω
Debido a que ω < 2ω0 entonces 2ω 0
tiene una magnitud pequeña. En
general para cualquier partı́cula las amplitudes son:
 
 ω  2sπ 
Ap,n = h cos h i
 2ω0 sin nπ 
2(N +1)

Finalmente
 
 ω

2sπ   

 
yp,n (t) = h cos h i cos 2ω0 sin t
 2ω0 sin nπ  2(N + 1)
2(N +1)

17. Se demuestra en el texto que la frecuencia del modo normal elevada de


una lı́nea de masas puede hallarse considerando una partı́cula cerca de la
mitad de la lı́nea, rodeada por partı́culas que tienen deslazamientos casi
iguales y de sentido opuesto que el de la propia partı́cula. Demostrar que
puede calcularse la misma frecuencia considerando la primera partı́cula
de la lı́nea, sometida a la tensión en los primeros segmentos de la cuerda
que lo unen al segmento fijo y a la partı́cula 2 (véase fig 5-19 y el estudio
relacionado con ella.)

Solución

La figura 5-19 del texto es:

Figura 66: Figura problema 5.17

Entonces la dinámica para cada partı́cula de masa m es:

146
Figura 67: Figura problema 5.17

Las ecuaciones son:


d2 x
m = −T cos(α) + T cos(α) = 0
dt2
d2 y
m 2 = −T sin(α) − T sin(α) = −2T sin(α)
dt
donde se observa que la fuerzas en el eje horizontal se anulan. Teniendo
en cuenta que: sin(α) = √ 2y
2 2
, la ecuación queda
y +l

d2 y 2y 2y
m = −2T p = −2T q 
dt2 y 2 + l2 y 2
l l +1
y
entonces, debido a que l →0

d2 y
 
T
= −4 y = − 4ω02 y
dt2 ml |{z}
2
ωmax

entonces la frecuencia angular máxima es ωmax = 2ω0 (ver texto).

Para la primera partı́cula una gráfica de las fuerzas serı́a (ver figura 68).
Las ecuaciones para p = 1 son:
d2 x1
m = −T cos(α) + T cos(β)
dt2
2
d y1
m 2 = −T sin(α) − T sin(β)
dt

147
Figura 68: Figura problema 5.17

pero
2yα
sin(α) = p
yα2 + lα
2

2yβ
sin(β) = q
yβ2 + lβ2

cos(α) = p
yα2 + lα
2


cos(β) = q
yβ2 + lβ2

Por tanto las ecuaciones de dinámica quedan:

d2 x1 lα lβ
m = −T q +Tq
dt2 2
y +l 2 y + l2
2
α,1 α β,1 β
2
d y1 2yα,1 2yβ,1
m = −T q −Tq
dt2 2
y +l 2 y 2 + l2
α,1 α β,1 β

factorizando
d2 x1 lα lβ
m = −T +T
dt2
r 2 r 2
yα,1 yβ,1
lα lα +1 lβ lβ +1

d 2 y1 2y 2y
m = −T r α,1 −T r β,1
dt2 yα,1
2 2
yβ,1
lα lα +1 lβ lβ +1

148
simplificando,

d2 x1 1 1
m = −T r 2 + T r
dt2 y

yβ,1
2 
1 + lα,1
α
1 + lβ
2
   
d y1 2T yα,1 2T yβ,1
m 2 =− r 2 − lβ r
dt lα 
y

yβ,1
2
1 + lα,1α
1 + lβ

El problema cuando es fijo en ambos extremos el desplazamiento vertical


es pequeño respecto al horizontal y es más pequeño respecto a la distancia
y
entre las partı́culas. Esto hace que las relaciones ylαα y lββ , a pesar que sean
para ángulos diferentes (α y β) tienen a cero, o al menos a un número con
una muy baja magnitud, que está elevado al cuadrado que tiende a cero
también. Esto hace que:

d2 x1 1 1
m = −T v 2 + T v
dt2 u 
yα,1
u 
yβ,1
2
u1 + u1 +
u u
t lα t lβ
| {z } | {z }
→0 →0

entonces
d2 x1
m = −T + T = 0
dt2
usando lo mismo para las ecuaciones transversales,

d2 y1
   
2T 2T
m 2 =− yα,1 − yβ,1
dt lα lβ
 
yα,1 yβ,1
= −2T +
lα lβ
y y
Las relaciones lα,1
α
y lβ,1
β
tienen valores pequeños individualmente, pero
debido a las condiciones del sistema en el cual las fuerzas longitudinales
se anulan, se verifica que
yα,1 yβ,1 y1
→ =
lα lβ l

Por tanto,

d2 y1 y
1 y1  d2 y1 4T
m = −2T + ∴ = − y1
dt2 l l dt2 ml
que coincide con ωmax = 2ω0 .

149
6. Modos normales de sistemas continuos. Análi-
sis de Fourier
1. Una cuerda uniforme de 2.5 m de longitud y 0.01 Kg de masa se somete
a una tensión de 10 N .
(a) ¿Cuál es la frecuencia de su modo fundamental?
(b) Si se pulsa transversalmente la cuerda y luego se toca en un
punto a 0.5 m de su extremo, ¿qué frecuencias persistirán?

Solución

2L 2(2.5)
(a) La longitud de onda es, λn = n = que para el modo fun-
n q
damental es: λ1 = 5 m. La velocidad de propagación es: v = Tµ , donde
kg
la densidad lineal de masa es, µ = 0.01
2.5 = 0.004 m , y por tanto la veloci-
q
10
dad esv = 0.004 = 50 m
s . Usando la ecuación de dispersión, v = λν, se
halla la frecuencia como ν1 = λv = 50 5 = 10 Hz.
(b) Al tocarlo a 0.5 m del extremo que es la quinta parte de la cuerda,
se formarán nodos cada 0.5 m. Esto darı́a el quinto modo de oscilación.
Por tanto las frecuencias que persistirán teniendo en cuenta que se de-
be verificar la ecuación νn = nν1 , que dice que todas las frecuencias son
mútiplos enteros del modo fundamental, son: ν5 = 5ν1 = 5(10) = 50 Hz y
los múltiplos de 5. Finalmente las frecuencias que persisten son: 50, 100,
150, 200, etc. [Hz].
2. Una cuerda de longitud L y de masa total M se estira mediante una ten-
sión T . ¿Cuáles son las frecuencias de los tres modos normales inferiores
de oscilación de la cuerda, cuando estas son transversales? Comparar esta
frecuencias con las tres frecuencias de modos normales de 3 masas cada
una de ellas de valor M/3 separadas a intervalos iguales sobre una cuerda
sin masa sometida a una tensión T y cuya longitus total es L.

Figura 69: Figura problema 6.2

Solución

Las frecuencias de los modos normales en una cuerda de longitud L y

150
de masa total M son,
 1  1   12   12
nπ T 2 n T 2 n T T
ωn = donde νn = = =n
L µ 2L µ 2 ML 4M L
| {z }
ν1

T
 21
Por tanto las frecuencias son νn = nν1 . Esto es ν1 = 4M L , ν2 =
T
 12 T
 12
2 4M L y ν 3 = 3 4M L .

Para un sistema acoplado de tres masas se usa la ecuación usada para


calcular N cuerpos acoplados:
 

ωn = 2ω0 sin = 2πνn
2(N + 1)
que para tres partı́culas la frecuencia lineal es:
 1
ω0  nπ  1 k 2  nπ 
νn = sin = sin
π 8 π m 8
Entonces la tensión es T = kx = k L4 y la masa es m = M
3 ; ası́
  12 ! 12 1
4T 
k L 12T 2
ω0 = = M =
m 3
ML
Por tanto la frecuencia es:
  12
1 12T  nπ 
νn = sin
π ML 8
que para los tres modos de oscilación inferiores es:
 1
1 12T 2 π
ν1 = sin
π ML 8
  12    1
1 12T 2π 1 12T 2 π
ν2 = sin = sin
π ML 8 π ML 4
  12  
1 12T 3π
ν3 = sin
π ML 8
T
 12
pero usando el modo fundamental de la cuerda: ν1,c = 4M L

1 48T 2
1  π  √48  T  12 π
ν1 = sin = sin
π 4M L 8 π 4M L 8
  12 √   1
1 48T π 48 T 2 π
ν2 = sin = sin
π 4M L 4 π 4M L 4
  12   √   12  
1 48T 3π 48 T 3π
ν3 = sin = sin
π 4M L 8 π 4M L 8

151
se reescriben las ecuaciones resultando los valores de las frecuencias com-
paradas con la de la cuerda:

48 π
ν1 = sin ν1,c = 0.844ν1,c
√π 8
48 π
ν2 = sin ν1,c = 1.559ν1,c
√π 4
 
48 3π
ν3 = sin ν1,c = 2.037ν1,c
π 8

3. La deducción de las vibraciones libres de una cuerda tensa que se da en el


texto ignora la gravedad. ¿Está justificada esta omisión? ¿Cómo se harı́a
el análisis si hubiera que incluir los efectos gravitatorios?

Solución

Los efectos de la aceleración gravitatoria son equilibrados por la tensión


en la cuerda. Cuando esta está en equilibrio estático y está actuando sobre
ella una tensión T que tiene una magnitud especı́fica tanto para mantener
tensa la cuerda, como para equilibrar los efectos de la aceleración gravi-
tatoria sobre la masa de la cuerda.

Figura 70: Figura problema 6.3

Si en un momento especı́fico se desearı́a usar el peso de la cuerda, el gráfi-


co de las fuerzas serı́a como el mostrado en la figura 70, para una cuerda
con densidad completamente uniforme. Las ecuaciones de dinámica se es-
cribirı́an no exactamente para los ejes x y y, debido a que existe un arco
con águlo pequeño para cada eje, a pesar que el punto especı́fico tenga la

152
coordenada x y x + dx como se ve en la figura 70, se tomarán los valores
arco en x: Sx y arco en y: Sy resultando las ecuaciones,
d2 Sx
m = T cos(θ + dθ) − T cos(θ)
dt2
2
d Sy
m 2 = T sin(θ + dθ) − T sin(θ) − mg
dt
pero ahora las tensiones sobre el eje x no se cancelan debido a que aún
la cuerda puede tensarse más para que los ángulos de oscilación sean tan
pequeños que las fuerzas longitudinaes desparezcan. Esto hace que la de-
finición de las funciones sinusoidales con catetos e hipotenusa no sea ade-
cuada. Por tanto se escribirán las funciones sinusoidales como series,
d2 Sx (θ + dθ)2 (θ + dθ)4 θ2 θ4
   
T T
= 1 − + − · · · − 1 − + − · · ·
dt2 m 2 24 m 2 24
2 3 5
θ3 θ5
   
d Sy T (θ + dθ) (θ + dθ) T
= (θ + dθ) − + − ··· − θ− + − ··· − g
dt2 m 6 120 m 6 120
Al tener en cuenta solo los términos cúbicos, debido a que los ángulos que
forma la cuerda con la horizontal hace que los términos mayores se anulen,
las ecuaciones son:
d2 Sx (θ + dθ)2 θ2
   
T T
= 1 − − 1 −
dt2 m 2 m 2
2 3
θ3
   
d Sy T (θ + dθ) T
= (θ + dθ) − − θ − −g
dt2 m 6 m 6
simplificando,
d2 Sx (θ + dθ)2 θ2
 
T
= − +
dt2 m 2 2
2 3
θ3
 
d Sy T (θ + dθ)
= dθ − + −g
dt2 m 6 6
expandiendo (teniendo en cuenta solo los diferenciales lineales debido a
que cada ecuación es unidimensional),
d2 Sx
 2
θ + 2θdθ θ2

T T
2
= − + = [−θdθ]
dt m 2 2 m
d2 Sy θ3 + 3θ2 dθ θ3 θ2 dθ
   
T T
= dθ − + − g = dθ − −g
dt2 m 6 6 m 2
resultando finalmente,
d2 Sx
 
T
= − θdθ
dt2 m
d2 Sy θ2
  
T
= 1 − dθ − g
dt2 m 2

153
suponiendo que existe una longitud igual de la cuerda en cada ecuación (lo
cual no necesariamente es cierto en este caso) solo para que las frecuen-
cias angulares sean iguales para las dos ecuaciones, se divide miembro a
miembro entre l,

1 d2 Sx
 
T
2
=− θdθ
l dt ml
1 d2 Sy θ2
   
T g
2
= 1− dθ −
l dt ml 2 l
T
llamando ω02 = ml ,

1 d2 Sx
= −ω02 θdθ
l dt2
1 d2 Sy θ2
 
2 g
= ω0 1 − dθ −
l dt2 2 l

se observa que en la ecuación pseudo-vertical queda el término gl que es la


frecuencia fundamental de un péndulo simple. Esto hace que el compor-
tamiento del sistema no sea solo de una onda transversal, sino también
longitudinal, caótico y posiblemente no periódico. Pero las pendientes de-
∂y
finidas por ∂x = tan(θ), varı́an constantemente y por tanto su variación
∂2y
es, ∂x2 = sec2 (θ)dθ y por tanto

∂2y 1 1
= sec2 (θ)dθ = dθ = 2 2

∂x2 cos2 (θ) 1 − θ2
entonces 2
θ2 ∂2y

dθ = 1−
2 ∂x2
no hay necesidad de reemplazar, porque las ecuaciones quedan dependien-
tes de θ, y por tanto son muy dificlmente solubles, si lo son. La conclu-
sión es que si se desea un movimiento ondulatorio, la cuerda debe estar
lo suficientemente tensa para que los ángulos sean tan pequeños que las
componentes longitudinales de las tensiones se anulen.
4. Demostrar que el análisis del texto para vibraciones libres de una cuerda
horizontal es también válido para una cuerda vertical si T  mg.

Solución

Para la demostración se analizará una cuerda fija en ambos extremos de


forma vertical (ver figura 71) con masa total M y longitud L. Se puede
analizar que el vector aceleración gravitatoria es longitudinal a la cuerda
y si solamente se deja colgando la cuerda hasta qu eesta esté competa-
mente inmóvil, habrá un equilibrio estático y por tanto el peso de esta

154
Figura 71: Figura problema 6.4

será compensado por el soporte superior. La cuerda al fijarla al soporte


inferior queda sin tensión adicional a la del peso de ella misma. Para una
propagación de onda correcta la velocidad de fase debe ser
s r
T TL
vp = =
µ M

si la tensión es solo el peso, esto es T = M g,


r
M gL p
vp = = gL
M
m
que con g = 9.8 s2 , √
vp = 3.13 L
Por tanto esto significa que la velocidad de fase depende directamente de
la longitud de la cuerda, y para adquirir una tensión aceptable se tendrı́a
que alargar la cuerda para que pese más, pero esto no es posible porque
la cuerda tiene una longitud fija y aumentar la masa no servirı́a porque
es inversamente proporcional a la velocidad. Entonces la única solución es
tensar la cuerda más que el peso de esta. Esto es al menos T > mg. Si
se observa la gráfica 71, se puede ver que los ángulos tienden a π2 y esto
hace que las componentes verticales (longitudinales) tiendan a anularse.

155
Las ecuaciones serı́an:
d2 x
m = −T cos(θ) + T cos(θ + dθ)
dt2
d2 y
m 2 = T sin(θ) − T sin(θ + dθ) − mg
dt
π
pero ahora θ → 2. Expandiendo solo para observar una aproximación
usando lı́mites,
d2 x
m = −T cos(θ) + T cos(θ) cos(dθ) − T sin(θ) sin(dθ)
dt2
d2 y
m 2 = T sin(θ) − T sin(θ) cos(dθ) − T cos(θ) sin(dθ) − mg
dt
entonces
d2 x
 
T
=− dθ
dt2 m
d2 y
= −g
dt2
Como se observa en esta rápida aproximación, la primera ecuación con-
duce a la ecuación de onda unidimensional, pero queda una aceleración
gravitatoria residual en la vertical que debe ser compensada para anularse
y que el comportamiento sea unidimensional, como se busca. Por tanto
con esto y lo visto anteriormente la tensión debe ser mucho mayor que el
peso de la cuerda.
5. Una cuerda estirada de mas m, longitud L y tensión T se ve impulsada
por dos fuentes, una en cada extremo. Ambas fuentes tienen la misma fre-
cuencia ν y amplitud A, pero están desfasadas exactamente 180o entre sı́.
¿Cuál es el valor más pequeño posible de ω consistente con las vibraciones
estacionarias de la cuerda?

Solución

Las condiciones del enunciado confirman que la ecuación de onda uni-


dimensional se usa para este sistema. Esto es
∂ 2 y(x, t) 1 ∂ 2 y(x, t)
=
∂x2 v 2 ∂t2
La solución de esta es de la forma y(x, t) = X(x)T (t). Por tanto
1 d2 X(x) 1 d2 T (t)
= = −a2
X(x) dx2 T (t)v 2 dt2
Resultando las ecuaciones,
d2 T (t)
1) + a2 v 2 T (t) = 0
dt2

156
y
d2 X(x)
2) + a2 X(x) = 0
dx2
Donde es claro que en la ecuación temporal ω 2 = a2 v 2 donde a es el
número de onda k (construyendo la ecuación de dispersión v = ωk ), y esto
lleva a la solución T (t) = cos(ωt) (ver el texto). En la segunda ecuación,
la función X(x) tiene que ver con las condiciones frontera, que son las de
interés en este problema. La solución de la segunda ecuación es:
h ω  i
X(x) = X0 cos(kx) = X0 cos x
v
Usando la ecuación de onda e incluyendo las condiciones de frontera re-
sulta:
y(0, t) = A cos(ωt)
y
y(L, t) = A cos(ωt + π) = −A cos(ωt)
Por tanto para la parte espacial,

X(0) = X0 = A

y h ω  i h ω  i
X(L) = X0 cos L = −A ∴ A cos L = −A
v v
resultando h ω  i
cos L = −1
v
La parte espacial es h ω  i
X(x) = A cos x
v
y ası́ la función de onda da
h ω  i
y(x, t) = A cos x cos(ωt)
v
que en x = 0,
y(0, t) = A cos(ωt)
y en x = L,
h ω  i
y(L, t) = A cos L cos(ωt) = −A cos(ωt)
| {zv }
−1

Cumpliendo con las condciones de frontera.

Para halla el valor más pequeño de omega consistente con las vibraciones
estacionarias de la cuerda, se usa
h ω  i ω
cos L = −1 ∴ L = (2s − 1)π donde s = 1, 2, 3, . . .
v v

157
por tanto el valor que toma ω es

(2s − 1)v
ω= π donde s = 1, 2, 3, . . .
L
y su valor más pequeño es para n = 1:
  12
T   12   12
v µ T T
ω=π =π =π =π
L L µL2 ML

6. Se sujeta una varilla uniforme en el centro, dejando ambos extremos libres.


(a) ¿Cuáles son las frecuencias naturales de la varilla en la vibración
longitudinal?
(b) ¿Cuál es la longitud de onda en el modo enésimo?
(c) ¿Dńde están los nodos para el modo enésimo?

Solución

(a) Debido a que es una vibración longitudinal y tiene los extremos li-
bres, entonces en estos las fuerzas son nulas. Para definir las frecuencias
naturales se toma un volumen finito entre los extremos y entonces en este
volumen habrá una distribución de moléculas propias de la varilla de las
cuales depende la frecuencia longitudinal. Por tanto la fuerza neta sobre
ese volumen depende de las caracterı́sticas de rigidez de la varilla o del
módulo de Young
σN ∆l
Y = ∆l → σN = Y
l
l0
0

donde ∆ll0 es la deformación unitaria que para la varilla es la deformación


longitudinal que sufre en forma de compresión-distensión; esto implica que
∆ξ
la relación ∆x varı́a respecto a la longitud. En su forma diferencial es,

∆l ∂2ξ
→ ∆x (cantidades adimensionales)
l0 ∂x2
también el esfuerzo normal σN que relaciona la fuerza (F ) ejercida por
unidad de área (σ), donde esta fuerza es la fuerza neta sobre el elemento
de volumen. Finalmente,
F ∂2ξ
=Y ∆x
α ∂x2
que para un sistema de masa constante, F = ma, que escrita en función
de la densidad volumétrica de masa da
2
ρα∆x ∂∂t2ξ ∂2ξ
=Y ∆x
α ∂x2

158
∂2ξ ∂2ξ
ρ 2
=Y
∂t ∂x2
donde se obtiene la ecuación monodimensional de onda para la varilla
rı́gida:
∂ 2 ξ(x, t) ρ ∂ 2 ξ(x, t)
=
∂x2 Y ∂t2
cuya solución es
ξ(x, t) = X(x)T (t)
entonces,
1 d2 X(x) ρ d2 T (t)
= = −a2
X(x) dx2 Y T (t) dt2
resultando las ecuaciones
d2 X(x) d2 T (t)
= −a2 X(x) y = −a2 vp2 T (t)
dx2 dt2
donde la velocidad de fase es vp2 = Yρ . La ecuación temporal da la solución
de un oscilador armónico simple, donde a2 vp2 = ω 2 y de esta forma se
analiza que a = k. La ecuación espacial se usa para imponer las condiciones
de frontera. Por tanto,
d2 X(x)
+ k 2 X(x) = 0
dx2
como los extremos de la varilla están libres esta está sujeta en la mitad,
por simetrı́a los extremos deben tener valores iguales a la amplitud. Con
este análisis la solución de esta ecuación es:
X(x) = X0 sin(kx)
Reemplazando esta condición en X L2 resultando,


      
L L ω L
X = X0 sin k = X0 sin = ±X0
2 2 vp 2
pero la función seno siempre es máxima cuando
 
ω L π
= (2n − 1)
vp 2 2
resultando la frecuencia de la varilla
      12
2 π 2n − 1 Y
ωn = (2n − 1) vp = π
L 2 L ρ
(b) Con ωn = 2πνn , la frecuencia lineal es
    12
2n − 1 Y
νn =
2L ρ
| {z } | {z }
λ−1
n
vp

159
por tanto la longitud de onda para el modo enésimo es:
1 2L L
λn = 2n−1
= = 1
2L
2n − 1 n− 2

(c) Los nodos para el modo enésimo son los valores nulos para X en el
modo enésimo. En la figura 72 se puede observar que el nodo comienza en
el punto donde stá sujeta la varilla; esto es x = L2 .

Figura 72: Figura problema 6.6

Entonces los nodos siguientes son simétricos a partir de este punto, y


dependen de λn comenzando desde x = L2 , cuya posición inicial es x0 =
L
 1
2 n− 1 . Los nodos siguientes se hallan simetricamente con
2

x = x0 ± sx0

donde s es un entero; por tanto los nodos son


 
L
x0 =
2
   
L L 1
x1 = ±
2 2 n − 12
   
L L 1
x2 = ±2
2 2 n − 21
   
L L 1
x3 = ±3
2 2 n − 21
..
.
   
L L 1
xs = ±s
2 2 n − 12

factorizando el término s-ésimo:


n − 12 ± s
     
L s L
xs = 1± =
2 n − 12 2 n − 21

donde finalmente los nodos están en:


L (n − 21 ± s)
 
x= s = 0, 1, 2, . . .
n − 12

2

160
7. Deducir la ecuación de onda para las vibraciones de una columna de aire.
El resultado final deberá ser
∂2ξ ρ ∂2ξ
=
∂x2 K ∂t2
en donde ξ es el desplazamiento a partir de la posición de equilibrio, ρ es
la densidad máxima y K es el módulo elástico.

Solución

La caracterı́stica de una varilla es su rigidez y se estudia con el módulo


de Young Y . Cuando se tiene un gas encerrado en un recipiente cilı́ndrico
y cerrado con un pistón en el extremo superior. El gas por el peso del
pistón se comprime y al comprimirse aumneta la presión del gas en el re-
cipiente que hace que este se expanda y suba el pistón; pero al subir el gas
se expande y disminuye la presión del gas que hace que el pistón vuelva
a bajar completándose un ciclo. Se observa aquı́ que este movimiento es
periódico. Por tanto hay una relación entre el cambio de presión ∆P (que
es el análogo a la deformación normal en una varilla) y la variación uni-
taria del volumen del gas (que es el análogo a la deformación unitaria de
la longitud en la varilla). El cociente entre los dos se denomina módulode
compresibilidad K. Por tanto se tiene:
∆P ∆V
K= ∆V
→ ∆P = K
V0
V0

entonces
∆F α∆ξ ∆ξ ∂2ξ
=K → ∆F = Kα = ∆m 2
α αξ ξ ∂t
∆m
pero ρ = ∆V ; entonces,

∆y ∂2ξ ∂2ξ
Kα = ρ∆V 2 = ρα∆y 2
y ∂t ∂t
∆y
donde y es la deformación unitaria que para el gas es lacompresión-
∆ξ
distensión que sufre; esto implica que la relación ∆x varı́a respecto a la
dimensión longitudinal del cilindro. En su forma diferencial es,

∆y ∂2ξ
→ ∆y (cantidades adimensionales)
y ∂x2
por tanto,
∂2ξ ∂2ξ
Kα 2
∆y = ρα∆y 2
∂x ∂t

161
y debido a que este problema se analiza para las mismas variaciones de
volumen, la ecuación queda como:
∂2ξ ∂2ξ
K = ρ
∂x2 ∂t2
Finalmente se obtiene la ecuación de onda para las vibraciones de una
columna de aire:
∂2ξ ρ ∂2ξ
2
=
∂x K ∂t2
8. Demostrar que para la vibración de una columna de aire:
(a) Un extremo abierto representa una condición de variaciones de
presión nula durante la oscilación y de aquı́ un lugar en el que se presenta
un movimiento máximo del aire (un antinodo).
(b) Un extremo cerrado es un lugar de movimiento cero (un nodo)
y de aquı́ uno de variación de presión máxima.

Solución

En general para la ecuación de onda de una columna de aire


∂2ξ ρ ∂2ξ
=
∂x2 K ∂t2
la solución es ξ(x, t) = X(x)T (t), donde al reemplazarla queda:

1 d2 X(x) 1  ρ  d2 T (t)
2
= = −a2
X(x) dx T (t) K dt2
La ecuación de la parte espacial es,
d2 X(x)
+ a2 X(x) = 0
dx2
con solución,
X(x) = X0 sin(ax)
La ecuación de la parte temporal es,
d2 T (t)
 
2 K
2
+a T (t) = 0
dt ρ
cuya solución es   1 
ρ 2
T (t) = T0 cos a t+φ
K
1
ρ 2 ρ
 21
donde ω = a K y como la velocidad de fase es, v = K , entonces a
es el número de onda k. Ası́,

T (t) = T0 cos (ωt)

162
Entonces la función de onda es,

ξn (x, t) = ξ0,n sin(kn x + φ) cos (ωn t)

(a) Para el extremo abierto, el comportamiento es igual al de una varilla


v
sostenida en la mitad ξ L2 , t = 0. Su longitud de onda es ν1 = 2L donde
2π π
su longitud de onda es λ1 = 2L y por tanto k1 = λ1 = L . Entonces usando
π

X(x) = X0 cos L x ,
    
L π L
X = X0 cos =0
2 L 2
Finalmente, h π  i
X(0) = X0 cos 0 = X0
L
y h π  i
X(L) = X0 cos L = −X0
L
entonces en general el valor es de X(L) = X(0) = ±X0 o con el valor de
la amplitud.

(b) El movimiento cero se escribe como una oscilación con la frecuencia


v
ν1 = 4L , y por tanto k1 = 2π π
λ1 = 2L ; entonces
h π  i
X (x) = X0 cos x =0
2L
y su solución es,
h π  i  π  π
cos x =0 → x= → x=L
2L 2L 2
entonces se tiene que X(L) = 0.
9. Una habitación tiene dos paredes opuestas que están alicatadas (revestidas
de baldosas). Las paredes restantes, el suelo y el techo están recubiertas
por un material absorbente del sonido. La frecuencia más baja para la
cual es acusticamente resonante la habitación es 50 Hz.
(a) Se produce un ruido complejo en la habitación que excita solo
a los dos modos inferiores, de tal modo que cada modo tiene su máxima
amplitud para t = 0. Hacer un esquema del aspecto del desplazamiento
1
para cada modo separado en función de x para t = 0 s, t = 200 s y
1
t = 100 s.
(b) Se observa que el desplazamiento máximo de las partı́culas de
polvo en el aire (que no se produce necesariamente en el mismo instante
en cada posición) en diversos puntos entre las paredes es el siguiente:
L L 3L
x 4 2 4
ξmáx +10µ +10µ −10µ

163
¿Cuáles son las amplitudes de cada uno de los dos modos separados?

Solución

rad
(a) con ν = 50 Hz, entonces ω = 100π s ; las fases para los difern-
tes tiempos son:

φ1 = ωt1 = 100π(0) = 0 rad


 
1 π
φ2 = ωt2 = 100π = rad
200 2
 
1
φ3 = ωt3 = 100π = π rad
100

Las funciones de onda temporales son,

y1 = A cos(0) = A
π
y2 = A cos =0
2
y3 = A cos (π) = −A

Por tanto las funciones de onda ξ(x, t) son:

ξ1,n (x, 0) = A sin(kn x)


 
1
ξ2,n x, =0
200
 
1
ξ3,n x, = −A sin(kn x)
100

Como tiene su máxima amplitud en t = 0 s, entonces

ξ1,n (x, 0) = A sin(kn x) = A

entonces
 
π 1
sin(kn x) = 1 → kn x = (4p + 1) = 2p + π (p = 0, 1, 2, 3, . . .)
2 2

entonces los valores de x para las amplitudes máximas son


 
λn 1
x= 2p + π (p = 0, 1, 2, 3, . . .)
2π 2
2L
con λn = n , finalmente se llega a
 
L 1
x= 2p + (p = 0, 1, 2, 3, . . .)
n 2

164
que para los dos modos inferiores de oscilación dan:
 
1 L 5L 9L
x(n = 1) = L 2p + = , , ,...
2 2 2 2
 
L 1 L 5L 9L
x(n = 2) = 2p + = , , ,...
2 2 4 4 4

(b) Para n = 1 tomando L2


    
L π L π
ξ1 , t = 10 sin cos (100πt) = 10 sin cos (100πt)
2 L 2 2
= 10 cos (100πt)

Para n = 1 tomando L4
    
L π L π
ξ1 , t = 10 sin cos (100πt) = 10 sin cos (100πt)
4 L 4 4

10 2
= cos (100πt)
2
Para n = 1 tomando 3L 4
      
3L π 3L 3π
ξ1 , t = 10 sin cos (100πt) = 10 sin cos (100πt)
4 L 4 4

10 2
= cos (100πt)
2
Para n = 2 tomando L2
    
L 2π L
ξ2 , t = 10 sin cos (100πt) = 10 sin (π) cos (100πt)
2 L 2
=0

Para n = 2 tomando L4
    
L 2π L π
ξ2 , t = 10 sin cos (100πt) = 10 sin cos (100πt)
4 L 4 2
= 10 cos (100πt)

Para n = 2 tomando 3L 4
      
3L 2π 3L 3π
ξ2 , t = 10 sin cos (100πt) = 10 sin cos (100πt)
4 L 4 2
= −10 cos (100πt)

*********************************** ***********************************

165
10. Puede construirse un láser colocando un tubo de plasma en una cavidad
resonante óptica formada por dos espejos de alta reflectividad, que actúan
como paredes rı́gidas para las ondas luminosas (véase figura 73).El obje-
to del tubo de plasma es producir luz excitando modos normales de la
cavidad.

Figura 73: Figura problema 6.10

(a) ¿Cuáles son las frecuencias normales de la cavidad resonante?


(Expresar la respuesta en función de la distancia L entre los espejos y la
velocidad de la luz c.)
(b) Suponer que el tubo de plasma emite luz centrada en una
frecuencia ν0 = 5.0 × 1014 Hz con un ancho espectral ∆ν, como se ve
en el esquema. El valor de ∆ν es tal que todos los modos normales de
la cavidad cuya frecuencia está dentro de ±1.0 × 109 Hz de ν0 se verán
excitados por el tubo de plasma.
(1) ¿Cuántos modos se han excitado si L = 1.5 cm?
(2) ¿Cuál es el mayor valor de L que solo será excitado un modo
normal (de modo que el láser tendrá solamente una frecuencia de salida)?
(c = 3.0 × 108 m
s .)

Solución

(a) Para un sistema excitado de esta forma, las frecuencias normales se


calculan usando la ecuación de disperción. Esto es v = λn νn , donde la
velocidad de fase es la de la luz; ası́,
c
c = λn νn ∴ νn =
λn
2L
y como ya se conoce (ver texto) λn = n . Por tanto la frecuencia es
nc
∆νn =
2L
(b) Con L = 1.5 × 10−2 m y el valor de c,
3.0 × 108 n
∆νn = = 1.0 × 1010 n
3.0 × 10−2

166
Los modos normales que se excitan en la cavidad están entre los valores
ν = 5.0 × 1014 ± 1.0 × 109 ; esto es,
4.99999 × 1014 ≤ νn ≤ 5.00001 × 1014
por tanto
∆νn = 5.00001 × 1014 − 4.99999 × 1014 = 2.0 × 109 Hz
entonces reemplazando el valor de ∆νn en la ecuación resulta:
2.0 × 109 = 1.0 × 1010 n → n = 0.2
pero falta el modo en ν0 exactamente. Por tanto serán nT = 0.2 + 1 = 1.2
modos. Si la longitud fuera L = 1.5 m, entonces
2.0 × 109 = 1.0 × 108 n → n = 20 ∴ NT = 20 + 1 = 21 modos.
(b) Para una sola salida entonces n = 1. La longitud máxima es 2L; por
tanto
c 3.0 × 108
Lmáx = 2L = = = 0.15 m = 15 cm
νn 2.0 × 109
11. (a) Hallar la energı́a total de vibración de una cuerda de longitud L fija
en ambos extremos, que oscila en su modo caracterı́stico n con una am-
plitud A. La tensión de la cuerda es T y su masa total es M . (Indicación:
considérese la energı́a cinética integrada en el instante en que la cuerda
es rectilı́nea de modo que no ha almacenado ninguna energı́a potencial
respecto y por encima de la que tendrı́a cuando no estuviese vibrando en
absoluto.)
(b) Calcular la energı́a total de vibración de la misma cuerda, si está
vibrando en la siguiente superposición de modos normales.
 πx   
3πx  π
y(x, t) = A1 sin cos(ω1 t) + A3 sin cos ω3 t −
L L 4
(Deberı́a comprobarse que es la suma de las energı́as de los dos modos
tomados separadamente.)

Solución

(a) La función de onda de la cuerda fija en ambos extremos es:


 nπ 
yn (x, t) = A sin x cos (ωn t)
L
q q
pero v = ωknn , entonces ωn = kn v = λ2πn TML = nπ L
TL
M ; por tanto,
r !
 nπ  nπ T L
yn (x, t) = A sin x cos t
L L M
r !
 nπ  T
= A sin x cos nπ t
L ML

167
la energı́a cinética es
 2
1 ∂y(x, t)
dK = dm
2 ∂t
" r ! r !#2
µ T  nπ  T
= dx −A nπ sin x sin nπ t
2 ML L ML
r ! 
A2 n 2 π 2 T µ 2 T M  nπ 
= sin nπ t sin2 x dx
2M L ML L L
r !
A2 n 2 π 2 T 2 T 2 nπ
 
= sin nπ t sin x dx
2L2 ML L

usando la identidad trigonométrica sin2 (α) = 1−cos(2α)


2 , la integral queda:
L
r ! Z
A2 n 2 π 2 T
 
2 T 2nπ
K= sin nπ t 1 − cos x dx
4L2 ML L
0

teniendo en cuenta que la integral respecto a x de la función coseno se


anula y que la integral de dx es L,
r ! r !
A2 n 2 π 2 T T A 2 2 2
n π T T
K= sin2 nπ t L= sin2 nπ t
4L2 ML 4L ML
Cuando ha transcurrido un cuarto del perı́odo la energı́a mecánica es igual
a la cinética. Esto es cuando
r r r
τn 2π 2π 2πλn M L M 1 ML
t= = = = = =
4 4ωn 4vkn 4(2π) T L 2n T L 2n T
reemplazando,
r r !
A2 n 2 π 2 T T 1 ML A2 n 2 π 2 T π
E=K= sin2 nπ = sin2
4L M L 2n T 4L 2
ası́ la energı́a mecánica total es,
A2 n2 π 2 T
E=
4L
Sin usar el valor temporal, la energı́a potencial se halla teniendo en cuenta
que la pendiente de la cuerda tiene dos longitudes;
r la distancia horizontal
 2
∂y
p
dx y la hipotenusa dl = (dx)2 + (dy)2 = 1 + ∂x dx; esto hace que
la diferencia entre estas dos cantidades multiplicada por la tensión de la
cuerda sea el diferencial de la energı́a potencial:
s 
 2
∂y
dU = T (dl − dx) = T  1 + − 1 dx
∂x

168
donde usando el binomio de Newton se aproxima a
 2
1 ∂yn
dU ' T dx
2 ∂x
por tanto,
" r !#2
1 nπ  nπ  T
dU = T A cos x cos nπ t dx
2 L L ML
r !
A2 n 2 π 2 T 2 T 2 nπ
 
= cos nπ t cos x dx
2L2 ML L
integrando,
r !Z
L
A2 n 2 π 2 T T  nπ 
U= cos2 nπ t cos2 x dx
2L2 ML 0 L

usando la identidad trigonométrica cos2 (α) = 1+cos(2α) 2 , la integral queda:


r !Z
L
1 + cos 2nπ

A2 n 2 π 2 T 2 T L x
U= cos nπ t dx
2L2 ML 0 2
r !
A2 n 2 π 2 T 2 T
= cos nπ t
4L ML
2 2 2
 q 
Usando el resultado de la energı́a cinética K = A n4Lπ T sin2 nπ MTL t ,
la energı́a mecánica total es:
r ! r !
A2 n 2 π 2 T 2 T A2 n 2 π 2 T 2 T
E =K +U = sin nπ t + cos nπ t
4L ML 4L ML
" r ! r !#
A2 n 2 π 2 T 2 T 2 T
= sin nπ t + cos nπ t
4L ML ML
A2 n 2 π 2 T
=
4L
que coincide con el resultado anterior cuando se reemplazó un tiempo igual
a un cuarto de perı́odo.

(b) El gradiente de la posición y la velocidad están descritos por las ecua-


ciones:
 
dy(x, t) π  πx  π 3πx  π
= A1 cos cos(ω1 t) + A3 cos cos ω3 t −
dx L L L L 4
 
dy(x, t)  πx  3πx  π
= −ω1 A1 sin sin(ω1 t) − ω3 A3 sin sin ω3 t −
dt L L 4

169
y entonces usando las ecuaciones ya definidas,
Z   
1 2 πx
 
2 2 3πx  π
K= 2 2
ω1 A1 sin 2 2
sin (ω1 t) + ω3 A3 sin sin2 ω3 t −
2 L L 4
 πx    
3πx  π
+2ω1 ω3 A1 A3 sin sin sin(ω1 t) sin ω3 t − dm
L L 4
Z Z
ω12 A21 µ ω32 A23 µ
 
2 πx π 3πx
  
2 2 2
= sin (ω1 t) sin dx + sin ω3 t − sin dx
2 L 2 4 L
Z  
 π  πx  3πx
+ ω1 ω3 A1 A3 µ sin(ω1 t) sin ω3 t − sin sin dx
4 L L
integrando entre 0 y L,
L
Z L
Z   
 πx  1 2πx L
2
sin dx = 1 − cos dx =
0
L 2 L 2
0
L
Z L  
Z   
2 3πx 1 6πx L
sin dx = 1 − cos dx =
0
L 2 L 2
0
Z L  πx    Z L     
3πx 1 2πx 4πx
sin sin dx = cos − cos dx = 0
0
L L 2 0
L L
entonces,
ω12 A21 M ω 2 A2 M  π
K= sin2 (ω1 t) + 3 3 sin2 ω3 t −
4 4 4
y
Z " 2   
T  π 2 3π
 πx  3πx  π
U= A21 cos2 cos2 cos2 ω3 t −
cos2 (ω1 t) + A23
2 L L L L 4
 π   3π   πx  
3πx
  π  
+2A1 A3 cos cos cos(ω1 t) cos ω3 t − dx
L L L L 4
Z
T A21  π 2  πx 
= cos2 (ω1 t) cos2 dx
2 L L
 2 Z
T A23 3π
 
2
 π 2 3πx
+ cos ω3 t − cos dx
2 L 4 L
 π   3π 
Z  
 π  πx  3πx
+ T A1 A3 cos(ω1 t) cos ω3 t − cos cos dx
L L 4 L L
integrando entre 0 y L,
L
Z L
Z   
 πx  1 2πx L
2
cos dx = 1 + cos dx =
0
L 2 L 2
0

170
L
Z L  
Z   
2 3πx 1 6πx L
cos dx = 1 + cos dx =
0
L 2 L 2
0
Z L  πx    Z L     
3πx 1 2πx 4πx
cos cos dx = cos + cos dx = 0
0
L L 2 0
L L
entonces,

π 2 T A21 9π 2 T A23  π
U= cos2 (ω1 t) + cos2 ω3 t −
4L 4L 4
La energı́a mecánica total es,

ω12 A21 M ω 2 A2 M  π
E= sin2 (ω1 t) + 3 3 sin2 ω3 t −
4 4 4
π 2 T A21 9π 2 T A23  π
+ cos2 (ω1 t) + cos2 ω3 t −
4L 4L 4
pero

4L2 ωn2 Lω 2 n2 π 2 T
 
TL T
v2 = = λ2n ν 2 = 2 → = 2 n2 ∴ ωn2 M =
M n 4π 2 M n π L
π2 T 9π 2 T
reemplazando como ω12 M = L y ω32 M = L en la ecuación de energı́a,

π 2 T A21 9π 2 T A23  π
E= sin2 (ω1 t) + sin2 ω3 t −
4L 4L 4
π 2 T A21 2 9π 2 T A23  π
+ cos (ω1 t) + cos2 ω3 t −
4L 4L 4
Finalmente la energı́a mecánica es,

π 2 T A21 9π 2 T A23
E= + = E1 + E3
4L 4L

12. Una cuerda de longitud L está sujeta a ambos extremos y tiene una tensión
T , se empuja lateralmente a una distancia h de su centro y luego se deja
libre.

Figura 74: Figura problema 6.12

171
(a) ¿Cuál es la energı́a de las oscilaciones siguientes?
(b) ¿Con qué frecuencia reaparecerá la forma indicada en la figura?
(Admitir que la tensión permanece invariable debido a un pequeño incre-
mento de longitud causado por los desplazamiento transversales.) [Indica-
ción: En la parte (a), considerar el trabajo realizado conta la tensión para
dar a la cuerda una deformación inicial.]

Solución

(a) Las fuerzas en la cuerda son (seguidas las indicaciones del autor):

Figura 75: Figura problema 6.12a

Al sumar fuerzas se tiene que:


X
Fx → −Tx + Tx = 0
X
Fy → −2Ty = −2T sin(α) = Fy

En la figura 75 se observa un triángulo rectángulo con catetos y y L2 y


q
2
por tanto con hipotenusa y 2 + L4 y por tanto sin(α) = q y L2 . Ası́ la
y2 + 4
magnitud de Fy es
2T y
Fy = q
L2
y2 + 4

y por tanto el trabajo es


Z h
Z h
2T y y
E= q dy = 2T q dy
L2 L2
0
y2 + 4 0
y2 + 4

172
L2
con u = y 2 + 4 entonces du = 2ydy, y:
2
h2 + L4
r r !
√ h2 + L2 L2 L2
Z
du
E=T 2 √ = 2T u L2 4 = 2T h2 + −
L
4
u 4 4 4
 s 
 2
L 2h L
= 2T  1+ − 
2 L 2

donde finalmente la energı́a de la cuerda es


s 
 2
2h
E = TL 1 + − 1
L

q q
T TL
(b) La velocidad de fase de la cuerda es v = µ = M entonces la
frecuencia de oscilaciones es
r r r
TL 2L n TL n T
= λn νn = νn ∴ νn = =
M n 2L M 2 ML
Por tanto el estado aparece de nuevo cada perı́odo; esto es cuando se
cumpla r
2 ML
τn =
n T
13. Considérese un cubo uniforme de lado L en que la velocidad de onda
caracterı́stica es v. Demostrar que para este sistema el número total de
modos de oscilación correspondientes a las frecuencias entre ν y ν + dν es:

4L3 ν 2 ∆ν πv
si ≤ ∆ν ≤ ν.
π2 v3 L
1
[Indicación: Como νL 2 2
πv = n1 + n2 + n3
2 2
considérese una red cúbica de
puntos, poniendo x = n1 , y = n2 y z = n3 . El número de puntos en
cualquier región de la red es, por lo tanto, igual al volumen de dicha re-
gión, y los modos correspondientes a una frecuencia dada ν corresponden
a aquellos puntos situados a una distancia r = νL πv del origen. El resultado
deseado es, por tanto, 4πr2 ∆r. ¿Cuál serı́a el resultado para un cuadrado?
¿Para una varilla? ¿Cómo se verı́a alterada la respuesta si se considerase
en lugar de cubo un sólido ortoédrico de lados a, b y c?]

Solución

Inicialmente se observa que para un paralelepı́pedo existen ondas tridi-


mensionales con modos de oscilación λ2xx = nx , λ2yy = ny y λ2zz = nz , que
(ver figura 76):

173
Figura 76: Figura problema 6.13

λ λ λ
cos(α) = , cos(β) = , cos(γ) =
λx λy λz

elevándo estas ecuaciones al cuadrado y sumando,

λ2 λ2 λ2
cos2 (α) + cos2 (β) + cos2 (γ) = + +
λ2x λ2y λ2z

reemplazando los valores de lambda,

1 n2x 1 n2y 1 n2z


= , = , =
λ2x 4x2 λ2y 4y 2 λ2z 4z 2

la ecuación queda,

λ2 n2x λ2 n2y λ2 n2z


cos2 (α) + cos2 (β) + cos2 (γ) = + +
4x2 4y 2 4z 2

pero cos2 (α) + cos2 (β) + cos2 (γ) = 1, entonces para un cubo de lado L,

λ2 n21 λ2 n22 λ2 n23


1= + +
4L2 4L2 4L2
por tanto, se tiene que (ver figura 77),

4L2
n21 + n22 + n23 =
λ2

174
Figura 77: Figura problema 6.13

y entonces,
 21 2L
n21 + n22 + n23 =
λ
1 1
donde se observa en la figura 77 que n21 + n22 + n23 2 = x2 + y 2 + z 2 2 =
r y entonces:
2νL
r=
v
pero el texto en el enunciado dice que se tomará r = νL πv , donde se ve
una diferencia de resultados. Este cascarón circular de la figura 77 (que se
grafica ası́ porque es una onda tridimensional con frente de onda esférico)
2(ν+dν)L
tiene espesor entre 2νLv y v , y su densidad de frecuencias permi-
tidas en ese cuadrante es (esto es el número de puntos que hay entre los
cascarones) dada por N (r)dr = 18 (4πr2 )dr es
"  2 # 
4πL3 ν 2 dν

1 2νL 2dνL
N (ν)dν = 4π =
8 v v v3

Si usamos el valor de r dado en el enunciado r = νL πv , se obtiene el


resultado pedido
4L3 ν 2 dν
N (ν)cubo dν =
π2 v3
Para un cuadrado solo existen n1 y n2 por tanto se verifica que
λ2 n21 λ2 n22  12 2L 2νL
2
+ =1 → n21 + n22 = = =ρ
4L 4L2 λ v

175
que como se observa matemáticamente tiene la misma forma que la ecua-
ción en 3D (donde se usó la función r(x, y, z)). En el cuadrado la onda
se propaga con un frente de onda circular. Por tanto usando N (ρ)dρ =
1
4 (2πρ)dρ (coordenadas polares, el cuadrado centrado en el origen y to-
mando solamente la porción del primer cuadrante), el número de modos
de oscilación entre ν y ν + dν para un cuadrado es
2πL2 νdν
  
1 2νL 2dνL
N (ν)cuad dν = 2π =
4 v v v2
Resumiendo los resultados anteriores, se tiene que:
4πL3 ν 2 dν
N (ν)cubo dν = Cubo, con volumen L3 .
v3
2πL2 νdν
N (ν)cuad dν = Cuadrado, con área L2
v2  
2Lν
donde se observa que: N (ν)cubo dν = N (ν)cuad dν
v
| {z }
n

Para una varilla se observa que es un objeto tridimensional pero los ejes
ya no son x, y y z sino ρ, φ y z. En el cubo la onda tenı́a frente esférico,
pero en una varilla cilı́ndrica el frente de onda es una superficie curva y la
onda no es rotacional por tanto no depende del ángulo φ (ver figura 78);

Figura 78: Figura problema 6.13

entonces el área diferencial de la superficie de la onda es:


dA = r2 sin(θ)dθdφ = 2πr2 sin(θ)dθ

176
y se analiza que el ángulo θ varı́a desde 0 hasta arctan hρ . Por tanto la


densidad de los modos depende del ángulo y no de r como en los otros


casos. Esto es
N (θ)dθ = 2πr2 sin(θ)dθ = 2πrρdθ
2νh
usando r = v ,  
2νh
N (ν)dν = 2π ρdθ
v
****************************************************************
14. Hallar las series de Fourier para las funciones siguientes: (0 ≤ x ≤ L):

(a) y(x) = Ax(L − x).


 πx 
(b) y(x) = A sin .
( L
A sin 2πx , 0 ≤ x ≤ L2

(c) y(x) = L
L
0, 2 ≤ x ≤ L.

Solución

Para hallar la serie de Fourier es necesario hallar los coeficientes de Fourier


A0 , An y B n :
Z L
1
A0 = y(x)dx
L 0
Z L
2  nπ 
An = y(x) cos
x dx
L
0 L
2 L
Z  nπ 
Bn = y(x) sin x dx
L 0 L

(a) Para la primera función los coeficientes de Fourier son:


!
Z L Z L Z L
1 A
A0 = Ax(L − x)dx = L xdx − x2 dx
L 0 L 0 0
 3
L3

A L 1
= − = AL2
L 2 3 6

L
2A L
Z Z
2  nπ   nπ 
An = Ax(L − x) cos
x dx = x(L − x) cos x dx
L 0 L L 0 L
Z L
2A L 2
 nπ  Z  nπ 
= 2A x cos x dx − x cos x dx
0 L L 0 L

177


integrando por partes con: u = x, x2 y dv = cos L x dx; entonces du =
L
sin nπ

dx, 2xdx y v = nπ L x , resulta

2AxL  nπ  L 2AL Z L  nπ 
An = sin x − sin x dx
nπ L 0 nπ 0 L
2Ax2  nπ  L 4A Z L  nπ 
− sin x + x sin x dx
nπ L 0 nπ 0 L
2AxL  nπ  L 2AL2  nπ  L 2Ax2  nπ  L
= sin x + cos x − sin x

nπ L 0 (nπ)2 L 0 nπ L 0
L Z L
4AxL  nπ  4AL  nπ 
− cos x + cos x dx
(nπ)2 L 0 (nπ) 2
0 L
2AxL  nπ  L 2AL2  nπ  L 2Ax2  nπ  L
= sin x + cos x − sin x

nπ L (nπ)2 L nπ L

0 0 0

4AxL  nπ  L 4AL 2  nπ  L
− cos x + sin x
(nπ)2 L 0 (nπ)3 L 0

usando el teorema fundamental del cálculo,

2AL2 2AL2 2AL2 2AL2


An = sin (nπ) + cos (nπ) − − sin (nπ)
nπ (nπ)2 (nπ)2 nπ
4AL2 4AL2
− cos (nπ) + sin (nπ)
(nπ)2 (nπ)3

simplificando,

2AL2 4AL2
An = − 2
[cos (nπ) + 1] + sin (nπ)
(nπ) (nπ)3

teniendo en cuenta que n es un entero, entonces:

2AL2
An = − [cos (nπ) + 1]
(nπ)2

Si n es impar el coeficiente se anula. Entonces para n par,

4AL2
An = −
(nπ)2

Calculando el último coeficiente,

2 L 2A L
Z  nπ  Z  nπ 
Bn = Ax(L − x) sin x dx = x(L − x) sin x dx
L 0 L L 0 L
Z L
2A L 2
 nπ  Z  nπ 
= 2A x sin x dx − x sin x dx
0 L L 0 L

178


integrando por partes con: u = x, x2 y dv = sin L x dx; entonces du =
L
cos nπ

dx, 2xdx y v = − nπ L x , resulta

2AxL  nπ  L 2AL Z L  nπ  2Ax2  nπ  L


Bn = − cos x +
cos x dx + cos x
nπ L 0 nπ 0 L nπ L 0
Z L
4A  nπ 
− x cos x dx
nπ 0 L
2AxL  nπ  L 2AL2  nπ  L 2Ax2  nπ  L
=− cos x +
sin x +
cos x
nπ L 0 (nπ)2 L 0 nπ L 0
L Z L
4AxL  nπ  4AL  nπ 
− sin x + sin x dx
(nπ)2 L 0 (nπ)2
0 L
2AxL  nπ  L 2AL2  nπ  L 2  L
+ 2Ax cos nπ x

=− cos x + sin x
nπ L 0 (nπ)2 L
0 nπ L
0

4AxL  nπ  L 4AL2  nπ  L
− sin x − cos x

(nπ)2 L 0 (nπ)3 L 0

usando el teorema fundamental del cálculo,


2AL2 2AL2 2AL2
Bn = − cos (nπ) + 2
sin (nπ) + cos (nπ)
nπ (nπ) nπ
4AL2 4AL2 4AL2
− 2
sin (nπ) − 3
cos (nπ) +
(nπ) (nπ) (nπ)3
simplificando
2AL2 4AL2
Bn = − 2
sin (nπ) − [cos (nπ) − 1]
(nπ) (nπ)3
teniendo en cuenta que n es entero,
4AL2
Bn = − [cos (nπ) − 1]
(nπ)3
si n es par se anula el coeficiente; si n es impar,
8AL2
Bn =
(nπ)3
Finalmente los coeficientes de Fourier son
AL2 4AL2 8AL2
A0 = , An(par) = − , y Bn(impar) = .
6 (nπ)2 (nπ)3
ó,
AL2 4AL2 8AL2
A0 = , An = − , y Bn = (n = 1, 2, 3, . . .)
6 [(2n)π]2 [(2n − 1)π]3

179
π

(b) Para la función y(x) = A sin L x los coeficientes de Fourier se calculan
usando,
A L
Z π 
A0 = sin x dx
L 0 L
2A L
Z π   nπ 
An = sin x cos x dx
L 0 L L
2A L
Z  nπ  π 
Bn = sin x sin x dx
L 0 L L
El coeficiente A0 es,
A L
Z π   π  L
A A
A0 = sin x dx = − cos x = − [cos (π) − 1]

L 0 L π L 0 π
2A
=
π
Usando la identidad: sin(α) cos(β) = 12 [sin(α + β) + sin(α − β)] el segundo
coeficiente queda,
A L
Z     
(n + 1)π (1 − n)π
An = sin x + cos x dx
L 0 L L
A L A L
   
(1 − n)π
Z Z
(n + 1)π
= sin x dx + cos x dx
L 0 L L 0 L
  L   L
A (n + 1)π A (1 − n)π
=− cos x + sin x
(n + 1)π L 0 (1 − n)π L 0
A A
=− {cos [(n + 1)π] − 1} + sin [(1 − n)π]
(n + 1)π (1 − n)π

con n como entero,


A {cos [(n + 1)π] − 1}
An = −
(n + 1)π
para n impar, el coeficiente se anula; para n par
2A
An =
(n + 1)π
Usando la identidad: sin(α) sin(β) = 21 [cos(α − β) − cos(α + β)] el tecer
coeficiente queda,
A L
Z     
(n − 1)π (n + 1)π
Bn = cos x − cos x dx
L 0 L L
A L A L
   
(n − 1)π
Z Z
(n + 1)π
= cos x dx − cos x dx
L 0 L L 0 L

180
realizando un cambio de variables u = (n−1)π L x y v = (n+1)π
L x y entonces
(n−1)π (n+1)π
du = L dx y dv = L dx, las integrales quedan,
Z (n−1)π Z (n+1)π
A A
Bn = cos (u) du − cos (v) dv
(n − 1)π 0 (n + 1)π 0
A (n−1)π A (n+1)π
=− sin (u)|0 + sin (v)|0
(n − 1)π (n + 1)π
A A
=− sin [(n − 1)π] + sin [(n + 1)π]
(n − 1)π (n + 1)π
A
=− sin (nπ) cos (−π) − cos (nπ) sin (−π)
(n − 1)π
A
+ sin (nπ) cos (π) + cos (nπ) sin (π)
(n + 1)π
A A
= sin (nπ) − sin (nπ)
(n − 1)π (n + 1)π
 
A sin (nπ) 1 1 2A sin (nπ)
= − =
π n−1 n+1 π(n2 − 1)
como n es un entero con la condición n > 1 entonces,
Bn = 0
Finalmente los coeficientes de Fourier son:
2A 2A
A0 = , An(par) = y Bn(n>1) = 0
π (n + 1)π
ó:
2A 2A
A0 = , An = y Bn = 0 (n = 2, 3, 4, . . .)
π [(2n − 3) + 1]π
(c) Para la tercera función los coeficientes de Fourier son:
Z L  
A 2 2π
A0 = sin x dx
L 0 L
Z L  
2A 2  nπ  2π
An = cos x sin x dx
L 0 L L
Z L2  
2A  nπ  2π
Bn = sin x sin x dx
L 0 L L
Calculando el primero,
Z L       L
A 2 2π A 2π 2
A0 = sin x dx = − cos x
L 0 L 2π L 0
 
A A
=− [cos (π) − 1] =
2π π

181
Usando la identidad: cos(β) sin(α) = 12 [sin(α + β) + sin(α − β)] el segundo
coeficiente queda,
Z L   Z L  
A 2 (n + 2)π A 2 (2 − n)π
An = sin x dx + sin x dx
L 0 L L 0 L
(n+2)π (2−n)π (n+2)π
Haciendo u = L x, v = L x y entonces, du = L dx, dv =
(2−n)π
L dx
L   Z L  
(2 − n)π
Z
A 2 (n + 2)π A 2
An = sin x dx + sin x dx
L 0 L L 0 L
Z (n+2)π Z (n−2)π
A 2 A 2
= sin(u)du − sin(u)du
(n + 2)π 0 (n − 2)π 0
A (n+2)π
A (n+2)π
=− cos(u)|0 2 + cos(u)|0 2
(n + 2)π (n − 2)π
n h i o n h i o
(n+2)π (n+2)π
A − cos 2 + 1 A cos 2 − 1
An = +
(n + 2)π (n − 2)π
A cos nπ A cos nπ
     
2 +1 2 +1
= −
(n + 2)π (n − 2)π

Pero cos2 α2 = cos(α)+1



2 por tanto,
2A cos2 nπ 2A cos2 nπ 2A cos2 nπ
   
4 4 4 1 1
An = − = −
(n + 2)π (n − 2)π π n−2 n+2
2 nπ

8A cos 4
=
π(n2 − 4)
Si n es par, el coeficiente se anula. Para valores de n impar, el valor de An
varı́a de signo de la forma 1, −1, −1, 1, 1, −1, −1, . . . Y esta serie se
puede definir con:
   
kπ kπ
cos + sin (k = 0, 1, 2, 3, . . .)
2 2
el coeficiente de Fourier es entonces:
 √ 2
8A 22   

 

An = cos + sin
π(n2 − 4) 2 2
    
4A kπ kπ
= cos + sin (para n 6= 2)
π(n2 − 4) 2 2
Para hallar el tercer coeficiente de Fourier, se usa la identidad trigo-
nométrica sin(α) sin(β) = 21 [cos(α − β) − cos(α + β)]
Z L   Z L  
A 2 (n − 2)π A 2 (n + 2)π
Bn = cos x dx − cos x dx
L 0 L L 0 L

182
realizando el cambio de variable u = (n−2)π
L x, v = (n+2)π
L x y entonces,
(n−2)π (n+2)π
du = L dx, dv = L dx la integral queda:
Z (n−2)π Z (n+2)π
A 2 A 2
Bn = cos (u) du − cos (u) du
(n − 2)π 0 (n + 2)π 0
A (n−2)π
A (n+2)π
=− sin (u)|0 2 + sin (u)|0 2
(n − 2)π (n + 2)π
h i h i
A sin (n−2)π2 A sin (n+2)π
2
=− +
(n − 2)π (n + 2)π
A sin nπ A sin nπ 2A sin nπ
   
2 2 2 1 1
= − = −
(n − 2)π (n + 2)π π n−2 n+2
4A sin nπ

2
=
π(n2 − 4)

Para valores pares de n el coeficiente se anula. Para los valores impares


de n, los signos cambian de la forma 1, −1, −1, 1, 1, −1, −1, . . . Y esta
serie se puede definir con:
   
kπ kπ
cos + sin (k = 0, 1, 2, 3, . . .)
2 2

El coeficiente de Fourier es:


    
4A kπ kπ
Bn = 2
cos + sin (n = 0, 1, 3, . . .)
π(n − 4) 2 2

Finalmente los coeficientes de Fourier para la función dada y para n impar


y n 6= 2 son:
A
A0 =
π     
4A kπ kπ
An = cos + sin
π(n2 − 4) 2 2
    
4A kπ kπ
Bn = cos + sin
π(n2 − 4) 2 2

ó también,
A
A0 =
π     
4A kπ kπ
An = cos + sin
π[2(n − 1)2 − 4] 2 2
    
4A kπ kπ
Bn = cos + sin
π([2n − 1)2 − 4] 2 2

183
15. Hallar las series de Fourier para el movimiento de una cuerda de longitud
L si
 
∂y
(a) y(x, 0) = Ax(L − x); = 0.
∂t t=0
 
∂y
(a) y(x, 0) = 0; = Bx(L − x).
∂t t=0

Solución

Para el movimiento de una cuerda, la función de onda es solución de la


ecuación de onda unidimensional. Esta función resulta ser (ver el texto):
 nπ   nπ 
yn (x, t) = y0 sin x cos(ωn t) = y0 cos(ωn t) sin x
L | {z } L
Cn (t)
 nπ 
= Cn (t) sin x
L
Pero en general se tienen soluciones de la forma,
yn (x, t) = Cn (t)f (x)
(a) Con y(x, 0) = Ax(L−x) entonces f (x)
 = Ax(L−x); por tanto y(x, t) =
∂y
Ax(L − x)g(t) y y0 = A. También ∂t = 0; entonces al evaluar esta
t=0
condición inicial eligiendo h(t) = cos(ωn t), se observa que:
∂y(x, t) ∂
= [Ax(L − x) cos(ωn t)] = −Axω(L − x) sin(ωn t)
∂t ∂t
donde se observa que se cumple,
 
∂y(x, t)
= −Axω(L − x) sin(ωn t)|t=0 = 0
∂t t=0

y se puede concluir que el valor h(t) = cos(ωn t) es correcto, y este se


puede reescribir usando el hecho de que cualquier frecuencia de un modo
determinado es un múltiplo entero de la fecuencia fundamental ω1 . Por
tanto h(t) = cos(nω1 t). La ecuación de onda es:
yn (x, t) = Ax(L − x) cos(nω1 t), (que implica suma sobre n).
Para esta función ya se hallaron los coeficientes de Fourier resultando:
AL2
A0 =
6
4AL2 cos(nω1 t)
An(par) =−
(nπ)2
8AL2 cos(nω1 t)
Bn(impar) =
(nπ)3

184
(b) Para esta función de onda y(x, 0) = 0, por tantohn (t)
 = sin(nω1 t), que
∂y
la satisface inmediamente. La condición inicial es ∂t = Bx(L − x),
t=0
enntonces:
∂y(x, t) ∂
= [f (x) sin(nω1 t)] = f (x)nω1 cos(nω1 t)
∂t ∂t
evaluando la derivada,
 
∂y(x, t)
= f (x)nω1 cos(nω1 t)|t=0 = f (x)nω1 = Bx(L − x)
∂t t=o

donde se concluye que


Bx(L − x)
f (x) =
nω1
y por tanto la función de onda es,

Bx(L − x)
y(x, t) = sin (nω1 t)
nω1
Los coeficientes de Fourier para esta función ya fueron hallados, resultando

BL2
A0 =
6
4BL2 sin(nω1 t)
An(par) =−
ω1 n3 π 2
2
8BL sin(nω1 t)
Bn(impar) =
ω1 n 4 π 3

7. Ondas progresivas
1. Comprobar que las ecuaciones siguientes pueden utilizarse para describir
la misma onda progresiva:

y = A sin [2π(x − vt)]


y = A sin [2π(kx − νt)]
   
x t
y = A sin −
λ τ
h  x i
y = A sin ω t −
h v i
i2π(kx−ωt)
y = A Im Ae

Solución

185
ω
Comenzando con kx − ωt y usando la ecuación de dispersión v = λν = k,
k = 2π 1
λ y ν = τ , entonces se verifica que
   
2π 2π 2π x t
kx − ωt = x − 2πνt = x− t = 2π −
λ λ τ λ τ
   
kx kx x 
kx − ωt = −t ω =ω −t =ω −t
ω kv v
se encuentra que:
   
x t x 
2π − =ω −t
λ τ v

también,
ω 2πν 2π
kx − ωt = x − vkt = x − vkt = x − vkt = k(x − vt)
v λν λ
o si se usa 2πκ = k, entonces:
kx − ωt = 2πκx − 2πνt = 2π(κx − νt)

resultando:
k(x − vt) = 2π(κx − νt)
Finalmente se llega a
   
x t x 
kx − ωt = 2π − =ω − t = k(x − vt) = 2π(κx − νt)
λ τ v

Lo que no concuerda con lo pedido. Analizando dimensionalmente las fun-


ciones de onda, es evidente que no concuerdan porque los argumentos de
las funciones sinusoidales son (donde 2π tiene dimensión de radianes):
h  m  i
2π(x − vt) = rad · m − · s = rad · (m − m) = rad · m
  s   
rad 1 rad
2π(kx − νt) = rad · − · s = rad · −1 = ¿?
m s m
x  
t h m   s i
− = − = adimensional
λ τ m s
 
 x rad  m · s  rad
ω t− = s− = (s − s) = rad
v s m s
   
rad rad 2
2π(kx − ωt) = rad · ·m− · s = rad
m s

186
donde se esperarı́a que las dimensiones sean angulares; esto es en radianes,
de las cuales solo una la tiene. Las funciones de onda correctas son:

y = A sin [k(x − vt)]


y = A sin [2π(κx − νt)]
    
x t
y = A sin 2π −
λ τ
h x i
y = A sin ω −t
h v i
y = A Im Aei2π(kx−ωt)

2. La ecuación de una onda transversal que se mueve a lo largo de una cuer-


da viene dada por y = 0.3 sin [π (0.5x − 50t)], en donde y y x están en
centı́metros y t en segundos.
(a) Hallar la amplitud, la longitud de onda, el número de onda, la
frecuencia, el perı́odo y la velocidad de la onda.
(b) Hallar la velocidad transversal máxima de cualquier partı́cula
en la cuerda.

Solución

(a) Analizando la función de onda dada,


 

y = 0.3 sin [π (0.5x − 50t)] = |{z} |{z} x − |{z}


0.3 sin  0.5π 50π t
 
A k= 2π ω=2πν
λ


por tanto A = 0.3 cm, λ = 0.5π = 4 cm, k = 0.5π, ν = 50π
2π = 25 s−1 ,
1
τ = 25 = 0.04 s y v = λν = 4(25) = 100 cm
s .

∂y
(b) La velocidad transveral es vy = ∂t ; entonces,


vy = 0.3 sin [π (0.5x − 50t)] = −0.3(50)π cos [π (0.5x − 50t)]
∂t
= −15π cos [π (0.5x − 50t)]

la velocidad transversal máxima es cuando cos [π (0.5x − 50t)] = −1; por


lo tanto,
cm
vy,max = 15π
s
3. ¿Cuál es la ecuación para una onda longitudinal que se mueve en el sentido
de las x negativas con ampltud 0.003 m, frecuencia 5 s−1 , y una velocidad
de 3000 m s ?

187
Solución

Como la frecuencia es de ν = 5 s−1 , entonces la frecuencia angular es


ω = 2πν = 10π rads . Al moverse en el sentido negativo de las x, entonces
el modelo de la ecuación es

x = A cos(kx + ωt)

con v = 3000 = λν = 5λ se tiene que la longitud de onda es λ = 3000 5 =


600 m y entonces el número de onda es, k = 2πλ = 2π
600 = π rad
300 m . Reem-
plazando los valores se llega a
 π  h  x i
x = 0.003 cos x + 10πt = 0.003 cos 2π + 5t
300 600

4. Una onda de frecuencia 20 s−1 tiene una velocidad de 80 m


s .
(a) ¿A qué distancia están dos puntos cuyos desplazamientos estén
separados 30o en fase?
(b) En un punto dado, ¿cuál es la diferencia de fase entre dos des-
plazamientos que se producen en tiempos separados por 0.01 s?

Solución

1
(a) Con el perı́odo igual a τ = 20 s, entonces se tiene que la variación
de fase para el perı́odo es de 360o y ası́ el tiempo para una fase de 30o
30 1
se halla multiplicando por el factor de conversión 360 = 12 , resultando el
o
tiempo para recorrer una separación de fase de 30 :
 
1 1 1
t30 = τ30 = = s
20 12 240

Finalmente usando la velocidad v = λτ = 240λ y despejando la longitud


de onda resulta
80 1
λ= = m.
240 3
(b) Con el mismo procedimiento usado en  el ı́tem (a), para un perı́odo la
1
la longitud de onda es: λ = vτ = 80 20 = 4 m siendo esta la distancia
recorrida en una diferencia de fase de 360o . Para diferencia temporal de
0.01 s, la diferencia de fase es:

360o (0.01)
∆α = 1 = 360o (0.01)(20) = 72o
20

5. Se tensa una cuerda uniforme larga de densidad de masa 0.1 kg m , con una
fuerza de 50 N . Un extremo de la cuerda (x = 0) se hace oscilar transver-
salmente (sinusoidalmente) con ua amplitud de 0.02 m y un perı́odo de

188
0.1 s, de modo que se generan unas ondas que se mueven en el sentido de
las x positivas
(a) ¿Cuál es la velocidad de las ondas?
(b) ¿Cuál es la longitud de onda?
(c) Si en el extremo impulsor (x = 0) el desplazamiento (y) para
t = 0 es 0.01 m con ∂y∂t negativa, ¿cuál es la ecuación de las ondas que se
mueven?

Solución

(a) La velocidad de fase de onda en una cuerda unidimensional es,


s
√ √
r
T 50 m
v= = = 500 = 10 5 = 22.4
µ 0.1 s

(b) Usando la ecuación de dispersión,


v
λ= = vτ = 22.4(0.1) = 2.24 m
ν
(c) La ecuaciń de onda general es y = A cos(kx − ωt + α) y se tienen
los valores de A = 0.02 m, ω = 2π 2π
τ = 0.1 = 20π
rad
s = 62.8 rads y
2π rad
k = 2.24 = 2.80 m , resultando

y = 0.02 cos(2.80x − 62.8t + α)

Si se calcula la derivada ∂y
∂t , esta da positiva, por tanto la función de onda
correcta es
y = 0.02 sin(2.80x − 62.8t + α)
en t = 0 se sabe que x = 0,
1 π
y = 0.02 sin(α) = 0.01 → sin(α) = → α= = 0.52
2 6
Finalmente la ecuación de onda es:

y = 0.02 sin(2.80x − 62.8t + 0.52)

6. Se observa que un pulso necesita 0.1 s para recorrer de un extremo a otro


una cuerda larga. La tensión en la cuerda se obtiene haciéndola pasar sobre
una polea y colgando un peso que tiene 100 veces la masa de la misma.
(a) ¿Cuál es la longitud de onda de la cuerda?
(b) ¿Cuál es la ecuación del tercer modo normal?

Solución

189
(a) La tensión es entonces T = 100mg, y se verifica que
s s
T 100mg p
v= = m = 10 gL
µ L

pero,
∆x L
v= = = 10L
∆t 0.1
Entonces reemplazando este valor resulta gL = L2 y ası́ la cuerda tiene
una longitud L = g = 9.8 m. La longitud de onda es

19.6 m, para n = 1.
2L 2g 19.6 
λn = = = = 9.8 m, para n = 2.
n n n  19.6

3 m, para n = 3.

(b) La función de onda es


h nπ  i
yn (x, t) = A cos(kn x − ωn t) = A cos x − 2nπt
L
que para el tercer modo es,
  

y3 (x, t) = A cos x − 6πt
9.8

Si se hace la aproximación L = 10 m, entoces λ = 20


n los resultados son:

20 m, para n = 1.
2L 2g 19.6 
λn = = = = 10 m, para n = 2.
n n n  20

3 m, para n = 3.

7. Una cuerda larga de la misma tensión que la del problema 7-6 tiene una
onda móvil sobre ella con la ecuación siguiente

y(x, t) = 0.02 sin[π(x − vt)]

en donde x e y están en metros, t en segundos y v es la velocidad de onda


(que puede calcularse). Hallar el desplazamiento transversal y la velocidad
de la cuerda en el punto x = 5 m en el instante t = 0.1 s.

Solución

m
Según el problema anterior la velocidad de onda es v = 10L = 98 s ;
y la ecuación queda:

y(x, t) = 0.02 sin[π(x − 98t)]

190
el desplazamiento transversal en el punto x = 5 m en el instante t = 0.1 s
es:
y(x, t) = 0.02 sin[π(5 − 9.8)] = −0.01 m
Si (como en el problema anterior que en la parte final se aproxima a
L = 10 m) v = 100 m
s , entonces la ecuación queda:

y(x, t) = 0.02 sin[π(x − 100t)]

el desplazamiento transversal en el punto x = 5 m y en el instante t = 0.1 s


es:
y(x, t) = 0.02 sin[π(5 − 10)] = 0 m
La velocidad transversal es
∂y(x, t)
= vy = 0.02(−98π) cos[π(x − 98t)]
∂t
= −1.96π cos[π(x − 98t)]

en el punto x = 5 m y en el instante t = 0.1 s esta velocidad es:


m
vy = −1.96π cos[π(5 − 9.8)] = −6.16 cos(−4.9π) = 5.85 ' 6
s
m
O aproximando con v = 100 s ,

m
vy = −2π cos[π(5 − 10)] = −6.28 cos(−5π) = 6.28 ' 6
s

8. Se observan dos puntos en una cuerda cuando una onda móvil pasa por
ella. Los puntos están en x1 = 0 my x2 = 1 m. Los movimientos transver-
sales de los dos puntos resultaron ser del modo siguiente:

y1 = 0.2 sin(3πt)
 π
y2 = 0.2 sin 3πt +
8

(a) ¿Cuál es la frecuencia en Hz?


(b) ¿Cuál es la longitud de onda?
(c) ¿Con qué velocidad de mueve la onda?
(d) ¿De qué modo se mueve la onda? Explicar cómo puede llegarse
a esta conclusión.

Atención! Considerar con cuidadosi existe alguna ambig´’uedad debido a


la cantidad limitada de información que se ha dado.

Solución

(a) La frecuencia se halla teniendo en cuenta que en la ecuación dada

191
ω = 3π y es sabido que ω = 2πν. Por tanto 3π = 2πν, donde el valor de
la frecuencia lineal es: ν = 3π
2π = 1.5 Hz. Pero esta frecuencia debe ser
para un modo de oscilación determinado, pues como se sabe ωn = 2nπν.
Por tanto se supone que el modo de oscilación de estas ecuaciones es n = 1.

(b) Para la longitud de onda se observa en la ecuación de onda para el


punto dos y n = 1  
 π 
y1,2 = 0.2 sin 3πt +
 
8

 |{z}
2nπ
L

que 2nπ π 2π π
L = 8 , que para n = 1, L = 8 , y se concluye que la longitud de la
cuerda es L = 16 m. La ecuación general de la cuerda es entonces (donde
el valor que toma la longitud de onda es completamente independiente del
sentido de movimiento de la onda),
 nπ 
yn,x = 0.2 sin 3nπt + x
16
nπ 2π
donde kn = 16 = λn y entonces


 32 m para n = 1

16 m para n = 2

n 2 32 
= → λn = =
16 λn n 
 8m para n = 3
.. ..


. .

(c) La velocidad es independiente del modo de oscilación y por tanto es


solo una. Se puede observar que:
ωn 2πnν nν2L m
v= = 2π = = 2Lν = 2(16)1.5 = 48
kn λn
n s

(d) Se puede afirmar qe se mueve en sentido negativo de las x con una


velocidad de fase de 48 m 32
s , una longitud de onda de n y una frecuencia
lineal de 3.5 Hz.

9. Un pulso triangular simétrico de altura máxima 0.4 m y una longitud to-


tal 1.0 m se mueve en el sentido positivo de las x sobre una cuerda en la
que la velocidad de onda es de 24 m s . Para t = 0 s, el pulso está situado
totalmente entre x = 0 m y x = 1 m. Dibujar un gráfico de la velocidad
transversal en función del tiempo para x = x2 = +1 m.

Solución

El pulso trinagular puede observarse como en la figura 79.

192
Figura 79: Figura problema 7.9

El pulso será una función de la forma y(x, t) = f (x−24t), donde en general


y1 (x1 , t) = f (x − 24t) y y2 (x2 , t) = f ((x + 1) − 24t). Su derivada temporal
(velocidad transversal) es en t = 0 s para las posiciones x1 y x2 :

∂f (x, 0)
vy (x1 , 0) = = g(vx )
∂t
∂f (x + 1, 0)
vy (x2 , 0) = = g(vx )
∂t
donde se observa que g(vx ) = 0 y por tanto esa velocidad permanece nula
durante el movimiento del pulso, donde vx es constante. Adicionalmente
en el punto medio del pulso la velocidad transversal es nula; esto es

∂f ∆x
  
∆x 2 ,0
vy ,0 = =0
2 ∂t

y esta velocidad nula localizada en el punto medio, se mantiene nula du-


rante todo el movimiento del pulso. Debido a que no existe una función
matemática definida, entoces la gráfica debe hacer con intuición fı́sica
usando los resultados anteriores. Se observa en a la parte final del pul-
so las velocidades transversales deben ser negativas hasta anularse en la
parte media y en la parte al comienzo del pulso estas deben ser positivas
hasta anularse en la parte media, pero debido a que es una lı́nea recta, las
velocidades por unidad de longitud son iguales siempre y sin disminuir su
magnitud. Por tanto la gráfica es (ver la figura 80):

193
Figura 80: Figura problema 7.9

10. El extremo (x = 0 m) de una cuerda tensa se mueve transversalmente con


una velocidad constante de 0.5 ms durante 0.1 s (empezando en t = 0 s) y
vuelve a su posición normal durante los siguientes 0.1 s, de nuevo a velo-
cidad constante. El pulso de onda resultante se mueve con una velocidad
de 4 m
s .
(a) Dibujar el espectro de la cuerda para t = 0.4 s, y para t = 0.5 s.
(b) Dibujar un gráfico de la velocidad transversal en función de x
para t = 0.4 s.

Solución

(a)
11. Suponer que un pulso de una pnda móvil está descrito por la ecuación

b3
y(x, t) =
b2 + (x − vt)2

con b = 5 cm y v = 2.5 cm s . Dibujar el perfil del pulso como aparecerı́a


cuando t = 0 s y t = 0.2 s. Por sustracción directa de las ordenadas de
ambas curvas obtener una descripción apropiada de la velocidad trans-
versal en función de x para t = 0.1 s. Compararla con la que se obtiene
calculando ∂y
∂t para un valor de t arbitrario y luego haciendo t = 0.1 s.

Solución

Para dibujar el pulso se evalúa primero el valor de la función en t = 0 s:

b3
y(x, 0) =
b2 + x 2
Esta función en x = 0 m vale
b3
y(0, 0) = =b
b2

194
que no informa que la altura máxima del pulso es y = b =constante. El
valor de y(0, t) es (donde la velocidad v es constante):

b3
y(0, t) =
b2 + (vt)2
y como se observa,
b3
lı́m y(0, t) = lı́m →0
t→±∞ t→±∞ b2 + (vt)2

donde se puede ver que el dominio es: −∞ ≥ t ≥ ∞ y el rango es: b ≥


y(0, t) ≥ 0. Con esta descripción matemática, las gráficas de la función en
t = 0 s y t = 0.2 s son (ver figura 81):

Figura 81: Figura y(x, 0) y y(x, 0.2) problema 7.11

La sustracción directa ∆y = y(x, t2 ) − y(x, t1 ), para dos tiempo diferentes


es:
b3 b3
∆y = − 2
b2 + (x − vt2 ) 2 b + (x − vt1 )2
 
1 1
= b3 2 −
b + (x − vt2 )2 b2 + (x − vt1 )2
 2
b + (x − vt1 )2 − [b2 + (x − vt2 )2 ]

= b3
[b2 + (x − vt2 )2 ][b2 + (x − vt1 )2 ]
(x − vt1 )2 − (x − vt2 )2
 
= b3
[b2 + (x − vt2 )2 ][b2 + (x − vt1 )2 ]
2x(t2 − t1 ) − v(t22 − t21 )
 
= b3 v
[b2 + (x − vt2 )2 ][b2 + (x − vt1 )2 ]
Con ∆t = t2 − t1 ,
 
2x − v(t2 + t1 )
∆y = b3 v ∆t
[b2 + (x − vt2 )2 ][b2 + (x − vt1 )2 ]
 
2x − v(t2 + t1 )
= b3 v ∆t
[b2 + (x − v(∆t + t1 ))2 ][b2 + (x − v(t2 − ∆t))2 ]

195
entonces expandir el denominador no lleva a algo interesante, debido a
que no se puede construir el ∆t porque son productos con el mismo signo.
Si solo se calcula la velocidad media transversal sin hacer algo más, esta
queda como:
 
∆y 2x − v(t2 + t1 )
= b3 v
∆t [b2 + (x − v(∆t + t1 ))2 ][b2 + (x − v(t2 − ∆t))2 ]
donde hay dependencia explı́cita de los tiempos t1 y t2 . Calculando el
lı́mite cuando ∆t → 0, esta queda:
∆y
vy = lı́m
∆t→0 ∆t  
2x − v(t2 + t1 )
= lı́m b3 v
∆t→0 [b2 + (x − v(∆t + t1 ))2 ][b2 + (x − v(t2 − ∆t))2 ]
 
2x − v(t2 + t1 )
= b3 v
[b2 + (x − vt1 )2 ][b2 + (x − vt2 )2 ]
Si t1 ' t2 ' t (que equivale al lı́mite ∆t → 0), entonces,
 
2x − v(2t)
v y = b3 v
[b2 + (x − vt)2 ][b2 + (x − vt)2 ]
 
x − vt
= 2b3 v .
[b2 + (x − vt)2 ]2
Si por otro lado, se calcula la velocidad de forma diferencial, se tiene que:
∂y(x, t) b3 [2(x − vt)(−v)] 2vb3 (x − vt)
= vy = − 2 =
∂t [b + (x − vt)2 ]2 [b2 + (x − vt)2 ]2
resultando que los cálculos llegan al mismo resultado.

Siguiendo con lo pedido, se escribirán las ecuaciones con los valores dados
b = 5 cm y v = 2.5 cm s . Para la velocidad hallada mediante la sustracción
esta queda como:
 
2x − 2.5(t2 + t1 )
vy (x, t1 , t2 ) = 53 (2.5)
[52 + (x − 2.5t1 )2 ][52 + (x − 2.5t2 )2 ]
312.5[2x − 2.5(t2 + t1 )]
=
[25 + (x − 2.5t1 )2 ][25 + (x − 2.5t2 )2 ]
reemplazando t1 = 0 s y t2 = 0.1 s,
312.5(2x − 0.25) 625(x − 0.125)
vy (x, 0.0, 0.1) = 2 2
= .
(25 + x )[25 + (x − 0.25) ] (25 + x2 )[25 + (x − 0.25)2 ]
Para la velocidad obtenida por derivación esta queda:
2(2.5)53 (x − 2.5t) 625(x − 2.5t)
vy (x, t) = =
[52 + (x − 2.5t)2 ]2 [25 + (x − 2.5t)2 ]2

196
reemplazando el valor t = 0.1 s,

625(x − 0.25)
vy (x, 0.1) = .
[25 + (x − 0.25)2 ]2

Quedando las velocidades,

625(x − 0.125)
vy (x, 0.0, 0.1) =
(25 + x2 )[25 + (x − 0.25)2 ]
625(x − 0.25)
vy (x, 0.1) =
[25 + (x − 0.25)2 ]2

Las gráficas de las dos velocidades son (ver gráfica 82),

Figura 82: Figura vy (x, 0.0, 0.1) y vy (x, 0.1) problema 7.11

Se observa que hay una pequeña diferencia en la región −2.5 ≤ x ≤ 2.5,


que es mayor en x = 0 donde:

625(0.125)
vy (0, 0.0, 0.1) = −
25(25 + 0.252 )
625(0.25)
vy (0, 0.1) = −
(25 + 0.252 )2
vy (0,0.1)
donde es evidente que, vy (0, 0.0, 0.1) = 25 , pero esta diferencia de-
crece fuera de la región mencionada.
12. La figura muestra un pulso sobre una cuerda de longitud 100 m con extre-
mos fijos. El pulso se mueve hacia la derecha sin ningún cambio de forma
a una velocidad de 40 m s .
(a) Hacer un dibujo claro que muestre el modo en que la velocidad
transversal de la cuerda varı́a con la distancia sobre la misma en el instante
en que el pulso está en la posición indicada.
(b) ¿Cuál es la velocidad máxima transversal de la cuerda (apro-
ximadamente)?

197
Figura 83: Problema 7.12

(c) Si la masa total de la cuerda es de 2 Kg, ¿Cuál es la tensión T


sobre ella?
(d) Escribir una ecuación para y(x, t), que describa numericamente
las ondas sinusoidales de longitud de onda 5 m y amplitud 0.2 m, que se
mueven hacia la izquierda (es decir, en el sentido negativo de la x) sobre
una cuerda muy larga hecha del mismo material y bajo la misma tensión
que anteriormente.

Solución

(a) Como se observa en la figura 84, las lı́neas curvas AB y EF son

Figura 84: Problema 7.12

cóncavas hacia arriba, y la lı́nea curva CD es cóncava hacia abajo. Esto


implica para la región a la izquierda de la ordenada en donde la velocidad
va hacia abajo, un aumento por unidad de longitud de la magnitud de esta
y para la región a la derecha de la ordenada un aumento por unidad de
longitud de la velocidad que va hacia arriba. Se ve también en la figura 84,
que la lı́nea curva CD es cóncava hacaia abajo, lo que indica decrecimiento
por unidad de longitud en la velocidad que va hacia abajo hacia el valor
medio de la curva CD, donde hay una lı́nea horizontal LH que indica

198
que la velocidad allı́ es nula y decrecimiento por unidad de longitud de la
velocidad que va hacia arriba a la derecha de la ordenada. También hay
dos lı́neas L1 y L2 con pendientes positiva y negativa respectivamente que
indican gradientes positivos y negativos relacionados con las velocidades
hacia abajo y hacia arriba respectivamente. Con todo esto la gráfica de
las velocidades transversales es (ver figura 85):

Figura 85: Problema 7.12

(b) Como lleva una velocidad de 40 m s , entonces recorre la distancia de


1
1.0 m en un tiempo de ts = 40 s que se llamará tiempo de subida, porque
es el tiempo en que un punto de cuerda va de la parte m as baja a la más
alta. Usando este tiempo la velocidad transversal es aproximadamente (es
un valor aproximado porque la distancia horizontal no se puede definir
exactamente):
0.1 m
vy ' 1 = 4
40
s
2
(c) Con la masa total se calcula la densidad lineal de masa, µ = 100 =
1 kg
50 m . Entonces,
 
2 2 1 1600
T = v µ → T = 40 = = 32 N
50 50

(d) Con una longitud de onda de 5 m, el número de onda es k = 2π


5
rad
m .
m
Además con una velocidad de 40 s entonces v = λν y entonces 40 = 5ν
donde la frecuencia lineal es ν = 8 Hz y ası́ ω = 2πν = 16π
 
2π h x i
y(x, t) = 0.2 sin x + 16πt = 0.2 sin 2π + 8t
5 5

13. Unn pulso que se mueve sobre una cuerda tensa viene descrito por la
ecuación siguiente:
b3
y(x, t) = 2
b + (2x − ut)2

a)Dibujar el gráfico de y en función de x para t = 0.


b) ¿Cuáles son la velocidad del pulso y su sentido de movimiento?

199
c) La velocidad transversal de un punto dado de una cuerda se
define por
∂y
vy =
∂t
calcular vy en función de x para el instante t = 0, y demostrar por medio
de un dibujo lo que esto nos dice sobre el movimiento del pulso durante
un tiempo corto ∆t.

Solución

(a) Los diferentes gráficos del pulso son:

Figura 86: Problema 7.13 Vista frontal del pulso y(x, 0)

En la gráfica 86 se observa la forma del pulso en t = 0 s en el frente de la


gráfica. En este frente la lı́nea de gráfica está dada por:

23 8 2
y(x, 0) = = =
22 + 4x2 4 + 4x2 1 + x2

En la gráfica 87 se observa la vista superior del pulso donde se puede ver


que no es paralelo a cualquiera de los ejes del plano horizontal y que por
tanto la lı́nea de gráfica del Plano yx en t = 0 s no es perpendicular a la
propagación de este.

En la gráfica 88 se observa la forma tridimensional del pulso y(x, t) =

200
Figura 87: Problema 7.13 Vista superior del pulso

23
22 +(2x−10t)2 cuyo eje longitudinal forma un ángulo con el eje x de 11.3o .

(b) La velocidad del pulso está relacionada con la función que describe
la propagación de este. La función es

f (x, t) = 2x − ut

Esta función se mantiene constante y esto es lo que hace que la forma del
pulso no varı́e. Al hallar la diferencial de esta resulta

df (x, t) = 2dx − udt = 0 → 2dx = udt

Hallando la variación temporal, se halla la velocidad de fase o de propa-


gación del pulso:
dx dx u
2 =u → = vp =
dt dt 2
resultando positiva; esto es, se mueve en sentido positivo de las x.

(c) La velocidad transversal del pulso es

b3
 
∂y(x, t) ∂
vy (x, t) = =
∂t ∂t b2 + (2x − ut)2
3
2b (2x − ut)
=− 2
[b + (2x − ut)2 ]
2

201
Figura 88: Problema 7.13 Vista tridimensional del pulso

en t = 0 s,
2b3 (2x) 4b3 x
vy (x, 0) = − 2 =− 2
[b2 + (2x)2 ] (b2 + 4x2 )
Su gráfica es en los tiempos sucesivos t = 0.00, 0.05, . . . , 0.25 s es:

Figura 89: Problema 7.13. Gráfica de la velocidad en diferentes tiempos para


observar la propagación del pulso.

donde puede analizarse que la forma del pulso no cambia y que el movi-
miento es hacia las x positivas, como ya se demostró.
14. Un lazo cerrado de una cuerda uniforme se hace girar rápidamente a cierta

202
velocidad constante ω. La masa de la cuerda es M y radio es R. Como
resultado de su rotación se produce una tensión ”T ” circunferencialmente
en la cuerda.
(a) Considerando la aceleración centrı́peta instantánea de un pe-
queño segmento de la cuerda, demostrar que la tensión debe ser igual a
M ω2 R
2π .
(b) Se deforma repentinamente la cuerda en un punto, haciendo
que aparezca una protuberancia como se ve en el diagrama. Demostrar
que esto podrı́a producir una distorsión en la cuerda que permaneciese
estacionaria respecto al laboratorio, sin tener en cuenta los valores par-
ticulares de M , ω y R. Pero ¿no sucerı́a algo más? (Recuérdese que los
puntos en una cuerda pueden moverse en los dos sentidos.)

Figura 90: Problema 7.14

Solución

(a) Debido a que es una cuerda, su tensión está dada por:

T = v2 µ
M
donde la densidad lineal de masa es µ = 2πR ; por tanto

M v2
T =
2πR
y debido a que sufre una tensión ”T ” circunferencialmente en la cuerda con
aceleración centrı́peta, la velocidad está definida por v = ωR; resultando:
M ω 2 R2 M ω2 R
T = =
2πR 2π
(b) Una deformación repentina en una cuerda recta genera un pulso que
obedece una función f (x ± vt). En una cuerda que forma un circunferencia
la función es f (S ± ωRt), donde S = θR es una longitud de la trayectoria
curva. Inicialmente al trabajar la ecuación de onda ya se ha demostra-
do que la solución de la ecuación de onda unidimensional produce ondas
viajeras en dos sentidos; por tanto la cuerda aquı́ siguue siendo unidimen-
sional a pesar que se cierre sobre si misma. la ecuación que gobierna un

203
pulso unidimensional, es solución de la ecuación de onda unidimensional
cumpliendo sus caracterı́sticas fı́sicas. Podrı́a suceder que se presenten si-
multaneamente dos pulsos que viaje en la misma cuerda y se superpongan
periodicamente.
15. Dos pulsos idénticos de amplitudes iguales pero opuestas se aproximan uno
al otro al propagarse sobre una cuerda. Cuando t = 0 están en la posición
indicada en la figura 91. Dibujar a escala la cuerda y el perfil de velocidad
de los elementos de masa de la cuerda para t = 1.0 s, t = 1.5 s y t = 2.0 s.

Figura 91: Problema 7.15

Solución

Cuando ha pasado un segundo el desplazamiento para cada pulso será


de x = vt = 10 cm. Su gráfica es (ver gráfica 92):

Figura 92: Problema 7.15

Figura 93: Problema 7.15

204
Cuando han pasado 1.5 s el desplazamiento para cada pulso será de
x = vt = 15 cm. Su gráfica es (ver gráfica 93). Se obseva claramente
que hay una superposición destructiva que anula los pulsos en la cuerda,
pero no la energı́a. Esto hace que los pulsos se sigan propagando en sentido
contrario con la misma velocidad de 10 cm s .

Finalmente cuando han pasado 2 s el desplazamiento para cada pulso


será de x = vt = 20 cm. Su gráfica es (ver gráfica 94), donde se observa
que los pulsos siguen su camino sin perder amplitud ni velocidad.

Figura 94: Problema 7.15

16. Se desea estudiar el movimiento vertical bastante rápido del contacto móvil
de un interruptor que funciona magnéticamente. Para ello, se sujeta el
contacto a un extremo (O) de un sedal horizontal de masa total 5 g (5 ×
10−3 Kg) y longitud total 12.5 m. El otro extremo del sedal para sobre
una polea pequeña y sin rozamiento, colgándose una masa de 10 Kg de
su extremo, como se ve en la figura 95. El contacto se hace funcionar de
modo que el interruptor (generalmente abierto) pasa a su posición cerrada,
permanece cerrado durante un tiempo corto y se abre de nuevo. Un poco
después la cuerda se fotografı́a usando una lámpara de destellos de alta
velocidad y que se ve deformada entre 5 y 6 mm como se ve en la figura
95. (x = 0 es el punto O en donde la cuerda está conectada al contacto.)
(a) ¿Durante cuánto tiempo estuvo el interruptor completamente
cerrado?
(b) Dibujar un gráfico del desplazamiento del contacto en función
del tiempo, tomando t = 0 como el instante en el que el contacto empezó
primeramente a moverse.
(c) ¿Cuál era la velocidad máxima del contacto? (¿Se presentó

205
Figura 95: Problema 7.16

durante el cierre o la abertura del interruptor?)


(d) ¿Para qué valor de t fue tomada la fotografı́a? (Admitir que
m
g = 10 s2 )

Solución
−3
(a) Con los datos se calcula la densidad lineal de masa µ = 5×10 12.5 =
4 × 10−4 Kgm y una tensión T = 10(9.8) = 98 N , y por tanto la velocidad
q
98 m
de fase es vp = 4×10 −4 = 494.98 s . El pulso tiene una altura máxima

de 40 cm de longitud, y por tanto el tiempo fue de


x 0.4
t= = = 0.0008 = 8 × 10−4 s
vp 494.98

(b) El gráfico es:


(c) La velocidad máxima es durante la apertura porque la pendiente es
mayor. Recorrió una distancia horizontal de 0.2 m en un tiempo de ta =
x 0.2 −4
vp = 494.98 = 0.0004 = 4 × 10 s y por tanto la velocidad de apertura
(donde recorrió una distancia de 5 mm) fue de:

5 × 10−3 m
vy = = 12.5
4 × 10−4 s
(d) El tiempo en el cual se tomó la fotografı́a debe ser posterior a la
formación del pulso y como se observa en la foto, el pulso no alcanza a
avanzar desde el origen, por tanto se toma en el instante cuando el pulso se
forma. Esto incluye el cierre, el tiempo que permanece cerrado y el tiempo

206
de la apertura del interruptor. Este tiempo es:
tf = ta + t + tc = 4 × 10−4 + 8 × 10−4 + 8 × 10−4 = 2 × 10−3 s
que es el tiempo justo cuando el pulso se forma. Con el espacio adicional
de 0.15 m a la misma velocidad de fase el tiempo es:
tf = 2 × 10−3 + 0.3 × 10−3 = 2.3 × 10−3 s

17. Se superponen en un medio las dos ondas siguientes:


y1 = A sin(5x − 10t)
y2 = A sin(4x − 9t)
en donde x está en metros y t en segundos.
(a) Escribir una ecuación para la perturbación combinada.
(b) ¿Cuál es su velocidad de grupo?
(c) ¿Cuál es la distancia entre los puntos de amplitud nula en la
perturbación combinada?

Solución

(a) La perturbación combinada es:


y = A[sin(5x − 10t) + sin(4x − 9t)]
| {z } | {z }
α+β α−β

donde sin(α+β)+sin(α−β) = 2 cos(β) sin(α); entonces (α+β)+(α−β) =


2α = (5x − 10t) + (4x − 9t) = 9x − 19t, y (α + β) − (α − β) = 2β =
(5x − 10t) − (4x − 9t) = x − t; por tanto α = 92 x − 19 1 1
2 t y β = 2 x − 2 t,
resultando:    
1 1 9 19
y = 2A cos x − t sin x− t
2 2 2 2
(b) La velocidad de grupo es la de la envolvente (esto es la de la resta de
fases):
1 1 1 1 dx m
ε= x− t → dε = dx − dt = 0 → = vx = 1
2 2 2 2 dt s
(c) La amplitud nula combinada es cuando y(x, t) = 0; esto es
   
1 1 9 19
y = 2A cos x − t sin x− t =0
2 2 2 2
pero es cero para la envolvente; por tanto para t = 0 (como la onda es
monocromática, entonces se fija el tiempo y queda la onda en un tiempo
fijo),
 
1 π x
cos x = 0 → (2n − 1) = ∴ x = (2n + 1)π
2 2 2

207
para valores sucesivos de n:

x0 = π, x1 = 3π, x2 = 5π, x3 = 7π, ...


y las diferencias entre valores sucesivos de ceros son:
∆x1,0 = 3π − π = 2π, ∆x2,1 = 5π − 3π = 2π,
∆x3,2 = 7π − 5π = 2π, ...

Por tanto la distancia entre los puntos de amplitud nula en la perturbación


combinada claramente es 2π m.

18. El movimiento de la onda de longitud corta es (. 1 cm) en agua está


controlado por la tensión superficial. La velocidad de fase de estas ondas
viene dada por:
 1
2πS 2
vp =
ρλ
en donde S es la tensión superficial y ρ es la densidad del agua.
(a) Demostrar que la velocidad de grupo para una perturbación
formada por longitudes de onda próximas a una longitud dada λ es igual
3v
a 2p .
(b) ¿Qué implica esto acerca del movimiemto observado de un
grupo de ondas que se mueven en la superficie del agua?
(c) Si el grupo se compone solamente de dos ondas, de longitud de
onda 0.99 y 1.01 cm, ¿Cuál es la distancia entre las crestas del grupo?

Solución

(a) La velocidad de fase es vp = ωk y por tanto ω = vp k; debido a que


la velocidad de grupo es vg = ∂ω
∂k , entonces

∂ ∂vp
vg = (vp k) = k + vp
∂k ∂k
pero como la velocidad de fase depende de la longitud de onda, entonces:
 
∂vp ∂λ
vg = k + vp
∂λ ∂k
 − 21     1
1 2πS 2πSρ ∂λ 2πS 2
= − 2 2 k+
2 ρλ ρ λ ∂k ρλ
 − 12     1
2πS πS ∂λ 2πS 2
=− k +
ρλ ρλ2 ∂k ρλ

208
2π 2π ∂λ
con k = λ entonces λ = k y por tanto ∂k = − 2π
k2 . Reemplazando:
− 12     1
2πS πS 2π 2πS 2
vg = − − 2 k+
ρλ ρλ2 k ρλ
 − 12      12
2πS πS 2π 2πS
= +
ρλ ρλ2 k ρλ
 − 12     1  1
2πS 2πS π 2πS 2 2πS 2  π 
= + = 1+
ρλ ρλ kλ ρλ ρλ kλ
π
pero kλ = 2π que lleva a kλ = 12 ; reemplazando en la ecuación principal,
  12     12
2πS 1 2πS 3
vg = 1+ =
ρλ 2 ρλ 2
Finalmente,
3
vg =
vp .
2
(b) Por lo general la velocidad de fase vp es mayor que la velocidad de gru-
po vg ; esto es debido a que las dos recorren el mismo espacio en el mismo
tiempo, pero las longitudes de onda de la de fase son mucho menores. Acá
se tiene como resultado que la velocidad de grupo es 1.5 veces mayor que
la de fase, y esto es debido al medio de propagación (agua en una capa
superficial) que no es muy rı́gido, y esto hace que la velocidad de per-
turbación del medio sea menor que la del grupo, formando convoluciones
superficiales que dan lugar a las olas. Parte de este comportamiento se da
por la diferencia de medios entre la capa superficial de agua; en la parte
inferior agua profunda con una densidad mucho mayor que la del aire en
la parte superior de la capa.

(c) Con λ1 = 0.99 cm y λ2 = 1.01 cm, las ecuaciones de onda en t = 0 s


son:
  

y1 = A cos x
0.99
  

y2 = A cos x
1.01
Cuya superposición es:
      
2π 2π
y = A cos x + cos x
0.99 1.01
     
2π 2π 2π 2π
= 2A cos − x cos + x
0.99 1.01 0.99 1.01
     
4π 4π
= 2A cos x cos x
99.99 0.9999

209
Para los valores máximos y = 2A, por tanto,
     
4π 4π
cos x cos x =1
99.99 0.9999
pero los máximos sucesivos no son de la envolvente, sino de la fase. Por
tanto,   

cos x =1
0.9999
para qe esto se cumpla, esto es que sea positivo se elige que cos(2π) = 1,
por tanto (usando la notación de ∆x):
 
4π 0.9999
∆x = 2π → ∆x = = 0.50 cm
0.9999 2

19. La relación entre la frecuencia ν y el número de ondas k para las ondas


de un cierto medio están indicadas en la figura. Hacer un razonamiento
cualitativo (y explicar las bases del mismo) sobre los valores relativos de
las velocidades de grupo y de fase para cualquier longitud de onda en el
intervalo representado.

Figura 96: Figura del problema 7.19

Solución

Se tiene que λ = 2π k y usando la relación de dispersión v = λν, enton-


ces la relación entre la frecuencia ν y el número de ondas k es:
2πν  v 
v= → ν= k
k 2π
donde se observa que es claro que la lı́nea de la gráfica representa la rela-
v m
ción 2π con unidades s·rad . Esto es velocidad de las ondas sobre unidad
angular. Por lo general las unidades angulares no se tienen en cuenta, por
tanto se considera como una ”velocidad”. Se observa que hay una rela-
ción lineal entre la frecuencia lineal y el número de ondas que a un valor
creciente del número de ondas se comienza a perder resultando un creci-
miento muy lento del valor de la frecuencia lineal respecto al del número

210
de ondas, esto es, llega un momento en el que el valor de la frecuencia
tiende a estabilizarse, pero el del número de ondas se incrementa a mayor
velocidad.
*************************
20. Consideremos un tubo en U de sección recta uniforme con dos brazos
verticales. Sea la lobgitud total de la columna del lı́quido l. Imaginemos
que el lı́quido está oscilando arriba y abajo de modo que en un instante
cualquiera los niveles de los brazos laterales están a ±y respecto al nivel
de equilibrio y todo el lı́quido tiene la velocidad dy
dt .
(a) Escribir una expresión para la energı́a potencial más la energı́a
cinética
q del lı́quido y a partir de ahı́ demostrar que el perı́odo de oscilación
es π 2l g.

Figura 97: Figura del problema 7.20

(b) Imaginar que puede utilizarse una serie de estos tubos para
definir una serie de crestas y valles como en una onda de agua (véase
diagrama 97). Tomando el resultado de (a) y la condición λ ≈ 2l implicada
por esta analogı́a, deducir que la velocidad de las ondas en el agua es
1

semejante a (gλ)
2
π (admitir que solo una pequeña fracción de lı́quido está
en los tubos verticales del tubo en U).
  12

(c) Utilizar el resultado exacto, v = 2π , para calcular la velo-
cidad de las ondas de longitud de onda 500 m en el océano.

Solución
 2
(a) La energı́a cinética es dK = 12 dm dy dt , y el diferencial de masa
se define mediante la densidad volumétrica de masa de la forma dm =
ρdV = ρAds, donde A es la sección transversal del tubo y ds es la lon-
gitud diferencial de la columna de agua ; entonces la energı́a cinética es
 2
dK = 12 ρAdl dy dt . La energı́a potencial gravitatoria es dU = dmgy don-
de y es la distancia que oscila y el diferencial de masa se define como en
el caso de la energı́a cinética; entonces dU = ρgAyds. La energı́a total dE

211
es:
Z l  2 Z y
1 dy
E= ρAds + ρgAyds
0 2 dt 0
 2 Z l Z y
1 dy
= ρA ds + ρgAy ds
2 dt 0 0
 2
1 dy
= ρAl + ρgAy 2
2 dt
Definiendo la ecuación para las velocidades cuadráticas,
 2
2E dy 2g 2
= + y
ρAl dt l
|{z}
ω2

se calcula la frecuencia angular,


  12   12   12
2π 2g l 2l
ω= = → τ = 2π =π
τ l 2g g
q
2l
(b) Usando la aproximación λ ≈ 2l y el resultado anterior τ = π g, se
observa que la velocidad es
r √
λ λ λ g gλ
v = λν = = q = =
τ π g 2l π λ π

  12
gλ m
(c) Con el valor v = 2π y con g = 9.8 s2 , se tiene que:
  12
9.8(500) m
v= = 27.906
2π s

21. Consideremos un sistema de N osciladores acoplados (N  1), cada uno


de ellos separado de sus vecinos más próximos por una distancia l.
(a) Hallar la longitud de onda y la frecuencia del modo n de osci-
lación.
(b) Hallar las velocidades de fase y de grupo para este modo.
¿Cuáles son estas para los casos en que n  N y n = N + 1?

Solución

(a) Se recuerda que la ecuación de movimiento para la partı́cula j-ésima


en un sistema de N osciladores acoplados es
d 2 yj
+ 2ω02 yj − ω02 (yj+1 + yj−1 ) = 0
dt2

212
que después de aceptar una solución oscilatoria yj = Aj cos(ωt), para la
partı́cula j-ésima queda:

(2ω02 − ω 2 )Aj − ω02 (Aj−1 + Aj+1 ) = 0

donde:
Aj−1 + Aj+1 2ω02 − ω 2
=
Aj ω02
resultando una dependencia de las amplitudes con un ángulo, pues la rela-
ción de amplitudes (que es constante) es igual a la relación de frecuencias
angulares (que es constante); esto lleva a proponer Aj = B sin(jα) que al
reemplazarlo resulta
Aj−1 + Aj+1
= 2 cos(α)
Aj

Usando Aj = B sin(jα) e imponiendo las condiciones de frontera fijas en



jnπ
ambos extremos, se llega a α = Nnπ +1 , y por tanto Aj = B sin N +1 ,
resultando:
2ω02 − ωn2
 

= 2 cos
ω02 N +1
que lleva a la frecuencia del modo n:
 

ωn = 2ω0 sin
2(N + 1)

Cuando N es muy grande (lo cual se


qhace para analizar un sistema conti-
T
nuo), se observa que (usando ω0 = ml ):
  r  
nπ T nπ
ωn = 2ω0 sin ≈2
2(N + 1) ml 2(N + 1)
r    s
T nπl nπ T
≈ =
ml (N + 1)l (N + 1)l µ

2πv
pero 2πν = λ ; entonces
s  s
2πv 2π T nπ T
= =
λn λn µ (N + 1)l µ

por tanto,

2 n 2(N + 1)l
= → λn =
λn (N + 1)l n

213
ωn
(b) La velocidad de fase está definida por ecuación de dispersión vp,n = kn ;
por tanto
ωn λn ωn (N + 1)l
vp,n = =
2π nπ
La velocidad de grupo vg,n está dada por la superposición de ondas (siste-
ma continuo) que serı́a el resultado de una onda que viaja en un sentido,
se refleja y regresa como una onda espejo para superponerse con las que
van. Entonces en este caso se usa el modo supremo; esto es cuando se
relacionan las amplitudes sucesivas para n = N :
h i

Aj sin (N +1)
Modo supremo = =− h i
Aj+1 sin (j+1)π
(N +1)

desde el punto de vista de la superposición de ondas, se relacionan los


modos sucesivos (n y n+1) debido a que es un sistema continuo resultando
la fecuencia angular
    
1 (n + 1)π nπ
ωg,n = 2ω0 sin − 2ω0 sin
2 2(N + 1) 2(N + 1)

donde se observa que la velocidad de grupo es


    
ω0 (n + 1)π nπ
vg,n = λn+1 sin − λn sin
2π 2(N + 1) 2(N + 1)

Reemplazando la longitud de onda λ anteriormente hallada λn = 2(Nn+1)l :


    
ω0 2(N + 1)l (n + 1)π 2(N + 1)l nπ
vg,n = sin − sin
2π n+1 2(N + 1) n 2(N + 1)

se llega a la velocidad de grupo:


    
ω0 (N + 1)l 1 (n + 1)π 1 nπ
vg,n = sin − sin
π n+1 2(N + 1) n 2(N + 1)

Cada término de la frecuencia angular tiene su propia longitud de onda


porque esta depende del modo de oscilación, como se observa en el ı́tem
(a).
22. Se nos da el problema de analizar la dinámica de una linea de coches que
se mueven sobre una carretera de una sola vı́a. Un modo de enfoque a este
problema es admitir que la linea de coches se comporta como un grupo
de osciladores acoplados. ¿Cómo se plantearı́a este problema de un modo
tratable? Hacer bastantes hipótesis.

Solución

214
La dinámica se analiza con la ecuación

Fneta = Facople

donde no hay fuerza de restitución debido a que los cuerpos no están fijos
en los extremos. Para tres cuerpos acoplados con amplitudes no nulas (ver
problema (5.9)):

−m1 ω 2 + k
    
−k 0 A1 0
 −k −m2 ω 2 + 2k −k  A2  = 0
0 −k −m3 ω 2 + k A3 0

Cuya solución es:

ω12 = 0
p
km1 m2 + 2km1 m3 + km2 m3 − k m21 m22 − 2m1 m22 m3 + 4m21 m23 + m22 m23
ω22 =
2m1 m2 m3
p
km1 m2 + 2km1 m3 + km2 m3 + k m21 m22 − 2m1 m22 m3 + 4m21 m23 + m22 m23
ω32 =
2m1 m2 m3
con cuatro cuerpos las ecuaciones para las frecuencias de oscilación de
tornan más extensas. Si se suponen vehı́culos de masas iguales, entonces:

mω12 = 0, mω22 = k, mω32 = 3k

ω1 = 0 → → →
r
k
ω2 = → · ←
m
r
3k
ω3 = → → ←
m
Para un sistema de cuatro cuerpos de igual masa m,

−mω 2 + k
    
−k 0 0 A1 0
2

 −k −mω + 2k −k 0  A2  0
  =  
 0 −k −mω 2 + 2k −k  A3  0
0 0 −k −mω 2 + k A4 0

los modos de oscilación del sistema son:


√ √
mω12 = 0, mω22 = 2k, mω32 = (2 − 2)k, mω42 = (2 + 2)k

215
las formas de oscilar los cuerpos acoplados en cada modo son:

ω1 = 0 → → → →
r
2k
ω2 = → → ← ←
m
s √
(2 − 2)k
ω3 = → ← ← →
m
s √
(2 + 2)k
ω4 = → ← → ←
m

Para cinco cuerpos las interacciones en cada modo de oscilación están


dadas por:
√ ! √ !
2 2 3− 5 2 5− 5
mω1 = 0, mω2 = k, mω3 = k,
2 2
√ ! √ !
2 3+ 2 2 5+ 5
mω4 = k, mω5 = k
2 2

y sus movimientos oscilatorios son:

ω1 = 0 → → → → →
s √
(3 − 5)k
ω2 = → → · ← ←
2m
s √
(5 − 5)k
ω3 = → ← · → ←
2m
s √
(3 + 2)k
ω4 = → · ← · →
2m
s √
(5 + 5)k
ω5 = → ← → ← →
2m

Finalmente para seis cuerpos acoplados las interacciones entre los cuerpos
en cada modo de oscilación son:

mω12 = 0, mω22 = k, mω32 = 2k, mω42 = 3k,


√ ! √ !
2− 3 2+ 3
mω52 = k, 2
mω6 = k
2 2

216
y sus movimientos oscilatorios son:

ω1 = 0 → → → → → →
r
k
ω2 = → → → ← ← ←
m
r
2k
ω3 = → → ← ← → →
m
r
3k
ω4 = → ← ← ← → ←
m
s √
(2 − 3)k
ω5 = → ← ← → → ←
2m
s √
(2 + 3)k
ω6 = → ← → ← → ←
2m

Como se observan en todos los casos, hay un estado del sistema donde la
frecuencia angular es nula, ω1 = 0, y esto significa que solo hay traslación
lo que es muy improbable en una linea de automóviles a no ser que sean
conducidos de forma automática (conductor no humano). Por lo general
la dinámica de una linea de carros está gobernada por aceleraciones y
desceleraciones intermitentes que se asemejan a oscilaciones. El primer
automóvil es que que lleva el liderazgo del movimiento, pero aún ası́ este
descelera cada cierto tiempo, lo que hace que se genera una oscilación
hacia atrás. Hay muchas posibilidades, que se pueden sacar de cada uno
de los modos de oscilación del sistema. El lector puede argumentar la(s)
que más desee.
23. Se mueve transversalmente el extremo de una cuerda tirante a velocidad
constante uy durante un tiempo τ , y se vuelve a llevar a su punto de par-
tida con velocidad −uy durante el intervalo siguiente τ . Como resultado
se produce un pulso triangular sobre la cuerda que se mueve a lo largo de
ella con velocidad v. Calcular las energı́as cinética y potencial asociadas
con el pulso y demostrar que su suma es igual al trabajo total realizado
por la fuerza transversal que ha de aplicarse al extremo de la cuerda.

Solución

La gráfica del pulso está dada en la figura 98. Se puede observar en esta
que la función de onda es nula (y(x, t) = 0) en los intervalos −∞ < x < −d
y d < x < ∞ y en el intervalo −d ≤ x ≤ d no es nula. Este intervalo se
divide en dos −d ≤ x ≤ 0 y 0 ≤ x ≤ d. A medida que el pulso avanza
el punto −d se acerca al origen con una velocidad de fase v; entonces la
distancia disminuye como d − (x − vt) y el punto d aumenta su distancia
desde el origen como d + (x − vt). La pendiente de la función de onda es
justamente la tangente; esto es: yd0 y como es desde el origen el intercepto

217
es nulo. La funciones de onda son:

Figura 98: Figura del problema 7.23

(
y0
d [d − (x − vt)] para: − d ≤ x ≤ 0,
y(x, t) = y0
d [d + (x − vt)] para: 0 ≤ x ≤ d

Entonces las velocidades están dadas por:

∂y− (x, t) ∂ n y0 o y
0
= [d − (x − vt)] = v
∂t ∂t d d
∂y+ (x, t) ∂ n y0 o y0
= [d + (x − vt)] = − v
∂t ∂t d d
donde,
2 2
y02 2
 
∂y− (x, t) ∂y+ (x, t)
= = v
∂t ∂t d2
y por tanto las energı́as cinéticas diferenciales son:

1 y02 2
dK− = dK+ = µ v dx
2 d2
Adicionalmente se observa que para que el pulso se propague la cuerda
debe tener una tensión T = v 2 µ, con la longitud de la cuerda dada por:
h i1   2  21
2 2 2 2 ∂y 2
ds = (dx) + (dy) = (dx) + ∂x (dx) , que al factorizar se
  1
 2 2
∂y
obtiene: ds = dx 1 + ∂x . Aproximando con el Binomio de Newton
  2 
∂y
se llega a: ds = dx 1 + 12 ∂x , y ası́ la diferencia de longitudes que
 2
∂y
determina la elongación es: ds − dx = 12 ∂x dx, y por tanto las energı́as
potenciales diferenciales son:
 2  2
1 ∂y 1 2 ∂y
dU− = dU+ = T dx = v µ dx
2 ∂x 2 ∂x

218
calculando las derivadas,
∂y− ∂ n y0 o y0 ∂ y0
= [d − (x − vt)] = (−x + vt) = −
∂x ∂x d d ∂x d
∂y+ ∂ n y0 o y0 ∂ y0
= [d + (x − vt)] = (x − vt) =
∂x ∂x d d ∂x d
donde,
2 2
y02
 
∂y− ∂y+
= =
∂x ∂x d2
y por tanto,
1 y02 2
dU− = dU+ = µ v dx
2 d2
verificándose que

1 y02 2
dK− = dK+ = dU− = dU+ = µ v dx
2 d2
El diferencial del trabajo es dW = T dl, donde dl = ds−dx es la elongación.
Por tanto
 2
1 ∂y 1  y0 2 1 y2
dW = T dx = T dx = µ 02 v 2 dx
2 ∂x 2 d 2 d

24. Consideremos una onda sinusoidal longitudinal ξ = ξ0 cos[2πk(x−vt)] que


se mueve a lo largo de una varilla de densidad de masa ρ, área de la sección
recta S y módulo de Young Y . Demostrar que si la tensión en una vari-
lla se debe solamente a la presencia de una onda, la densidad de energı́a
 2  2
cinética es 12 ρS ∂ξ
∂t , y la densidad de energı́a potencial es 1
2 Y S ∂ξ
∂x .
Demostrar ası́ que la energı́a cinética por longitud de onda y la energı́a
1 2
potencial por longitud de onda son ambas
  iguales a 4 (ρSλ) u0 , siendo u0
∂ξ
la velocidad máxima de la partı́cula ∂t .

Solución

En una varilla con forma de cilı́ndro recto, se orienta paralela al eje x,


y en una cooredenada cualquiera se especifican las secciones transversales
en los puntos x y x + dx. Cuando se realiza la propagación longitudinal de
la onda, esta se deforma una magnirud x+ξ en el punto x y x+dx+ξ +dξ
en el punto x+dx. Esta deformación es una alternancia entre la elongación
y compresión de la varilla en esa región, y por tanto debe actuar sobre
esta una fuerza neta de la forma:

dFN = −dFx + dFx+dx

219
 
σn
La tensión en un punto x1 es usando el módulo de Young: Y = ∆L ,
L0

σn F ∂ξ ∂ξ
Y = ∂ξ
→ σn = =Y ∴ Fx1 = Y S
∂x
S ∂x ∂x
pero como la fuerza depende de la distancia, entonces es necesario adicio-
nar una segunda variación de la función de onda para hallar la fuerza en
otro punto (x2 ) donde la elongación aumenta; esta está dada por :
∂2ξ 1 ∂3ξ
 
∂ξ 2
Fx 2 = Y S + ∆x + (∆x) + . . .
∂x ∂x2 2 ∂x3
que solo soporta segundas derivadas para establecer las ecuaciones de mo-
vimiento. Por tanto,
∂2ξ ∂2ξ
 
∂ξ
Fx2 = Y S + 2
∆x = Fx1 + Y S 2 ∆x
∂x ∂x ∂x
Es importante recordar que la fuerza neta es la diferencia entre las fuerzas
Fx1 y Fx2 ,
∂ 2 ξ(x, t)
FN = Y S ∆x
∂x2
y al usar la segunda Ley de Newton con masa constante,
∂ 2 ξ(x, t) ∂p ∂ξ(x, t)
FN = ρS ∆x = → p = ρS∆x
∂t2 ∂t ∂t
entonces,
2 2
p2 ρ2 (S∆x)2
 
∂ξ ρS∆x ∂ξ
K= = =
2m 2m ∂t 2 ∂t
donde la densidad de energı́a cinética ρK es
 2
1 ∂ξ
ρK = ρS .
2 ∂t
Con la fuerza neta como una fuerza de restitución, donde la fiuerza de-
pende solo de la función de onda ξ(x, t):
∂2ξ ∂U ∂2ξ ∂U ∂ξ
−YS 2
∆x = − → Y S 2 ∆x =
∂x ∂ξ ∂x ∂ξ ∂x
  2  2
∂U ∂ξ ∂ ξ 1 ∂ ∂ξ
=YS ∆x = Y S ∆x
∂ξ ∂x ∂x2 2 ∂x ∂x
y entonces,  
 2 
∂U ∂ 1 ∂ξ 
=  YS  ∆x
∂ξ ∂x  2 ∂x 
| {z }
ρU

220
Finalmente la densidad de energı́a potencial es
 2
1 ∂ξ
ρU = Y S
2 ∂x

Usando ξ = ξ0 cos[2πk(x − vt)] (donde k = λ−1 ), se observa que


 2
∂ξ
= 4π 2 ξ02 k 2 v 2 sin2 [2πk(x − vt)]
∂t
 2  2
∂ξ 2 2 2 2 1 ∂ξ
= 4π ξ0 k sin [2πk(x − vt)] = 2
∂x v ∂t
y entonces,
1
ρK = (ρS)u20
2
y,  
1 1
ρU = Y S u20
2 v2
Y Y
con v 2 = ρ → ρ= v2 , entonces

1
ρU = (ρS)u20 = ρK
2
Las energı́as son:
1
U =K= (ρS)u20 ∆x
2
y debido a que en intervalo de longitud es para una onda, este se identifica
con la longitud de onda, y como las energı́as son proporcionales al cuadrado
del coseno, entonces los valores máximo y mı́nimo están entre cero y su
valor máximo; esto es media longitud de onda. Esto lleva a definir el valor
de cada energı́a con el valor máximo que se logra en ∆x = λ2 . Por tanto
las energı́as quedan como:
1
U =K= (ρSλ)u20 (ρSλ tiene unidades de masa)
4
lo que da la energı́a total como:
1
E = 2U = 2K = (ρSλ)u20
2

25. Comprobar que la ecuación de onda para una onda con simetrı́a esférica
[Ec. (7.44)]

∂ 2 Ψ 2 ∂Ψ 1 ∂2Ψ
(simetrı́a esférica) + = (7.44)
∂r2 r ∂r v 2 ∂t2
se ve satisfecha por las ondas armónicas simples cuya amplitud disminuye
inversamente con r.

221
Solución

La función de onda cuya amplitud disminuye inversamente con r es:


A
Ψ(r, t) = cos(kr − ωt)
r
derivando esta función respecto a r, resulta:
∂Ψ A Ak
= − 2 cos(kr − ωt) − sin(kr − ωt)
∂r r r
∂2Ψ 2A Ak Ak
= 3 cos(kr − ωt) + 2 sin(kr − ωt) + 2 sin(kr − ωt)
∂r2 r r r
Ak 2
− cos(kr − ωt)
r 
A 2 2 2Ak
= − k cos(kr − ωt) + 2 sin(kr − ωt)
r r2 r

Derivando temporalmente,
∂Ψ Aω
=− sin(kr − ωt)
∂t r
∂2Ψ Aω 2
2
=− cos(kr − ωt)
∂t r
Entonces
2 ∂Ψ 2A 2Ak
= − 3 cos(kr − ωt) − 2 sin(kr − ωt)
r ∂r r r
y
1 ∂2Ψ Aω 2
= − cos(kr − ωt)
v 2 ∂t2 v2 r
La ecuación entonces queda como:
 
A 2 2 2Ak 2A
2
− k cos(kr − ωt) + 2 sin(kr − ωt) − 3 cos(kr − ωt)
r r r r
2Ak Aω 2
− 2 sin(kr − ωt) = − 2 cos(kr − ωt)
  r v r
2 2 2k 2
− k cos(kr − ωt) + sin(kr − ωt) − 2 cos(kr − ωt)
r2 r r
2k ω2
− sin(kr − ωt) = − 2 cos(kr − ωt)
r v
simplificando,

ω2
−k 2 cos(kr − ωt) = − cos(kr − ωt)
v2

222
verificando la ecuación de dispersión v = ωk , y por tanto comprobando que
la ecuación de onda para una onda con simetrı́a esférica es satisfecha por
las ondas armónicas simples cuya amplitud disminuye inversamente con
r.

8. Efectos debido a los lı́mites e interferencias


1. Se conectan juntas dos cuerdas de tensión T y densidades máximas µ1
y µ2 . Consideremos una onda móvil incidente sobre su lı́mite. Hallar el
cociente entre la amplitud reflejada y la amplitud incidente y el cociente
entre la amplitud transmitida y la amplitud incidente en los casos en que
µ2
µ1 = 0.00, 0.25, 1.00, 4.00, ∞.

Solución
   
La amplitud incidente es f1 x − vx1 , la reflejada es g1 x + vx1 y la trans-
 
mitida es f2 x − vx2 , cuyas relaciones entre ellas se realizan cuando se
analiza una posición x = x0 fija, donde se ubica la frontera entre los dos
medios, se verifican las condiciones de continuidad,
     
x0 x0 x0
f1 x0 − + g1 x0 + = f2 x0 −
v1 v1 v2
y
     
x0 x0 x0
1 1 x0 −
df v1 1 1 x0 +
dg v1 1 2 x0 −
df v2
− + =−
v1 dt v1 dt v2 dt
donde solucionando se llega a (ver texto)
   
x0 v2 − v1 x0
g1 x0 + = f1 x0 −
v1 v1 + v2 v1
   
x0 2v2 x0
f2 x0 − = f1 x0 −
v1 v1 + v2 v1

Las relaciones entre las amplitudes son


 
g1 x0 + xv10 g v − v1
 = 1 = 2
x0
f1 x0 − v1 f1 v1 + v2

y
 
x0
f2 x0 − v1 f2 2v2
 = =
f1 x0 − x0 f1 v1 + v2
v1

223
Las velocidades están dadas por v12 = µT1 y v22 = µT2 ; debido a que la
tensión es la misma en las cuerdas, se llega a


 a) 0.00 → v2  v1
v2

b) 0.25 → v12 = 14 ∴ v1 = 1, v2 = 2



2

µ2 v  2
= 12 = c) 1.00 → v2 = v1
µ1 v2  v2
d) 4.00 → v12 = 41 ∴ v1 = 2, v2 = 1




 2

e) ∞ → v v 1 2

Ası́ para el caso a):

g1 v2 − v1 v2 f2 2v2 2v2
= = =1 y = = =2
f1 v1 + v2 v2 f1 v1 + v2 v2

para el caso b):

g1 v2 − v1 2−1 1 f2 2v2 4
= = = y = =
f1 v1 + v2 1+2 3 f1 v1 + v2 3

para el caso c):

g1 v2 − v1 v1 − v1 f2 2v2 2v1
= = =0 y = = =1
f1 v1 + v2 v1 + v1 f1 v1 + v2 v1 + v1

para el caso d):

g1 v2 − v1 1−2 1 f2 2v2 2
= = =− y = =
f1 v1 + v2 2+1 3 f1 v1 + v2 3

para el caso e):

g1 v2 − v1 −v1 f2 2v2 v2
= = = −1 y = = =0
f1 v1 + v2 v1 f1 v1 + v2 v1

2. Se conectan juntas dos cuerdas de tensión T y densidades másicas µ1 y


µ2 . Consideremos una onda móvil incidente en el lı́mite entre ambas. De-
mostrar que el flujo de energı́a de la onda reflejada más el flujo de energı́a
de la onda transmitida es igual al flujo de energı́a de la onda incidente.
[Indicación: el flujo energético de una onda (la densidad de energı́a mul-
2
tiplicada por la velocidad de la onda) es proporcional a Av , siendo A la
amplitud y v la velocidad de la onda]

Solución

224
Las energı́as involucradas en este proceso son cinéticas, y por tanto:
 2  2
Kincidente 1 df1 1 µ1 df1
ρincidente = = µ1 =
L1 2 dt 2 v12 dt
 2  2
Kreflejada 1 dg1 1 µ1 dg1
ρreflejada = = µ1 =
L1 2 dt 2 v12 dt
 2  2
Ktransmitida 1 df2 1 µ2 df2
ρtransmitida = = µ2 =
L2 2 dt 2 v22 dt
Los flujos de energı́a son (la onda reflejada tiene sentido contrario a las
otras dos ondas):
 2  2
1 µ1 df1 1 µ1 df1
φincidente = v 1 =
2 v12 dt 2 v1 dt
 2  2
1 µ1 dg1 1 µ1 dg1
φreflejada = (−v1 ) = −
2 v12 dt 2 v1 dt
 2  2
1 µ2 df2 1 µ2 df2
φtransmitida = 2 v2 =
2 v2 dt 2 v2 dt
pero f1 + g1 = f + 2, por tanto
 2
1 µ2 df1 dg1
φtransmitida = +
2 v2 dt dt
" 2  2   #
1 µ2 df1 dg1 df1 dg1
= + +2
2 v2 dt dt dt dt
 2  2   
1 µ2 df1 1 µ2 dg1 µ2 df1 dg1
= + +
2 v2 dt 2 v2 dt v2 dt dt
donde no existe un término de interferencia; por tanto,
 2  2
1 µ2 df1 1 µ2 dg1
φtransmitida = +
2 v2 dt 2 v2 dt
pero
 2
v1 df1
2 φincidente =
µ1 dt
 2
v1 dg1
−2 φreflejada =
µ1 dt
 2
v2 df2
2 φtransmitida =
µ2 dt
entonces
1 µ2 v1 1 µ2 v1
φtransmitida = 2 φincidente − 2 φreflejada
2 v2 µ1 2 v2 µ1

225
y por tanto,

µ2 v1 φincidente = v2 µ1 φtransmitida + µ2 v1 φreflejada

dividiendo miembro a miembro entre µ1 µ2 se llega a


v1 v2 v1
φincidente = φtransmitida + φreflejada
µ1 µ2 µ1
que demuestra lo pedido.
3. Consideremos el circuito mostrado en la figura 99. Calcular el valor de la
resistencia X para obtener la máxima disipación de potencia en ella.

Figura 99: Figura problema 8.3

Solución

Como las dos resistencia están en serie, entonces la resistencia equiva-


lente está dada por: Req = X + R y por tanto la potencia del circuito
2
es: P = RVeq . debido a que el voltaje es constante ası́ como R, entonces el
valor de la potencia depende de X que está en el denominador; esto es
V2
P =
R+X
Con lo que la potencia disipada es nula, porque esta es constante como se
observa en la ecuación. Si por el contrario se supone un voltaje de la forma
V = V0 eiωt y una carga q = q0 eiωt , donde I = dq dt = iωq0 e
iωt
. Usando la
ley de voltajes de Kirchhoff,

V = I(R + X)

Con la potencia dada por

P = I 2 (R + X) = −ω 2 q02 ei2ωt (R + X)

La disipación de la potencia está dada por


dP
= −i2ω 3 q02 ei2ωt (R + X)
dt

226
y su valor máximo es:
d2 P
= −i4ω 4 q02 ei2ωt (R + X) = 0
dt2
que solo es posible para X = −R; pero como una resistencia con valor
negativo no es posible, entonces se concluye que para obtener la máxima
disipación de potencia en el circuito se debe cumplir que X = R.
4. Consideremos el circuito indicado en la figura. ¿Qué valor de ω produce

Figura 100: Figura problema 8.4

la máxima disipación de potencia en la resistencia R? (Indicación: con-


sidérese la impedancia del circuito.)

Solución

Este es un circuito RLC en serie con fuente alterna y por tanto con una
corriente I = I0 cos(ωt), en el cual en general se observa que hay una
resistencia R con una caı́da de potencial VR = I0 R, con una reactancia
inductiva XL = ωL con una caı́da de potencial VL = I0 XL y una reactan-
1
cia capacitiva XC = ωC con una caı́da de potencial VC = I0 XC . Como el
voltaje del inductor está adelantado π2 respecto al voltaje de la fuente y
el voltaje del capacitor está retrasado π2 respecto al voltaje de la fuente,
se forma un triángulo rectángulo donde los catetos con R y XL − XC ,
resultando la impedancia Z (con R = X como lo exige el problema),
i1
"  2 # 21
h
2 2 2 2 1
Z = X + (XL − XC ) = X + ωL −
ωC
Debido a que la potencia depende directamente de la corriente I0 , entonces
a mayor impedancia menor potencia; por tanto para que la impedancia
sea mı́nima se debe cumplir,
 2
1
ωL − =0
ωC
y entonces,
1 1 1
ωL = → ω2 = ∴ ω=√
ωC LC LC

227
5. Una onda plana de sonido en el aire incide sobre la superficie del agua con
incidencia normal. La velocidad del sonido en el aire es de unos 334 m s y
la velocidad en el agua es de 1480 m
s .
[Las condiciones lı́mites apropiadas en el caso de ondas longitu-
dinales son la continuidad del desplazamento de una onda y la presión
∂ξ
de la onda. La última viene dada por K ∂x , en donde K es el módulo de

∆ξ
compresibilidad del medio. Esto se deduce de ∆P = −K ∆V V = −K ∆x .
  12
Como la velocidad de onda v viene dada por K ρ , los coeficientes de
reflexión y transmisión son expresables en términos de ρ y v solamente.]
(a) ¿Cuál es la amplitud de la onda sonora que entra en el agua,
expresada como una fracción de la amplitud de la onda incidente?
(b) ¿Qué fracción del flujo de energı́a incidente entra en el agua?

Solución

(a) En la interface se tiene que (a=aire, w=agua)

∆xa = ∆xw = ∆x

y entonces,
∆ξa ∆ξw
Ka = Kw
∆x ∆x
2
pero K = v 2 ρ; por tanto va2 ρa ∆ξa = vw ρw ∆ξw . La relación entre ampli-
tudes está dada por:
∆ξa v 2 ρw
= w2
∆ξw va ρ a
donde usando la densidad de aire de ρa = 1.23 Kg m3 y la densidad de agua
de ρw = 997 Kg
m3 se llega a la relación de amplitudes ξ,

∆ξa 14802 (1.23)


= = 0.024
∆ξw 3342 (997)

(b) La fracción del flujo de energı́a


6. (a) Puede observarse que las ondas en el agua avanzan hacia la costa
teniendo sus frentes de ondas casi paralelos a la misma, independientemen-
te de la dirección del viento. Observando el hecho de que la velocidad de
las ondas en el agua disminuye cuando disminuye la profundidad del agua,
utilizar el principio de Huygens para explicar este fenómeno.
(b) Para hacer más especı́fico el análisis de (a), admitir que las
ondas inicialmente moviéndose en dirección x entran en una región en
la que su velocidad v tiene una variación sistemática con la distancia y
perpendicular a la dirección de movimiento. (Por ejemplo, x podrı́a ser la

228
dirección paralela a la costa e y serı́a entonces la dirección perpendicular
a la costa.) Demostrar que la dirección de las ondas empezán a seguir el
arco de la circunferencia de radio R tal que
v
R= dv
dy

Solución

(a) Para una costa y a extensión del mar, si la propagación de las on-
das aún si fuera circular, la fracción que llega a la costa es un arco que se
aproxima a una lı́nea paralela a la costa. y ese frente de onda tiene unos
puntos que son foco para la propagación (ver figura (101)). En la figura se

Figura 101: Figura problema 8.6 (vista superior)

observa que el frente de onda AB va avanzando a velocidad constante al

229
frente de onda CD que es paralelo a AB y a pesar que cambie de magnitud
de la velocidad, el frente de onda EF permanece paralelo a los frentes de
onda AB y CD. Se observa en el corte de la figura (102) que a medida que

Figura 102: Figura problema 8.6 (corte)

la onda viaja a la costa (la lı́nea AB es el corte de la superficie de agua),


los radios de los frentes de onda que se propagan tienen radios cada vez
menores y por tanto la velocidad de la onda que llega a la costa disminuye.

(b) La variación espacial de la velocidad, esto es el rotacional de la velo-


cidad, marca las variaciones de la velocidad de forma tal que genera una
rotación y por tanto está relacionada con la velocidad angular. Esto es
para v = v x̂, y con variación en y:
 
x̂ ŷ ẑ
∂ ∂ ∂  dv
∇ × v =  ∂x ∂y ∂z = − ẑ = ω
dy
v 0 0
donde se analiza su magnitud, debido a que es un análisis general
dv
(∇ × v)z = ωz =
dy
dv v
pero v = ωR y por tanto dy =ω= R. Finalmente,
v
R= dv
dy

230
7. (a) Como se desarrolló en el texto [Ec. (7.12)], la velocidad del
sonido en un gas es proporcional a la raı́z cuadrada de la temperatura
absoluta T . Utilizar este hecho y el resultado del problema anterior para
demostrar que cuando existe un gradiente térmico en dirección vertical (z)
las ondas sonoras se curvarán inicialmente con un radio de curvatura
2T
R= dT
dz

(b) En un dı́a calmado, la temperatura de la atmósfera resulta dis-


minuir más o menos liealmente con la altura. Dibujar la trayectoria de los
rayos de sonido emitidos desde un foco suspendido a bastante altura sobre
la atmósfera. Admitiendo que la velocidad del sonido al nivel del suelo es
de 330 m s , estimar la distancia adicional horizontal a la que un avión que
vuela a 4500 m empezará a resultar audible a un observador sobre el sue-
lo, si la temperatura disminuye 1o C por cada 150 m de aumento de altura.

Solución

(a) La ecuación (7.12):


  12
γRT
v=
M
y el resultado anterior dice que la dirección de las ondas en la costa em-
pezán a seguir el arco de la circunferencia de radio R tal que
v
R= dv
dz

por tanto la derivada de la velocidad respecto a z es


1 √ 1
γR 2 dT
 
dv γR 2 d T dz
= = √
dz M dz M 2 T
por tanto,
 12


γRT
v M T 2T
dv
=  21 = dT = dT
dT dz
dz γR dz
√ √ dz
M 2 T 2 T

donde se observa que:


2T
R= dT
dz
como se querı́a demostrar.

(b) Del resultado anterior se verifica la igualdad


v 2T 1 dv 1 dT dv 1 dT
dv
= dT
→ = → =
dz dz
v dz 2T dz v 2 T

231
entonces integrando,
1 1 1
ln v = ln T + c1 → ln v = ln T 2 + c1 ∴ v = ec1 T 2
2
m
Por otro lado la velocidad del sonido con un valor de 330 s se da a una
temperatura de 0o C; por tanto:
1
v = ec1 T 2

Pero la velocidad del sonido en el aire con un valor de 330 m s es a una


temperatura de 0o C y por tanto la ecuación anterior no lo corrobora debido
a que 330 6= ec1 (0), sin embargo esto es lo que exige el enunciado. La
temperatura varı́a con la altura de la forma:
 z   z 
T (z) = T0 − T0 = T0 1 −
150 150
Adicionalmente el enunciado dice que en el suelo la velocidad del sonido
tiene valor de 330 m o
s que es a una temperatura de 0 C, lo que no concuerda
con esta ecuación. Por esto la ecuación debe ser:
z
T (z) = −
150
y entonces,
dT (z) 1
=−
dz 150
La velocidad del sonido en el aire aumenta o disminuye 0.6 m
s por cada
grado Celsius que sube o baja la temperatura. Esto define un gradiente
de velocidad respecto a la temperatura. Por tanto,

v = 330 − 0.6T

8. (a) Un coche de policı́a, moviéndose a 90 km


h , pasa junto a un
inocente peatón mientras suena su sirena que tiene una frecuencia de
2000 Hz. ¿Cuál es el cambio total de frecuencia de la sirena según la
oirá al peatón?
(b) El coche de policı́a continúa por la calle cuyo extremo final está
bloqueado por una pared alta de ladrillo. ¿Qué oirá el peatón cuando las
reflexiones acústicas de la pared se superpongan al sonido que le llega di-
rectamente de la sirena?

Solución

(a) Con v = λν, entonces λ = νv . Por tanto


v v  u v v  u
= 1− y = 1+
νmin ν0 v νmax ν0 v

232
y por tanto,
ν0 ν0
νmin = y νmax =
1 − uv 1 + uv
 

La diferencia de frecuencias dará el cambio total de frecuencia. Por tanto,



−2 uv
ν 
0 ν0
|νmax − νmin | = ∆ν = −  = ν0

u u u u

1+ v 1− v 1+ v 1− v

usando los valores u = 90 km m m


h = 25 s , v = 330 s y ν0 = 2000 Hz, el
cambio de frecuencia es:

25

−2 330
∆ν = 2000  = 304.78 Hz

25 25

1 + 330 1 − 330

(b)

9. Los átomos de sodio térmicamente excitados emiten luz de longitud de


onda caracterı́stica λ ≈ 6000 Å. Resulta que la radiación de una fuente de
vapor de sodio no es perfectamente monocromática, sino que contiene una
distribución de longitudes de onda en el intervalo (6000 ± 0.02) Å. Si este
ensanchamiento de la lı́nea de sodio se debe predominantemente al efecto
Doppler (lo cual es cierto), determinar la temperatura aproximada de la
fuente de sodio. (Velocidad de la luz = 3 × 108 m s .)

Solución

Tomando el concepto de energı́a cinética molecular, se observa que la ve-


locidad de una molécula está dada por

v 2 = vx2 + vy2 + vz2

y que para todas las moléculas el promedio de la velocidad estará dado


por
v 2 = vx2 + vy2 + vz2
pero debido a la indistinguibilidad de las partı́culas y a la isotropı́a del
sistema se observa que
v 2 = 3vx2
También es conocido que para un gas que ejerce una presión sobre una
pared y que tiene un volumen V constante y actuando sobre un área σ, la
variación temporal de la cantidad de movimiento lineal está dada por

dpx N σmvx2
=
dt V

233
Resultando que la presión P ejercida sobre la pared es

N mvx2
P = → P V = N mvx2
V
pero ya se vió que v 2 = 3vx2 , y por tanto vx2 = 13 v 2 , que se puede reemplazar
en la ecuación anterior resultando
   
1 2 1 2
PV = Nm v =N mv
3 3
pero la ecuación de estado de los gases es P V = N RT . Por tanto,
1   1  2
N RT = N mv 2 ∴ T = mv
3 3R
La variación de la longitud de onda es (6000 ± 0.02) × 10−10 m, y la
ecuación de dispersión c = λν, donde se observa la dependencia directa
0.02
de la longitud de onda, y entonces existe un factor 6000 que hace que la
velocidad de la luz se modifique según el gas donde esté, y esta serı́a la
velocidad de las moléculas v. Por tanto esta velocidad es
0.02 0.02 m
v= c= (3 × 108 ) = 1000
6000 6000 s
La masa molar del sodio es, m = 22.989769 u = 0.0229 Kg y la constante
kg·m2
de los gases es R = 8.314472 K·mol·s2 ; reemplazando,

mv 2 0.0229(1000)2
T = = = 918.08 o K
3R 3(8.314472)

10. Lord Rayleigh, en su famoso tratado The Theory of Sound (Vol. 2 sec
298), hacı́a notar que si un observador pudiese alejarse de un recital de
música a una velocidad exactamente el doble de la del sonido, ’oirı́a la
pieza musical en el tiempo y tono correcto, pero en sentido temporal con-
trario’. Aunque esto parezca ciertamente posible, pensar en los detalles
que encierra este resultado tan curioso.

Solución

El texto del libro The Theory of Sound (Vol. 2 sec 298), se lee (ver fi-
gura 103) que se usa la razón R:
a±v
R=
a
donde a es la velocidad del sonido y v es la velocidad de la fuente sonora.
Donde se escribe que el fenómeno mencionado ocurre cuando v = 2a. Por
tanto, (
a ± 2a +3
R= =1±2=
a −1

234
Figura 103: Fragmento de la sección 298 tomado del texto The Theory of Sound
(Vol. 2)

lo que no dice mucho al respecto.


si se toma el signo positivo (el mismo sentido a la propagación sonora),
entonces R+ = 2; y si se toma el signo negativo (sentido contrario a la
propagación sonora), entonces R− = 0 que es lo que llama el fragmento del
texto, destrucción del carácter musical. Para el caso de R+ significarı́a que
las ondas sonoras y el receptor van a la misma velocidad sin adelantarse
o atrasarse, resultando un sonido único, una nota que nunca cambia (sin
tener en cuenta la resonancia en este momento).
Entonces basado en los valores de la razón R,


 0, se destruye el sonido.

1, se oye la melodı́a en el tono correcto y sin modificación.
R=
2, se oye una única nota.


3, se adelanta al sonido.

Debido a que no es un sistema relativista, las transformadas de Galileo se


verifican y son observables. Por tanto
vs|p = vs|o + vo|p
donde vs|p = velocidad del sonido respecto al piso, vs|o = velocidad del
sonido respecto al observador y vo|p = velocidad del observador respecto
al piso. Pero vo|p = 2vs|p ; entonces:
vs|p = vs|o + 2vs|p
resultando,
vs|o = −vs|p .

235
Lo que se puede interpretar como que la velocidad del sonido que llega al
observador, tiene sentido contrario a la velocidad del sonido respecto al
piso. Esto podrı́a indicar que el observador oye al revés la melodı́a pero
en el tono correcto.
11. Las ondas sonoras se mueven horizontalmente de un foco a un receptor.
Admitir que el foco tiene velocidad u, que el receptor tiene una velocidad
v (en la misma dirección) y que está soplando un viento de velocidad w
desde foco al receptor. Demostrar que, si el foco emite sonido de frecuencia
ν0 , y si la velocidad del sonido en el aire calmado es V , la frecuencia que
registra el receptor viene dada por
V −v+w
ν = ν0
V −u+w
Obsérvese que si las velocidades del foco y del receptor son iguales, la
existencia del viento no constituirá ninguna diferencia para la frecuencia
observada de la señal recibida.

Solución

Si se analiza desde el punto de vista de las transformadas de Galileo (sin


tener en cuenta el viento),

vf |p = vf |o + vo|p

donde vf |p = velocidad de la fuente respecto al piso, vf |o = velocidad de la


fuente respecto al observador y vo|p = velocidad del observador respecto
al piso. Entonces,
vf |o = vf |p − vo|p = u − v
La distancia que se ha movido la fuente es xF = ut, y la distancia que se
ha movido el observador es xO = vt. Por tanto la distancia entre la fuente
y el observador es xF O = vf |o t = (u − v)t. Incluyendo el viento que según
el enunciado tiene dos presentaciones V y w que se pueden asociar a la
velocidad del aire vA = V + w, este se presenta según lo que se acabó de
describir de dos formas; en la fuente: vA − u y en el observador vA − v.
Esats dos velocidades deben estar relacionadas con las ondas sonoras con
la ecuación de dispersión v = λν, que para la fuente es vF = λ0 ν0 , y para
el observador vO = λν. Pero al ser una onda monocromática las longitudes
de onda no varı́an, por tanto:
vF vO
λ= =
ν0 ν
Por tanto
vA − u vA − v
=
ν0 ν

236
Reemplazando vA ,
V +w−u V +w−v
=
ν0 ν
Despejando la frecuencia,
V +w−v
ν = ν0
V +w−u
que demuestra lo pedido.

12. El texto, pág. 309-310, desarrolla la teorı́a del efecto Doppler para una
fuente o foco móvil, con un observador distante en una dirección θ respecto
al movimiento del foco. Se demuestra [Ec. (8-14)] que la frecuencia recibida
viene dada por
ν0
ν(θ) = u cos(θ)
1− v
a) Demostrar que si el foco está en reposo y el observador tiene una
velocidad −u, de modo que la velocidad relativa del foco y del observador
es la misma que antes, la frecuencia detectada por el observador viene
dada por  
u cos(θ)
ν 0 (θ) = ν0 1 +
v
b) Hallar la diferencia aproximada entre ν y ν 0 . Es un tema de gran im-
portancia fı́sica el que en las ondas de la luz en el vacı́o no existe dicha
diferencia en contraste con las ondas sonoras en el aire; solo aparece en
el resultado de la velocidad relativa de la fuente y el observador. Esta es
una de las caracterı́sticas reseñada en la teorı́a especial de la relatividad
de Einstein, de acuerdo con la cual no existe ningún medio identificable
respecto al cual la velocidad de la luz tenga cierta velocidad caracterı́stica.

Solución

a) En la figura 104, que es la figura 8.12 del texto, se analiza el comporta-


miento de as ondas sonoras cuando se mueve una fuente con velocidad +u
y el observador está fijo. Es claro que la velocidad del sonido depende de
la variación de la distancia entre la fuente sonora y el observador debido a
que las distancias r0 y rn iban variando, pero en una zona lejana, esto es
donde el observador está muy retirado, se pueden aproximar las distancias
como semejantes. Esto hace que

r0 − rn ≈ S0 N = xn cos(θ)

donde se usó el hecho de que N P ∼ rn y esto llevó a que

r0 − rn ≈ unτ cos(θ)

237
Figura 104: Figura tomada del texto, Vibraciones y ondas de A. P. French, p
310.

Para el caso en el que la fuente está fija y el observador se mueve, la velo-


cidad que llega al observador varı́a según la velocidad de este. Por tanto
el análisis debe hacerse en función de las velocidades y no de las distan-
cias. Se observa en la figura 105 que a medida que el observador se mueve

Figura 105: Figura del problema 8.12a

con velocidad −u, el sonido que llega a el se ve afectado por la misma


velocidad que lleva; por tanto el fenómeno principal es una diferencia de
velocidades, tal cual se observa en el efecto Doppler. Entonces se observa
que existe una velocidad v 0 (θ) que (ver figura 105):

v 0 (θ) = v − u cos(θ)

donde se observa que solamente la componente paralela a u de la velocidad


de la onda sonora es la que afecta al sonido que percibe el observador, y

238
que para −u da:
v 0 (θ) = v + u cos(θ)
Por tanto usando la relación de dispersión v = λν se llega a
h u i
λν 0 (θ) = λν0 1 + cos(θ)
v
donde la longitud de onda es la misma porque es una onda monocromática.
Finalmente, h u i
ν 0 (θ) = ν0 1 + cos(θ)
v
b) Teniendo en cuenta los resultados anteriores, la diferencia entre las
frecuencias está dada por
ν0 h u i
ν(θ) − ν 0 (θ) = u − ν0 1 + cos(θ)
1 − v cos(θ) v
h u i −1 h u i
= ν0 1 − cos(θ) − ν0 1 + cos(θ)
v v
usando el binomio de Newton,
 i2 h u i3 
0 u hu
ν(θ) − ν (θ) = ν0 1 + cos(θ) + cos(θ) + cos(θ) + · · ·
v v v
h u i
− ν0 1 + cos(θ)
h  v
u i2 h u i3
= ν0 cos(θ) + cos(θ) + · · ·
v v

Si la relación entre las velocidades uv es pequeña v ∼ 340 m



s , se puede
hacer la aproximación a la diferencia de frecuencias como:
hu i2
ν(θ) − ν 0 (θ) ≈ ν0 cos(θ)
v
13. Un foco de sonido de frecuencia ν0 se mueve horizontalmente a una velo-
cidad constante u en la dirección x a una distancia h por encima del suelo.
Un observador está situado en el suelo en el punto x = 0; la fuente pasa
sobre dicho punto cuando t = 0.
a) Demostrar que la señal recibida en un tiempo cualquiera tR en
el suelo fue emitida por el foco en un tiempo anterior tS , tal que
 12
u2 u2
    
1 2 2 2
1 − 2 tS = tR − h 1 − 2 + u tR
v v v

b) Demostrar que la frecuencia de la señal recibida es una función


del tiempo de emisión tS , dada por
ν0
ν(tS ) = u utS
1+ v · 1
(h2 +u2 t2S ) 2

239
(La expresión para ν en función del tiempo de recepción tR es mucho más
complicada)
c) Se observa que la frecuencia del sonido recibido de uno de estos
focos es de 5500 Hz cuando la fuente está alejada y aproximándose; y
disminuye a 4500 Hz cuando la fuente está alejada y se aleja aún más.
Además, se observa que la frecuencia disminuye de 5100 Hz a 4900 Hz
durante un tiempo de 4 s cuando la fuente pasa sobre nosotros. Deducir la
velocidad y la altitud de la fuente. Aproximar lo necesario para simplificar
el cálculo. (Este tipo de análisis se utiliza para analizar la velocidad y la
altitud de los satélites terrestres a partir de la variación con el tiempo de
la frecuencia recibida de un transmisor de radio del satélite.)

Solución

a) Como la fuente emitió el sonido en un tS y habı́a recorrido antes un


tiempo tR , entonces la distancia que recorrió el sonido se determina en
una tiempo tR − tS . La fuente está a una altura h del receptor y cuando
se emitió la señal a una distancia dS habı́a transcurrido un tiempo tS . En

Figura 106: Figura del problema 8.13

la figura 106, se observa que se genera un triángulo rectángulo, que según


lo analizado anteriormente cumple la ecuación
2
h2 + d2S = v 2 (tR − tS )
pero la distancia dS depende tanto del tiempo tS coo de la velocidad de
la fuente. Por tanto,
2
h2 + u2 t2S = v 2 (tR − tS )
|{z}
d2S

al tener el tiempo tS en los dos miembros de la ecuación, se expande el


binomio resultando
h2 + u2 t2S = v 2 t2R + t2S − 2tR tS


reuniendo los tiempo en el segundo miembro de la ecuación, esta queda


como
h2 = v 2 t2R + v 2 t2S − u2 t2S − 2v 2 tR tS

240
factorizando v 2 t2S

u2
 
h2 = v 2 t2R + 1 − 2 v 2 t2S − 2v 2 tR tS
v
Para poder observar el comportamiento del movimiento de la fuente en
función del tiempo del receptor, esto es utR (esto indica qué distancia ha
viajado la fuente desde la emisión
 de la señal), se multiplica la ecuación
u2
por el factor unidimensional 1 − v2 , resultando
2
u2 u2 u2 u2
      
2
h 1− 2 = 1− 2 v 2 t2R + 1− 2 v 2 t2S − 2 1 − 2 v 2 tR tS
v v v v
que al expandir resulta el término deseado
2
u2 u2 u2
    
h2 1 − 2 = v 2 t2R − u2 t2R + 1 − 2 v 2 t2S − 2 1 − 2 v 2 tR tS
v v v

Finalmente se reunen los términos de la ecuación que contengan v 2 en un


miembro de la ecuación, quedando esta como
2
u2 u2 u2
    
h2 1 − 2 + u2 t2R = v 2 t2R + 1 − 2 v 2 t2S − 2 1 − 2 v 2 tR tS
v v v

factorizando v 2 ,
" #
2 2 2 2
     
u u u
h2 1 − 2 + u2 t2R = v 2 t2R + 1 − 2 t2S − 2 1 − 2 tR tS
v v v

donde se observa que se forma un binomio cuadrado de la forma,


 2
u2 u2
     
h2 1 − 2 + u2 t2R = v 2 tR − 1 − 2 tS
v v
para factorizar el tiempo tR se calcula la raı́z cuadrada a la ecuación dando
 12
u2 u2
      
2 2 2
h 1 − 2 + u tR = v tR − 1 − 2 tS
v v
se divide miembro a miembro entre v
 12
u2 u2
    
1 2 2 2
h 1 − 2 + u tR = tR − 1 − 2 tS
v v v
que al poner a tR en un solo miembro de la ecuación, se demuestra lo
pedido:
 12
u2 u2
    
1 2 2 2
1 − 2 tS = tR − h 1 − 2 + u tR
v v v

241
b) Al observar la figura 106, el rayo de la onda sonora que va de S a
R, tiene un tiempo de recorrido tR − tS , que desde su creación con una
frecuencia ν0 avanza hasta llegar al receptor con una frecuencia ν(tS ), pero
el tiempo depende de la relación entre las velocidades del sonido v y del
movimiento de la fuente
 u; este factor ya conocido desde la parte a) del
2
problema es 1 − uv2 y solo afecta al tiempo tS porque esta es la que se
mueve. Por tanto se debe verificar que el porducto de la diferencia de los
tiempos con ν0 debe ser igual al producto de la diferencia de los tiempos
transformados con ν(tS ). Esto es,

u2
   
ν(tS ) tR − 1 − 2 tS = ν0 (tR − tS )
v

Para hallar la frecuencia se factorizan los tiempos resultando

u2
 
ν0 tS − ν(tS ) 1 − 2 tS = ν0 tR − ν(tS )tR
v

multiplicando la ecuación por v 2 ,

u2
 
ν(tS ) 2 + ν0 − ν(tS ) v 2 tS = [ν0 − ν(tS )] v 2 tR
v

resulta

ν(tS )u2 tS + ν0 v 2 tS − ν(tS )v 2 tS = ν0 v 2 tR − ν(tS )v 2 tR

separando las velocidades u y v en los miembros de la ecuación,

ν(tS )u2 tS = ν0 v 2 tR − ν0 v 2 tS − ν(tS )v 2 tR + ν(tS )v 2 tS

donde se observa que se puede factorizar la diferencia de tiempos (tR − tS )


de la forma

ν(tS )u2 tS = ν0 v 2 (tR − tS ) − ν(tS )v 2 (tR − tS )

quedando solamente despejar la frecuencia. Por tanto,

ν(tS ) v 2 (tR − tS ) + u2 tS = ν0 v 2 (tR − tS )


 

y entonces,
ν0 v 2 (tR − tS )
ν(tS ) =
v 2 (tR − tS ) + u2 tS
ya solo queda acomodar el resultado a lo que pide el problema. Dividiendo
la fracción entre v 2 (tR − tS ) para el numerador y el denominador se llega
a
ν0
ν(tS ) = 2
1 + v2 (tuRt−t
S
S)

242
 21
donde se puede hacer el reemplazo v (tR − tS ) por h2 + u2 t2S (ver la
figura 106), quedando
ν0
ν(tS ) = u utS
1+ v · 1
(h2 +u2 t2S ) 2

que era lo que se querı́a demostrar.

c)

243

También podría gustarte